Combined Chemistry
Combined Chemistry
Combined Chemistry
SECONDARY 4
Express Exam Paper
Science Chemistry
1 CHIJ Katong Convent t SA1
2 Mayflower Secondary
Secondaary SA1
SA1
3 Pasir Ris Crest
Cr
restt SA1
4 Psir Ris SSecondary
ecoondary SA1
5 West Spring
Spr
ring Secondary
Seconddary SA1
6 Bedok
k Green
Gre
een Secondary
Se
econdary SA2
7 Bedok
Be
edok South
South Secondary
Secondary SA2
8 Bendemeer
Bend
demeer Secondary
Secondary SA2
9 Bukit
Bukit Batok
Batok Secondary SA2
10 Geylang
Gey
ylang Methodist SA2
www.KiasuExamPaper.com
1
www.KiasuExamPaper.com
2
Name: _____________________________( ) Class:______
There are forty questions on this paper. Answer all questions. For each question there are four possible
answers, A, B, C and D.
Choose the one you consider correct and record your choice in soft pencil on the separate Optical
Answer Sheet.
Complete the Chemistry and Biology sections on two separate Optical Answer Sheets provided.
Each correct answer will score one mark. A mark will not be deducted for a wrong answer.
Any rough working should be done in this booklet.
3
CHIJ Katong Convent Mid-Year Exams 2018 Science (Chemistry, Biology) 5078/01 Sec 4E/5N(A)
X Y Z
A X to Y
B Y to Z
C Z to X
D Z to Y
Which row describes the correct apparatus for the measurement made?
3 Which method of separation should be used to obtain pure water from copper(II) sulfate solution?
A crystallisation
B evaporation to dryness
C filtration
D simple distillation
www.KiasuExamPaper.com
2 [Turn over
4
CHIJ Katong Convent Mid-Year Exams 2018 Science (Chemistry, Biology) 5078/01 Sec 4E/5N(A)
4 A food colouring is compared with four different dyes. The chromatogram produced is shown in the
diagram.
X X X X X
food dye 1 dye 2 dye 3 dye 4
colouring
A 1 and 2 only
B 1 and 3 only
C 2 and 3 only
D 2 and 4 only
A 56 qC
B 78 qC
C 100 qC
D 104 qC
A B C D
www.KiasuExamPaper.com
3 [Turn over
5
CHIJ Katong Convent Mid-Year Exams 2018 Science (Chemistry, Biology) 5078/01 Sec 4E/5N(A)
What is the total volume of gas remaining at the end of the reaction?
A 20 cm3
B 40 cm3
C 60 cm3
D 80 cm3
10 Due to acid rain, the acidity of the soil is increased, making it unsuitable for plant growth.
A calcium carbonate
B calcium hydroxide
C calcium nitrate
D calcium sulfate
11 An unknown oxide was added separately to hydrochloric acid and aqueous sodium hydroxide. The pH
of the resulting solution was measured and shown in the table.
A aluminium oxide
B carbon monoxide
C potassium oxide
D sulfur dioxide
www.KiasuExamPaper.com
4 [Turn over
6
CHIJ Katong Convent Mid-Year Exams 2018 Science (Chemistry, Biology) 5078/01 Sec 4E/5N(A)
A XY 5
B XY 3
C XY
D X3Y
electrical
substance density/ g/dm3 melting point/ qC conductivity in
solid state
J 2.1 115 poor
K 5.7 232 good
L 6.3 1326 poor
M 19.3 1064 good
A J and K only
B J and L only
C K and M only
D L and M only
www.KiasuExamPaper.com
5 [Turn over
7
CHIJ Katong Convent Mid-Year Exams 2018 Science (Chemistry, Biology) 5078/01 Sec 4E/5N(A)
halide solution
halogen X Y Z
added
X 9 8 Key:
Y 8 8 9 visible reaction
Z 9 9 8 no visible reaction
lowest highest
A X Y Z
B Y X Z
C Y Z X
D Z X Y
16 Solid hydrated sodium carbonate was added to aqueous citric acid. The mixture was stirred and the
temperature was recorded every 10 seconds. The results are shown on the graph
temperature/ qC
time/ s
Which row describes the reaction?
www.KiasuExamPaper.com
6 [Turn over
8
CHIJ Katong Convent Mid-Year Exams 2018 Science (Chemistry, Biology) 5078/01 Sec 4E/5N(A)
A condensation
B freezing
C photosynthesis
D rusting
18 The effect of temperature on the rate of the reaction between zinc and hydrochloric acid can be
investigated by measuring the production of gas.
A condenser
B gas syringe
C stopwatch
D thermometer
A VO 2 ĺ9 2 O 3
B V 2 O 5 ĺ92 2
C V 2 O 3 ĺ92
D V 2 O 3 ĺ9 2 O 5
air
water
Which diagram best shows the level of the water at the end of the week?
A B C D
www.KiasuExamPaper.com
7 [Turn over
9
CHIJ Katong Convent Mid-Year Exams 2018 Science (Chemistry, Biology) 5078/01 Sec 4E/5N(A)
Data Sheet
Colours of Some Common Metal Hydroxides
www.KiasuExamPaper.com
8 [Turn over
10
CHIJ Katong Convent Mid-Year Exams 2018 Science (Chemistry, Biology) 5078/01 Sec 4E/5N(A)
www.KiasuExamPaper.com
9 [Turn over
11
Name: _____________________________( ) Class:______
Write your name, class and registration number on all the work you hand in.
Write in dark blue or black pen on both sides of the paper.
You may use a soft pencil for any diagrams or graphs.
Do not use staples, paper clips, highlighters, glue or correction fluid/ tape.
Section A
Answer all questions.
Write your answers in the spaces provided on the Question Paper.
Section B
Answer any two questions.
Write your answers in the spaces provided on the Question Paper.
www.KiasuExamPaper.com
This question paper consists of 16 printed pages.
[Turn over
12
CHIJ Katong Convent Mid-Year Exams 2018 Science (Chemistry) 5078/03 Sec 4E/5N(A)
Table 1.1
classification atoms
substance
(element, compound or mixture) found within the substance
hydrogen sulfide compound hydrogen, sulfur
brass
limestone compound
[3]
[Total: 3]
2 Iron is the fourth most common element in the Earth's crust and it is also believed to form a large
extent of the Earth's core.
(a) Pure iron can be prepared by the thermal decomposition of iron pentacarbonyl.
Fig. 2.1 shows the structure of iron pentacarbonyl.
Fig. 2.1
Predict the electrical conductivity of this compound by including the condition under
which conductivity is observed or not at all.
……………………………………………………………………………………………………..
…………………………………………………………………………………………………. [1]
www.KiasuExamPaper.com
2 [Turn over
13
CHIJ Katong Convent Mid-Year Exams 2018 Science (Chemistry) 5078/03 Sec 4E/5N(A)
2 (b) (ii) Complete Table 2.1 to show the number of electrons, neutrons and protons in iron(II) ion
and oxide ion.
Table 2.1
16
8 O 2- 8
[2]
(c) (i) 54 56
26 Fe and 26 Fe are two common isotopes of iron.
Define isotopes.
………………………………………………………………………………………………….….
…………………………………………………………………………………………………. [1]
(ii) These iron isotopes have different physical properties but exhibit same chemical
properties.
………………………………………………………………………………………………….….
…………………………………………………………………………………………………. [1]
[Total: 6]
3 When a colourless solution of copper(I) chloride is left in a beaker for a period of time, the following
reaction takes place.
(b) Explain, in terms of change in oxidation states, why CuCl is both oxidised and reduced in this
reaction.
………………………………………………………………………………………….………………...
………………………………………………………………………………………….……………. [2]
www.KiasuExamPaper.com
3 [Turn over
14
CHIJ Katong Convent Mid-Year Exams 2018 Science (Chemistry) 5078/03 Sec 4E/5N(A)
………………………………………………………………………………………….………………...
………………………………………………………………………………………….……………. [1]
[Total: 5]
4 (a) Name the pieces of apparatus most suitable to complete the following laboratory procedures:
…………………………………………………………………………………...…….…...…..[1]
…………………………………………………………………………………...…….…...…..[1]
…………………………………………………………………………………...…….…...…..[1]
…………………………………………………………………………………...…….…...…..[1]
(b) Chromatography can be used to separate the coloured pigments extracted from lavender
flowers. The apparatus used is shown Fig. 4.1.
After a few minutes, the solvent vapour fills the whole chromatography jar.
lid
chromatography paper
Fig. 4.1
Describe what happens to the movement and arrangement of the solvent particles as they
become a vapour.
………………………………………………………………………………………….………………...
………………………………………………………………………………………….………………...
………………………………………………………………………………………….………………...
………………………………………………………………………………………….……………. [2]
[Total: 6]
www.KiasuExamPaper.com
4 [Turn over
15
CHIJ Katong Convent Mid-Year Exams 2018 Science (Chemistry) 5078/03 Sec 4E/5N(A)
5 A solution of nitric acid is prepared by diluting 0.15 mol to make 100 cm3 of solution.
(b) The chemical equation for the reaction between nitric acid and potassium carbonate is as
follows:
2HNO 3 + K 2 CO 3 J 2KNO 3 + CO 2 + H 2 O
100 cm3 of 0.5 mol/dm3 nitric acid is added to an aqueous solution containing 0.02 mol of
potassium carbonate.
…………………………………………………………………………………...…….…...…..[1]
[Total: 5]
www.KiasuExamPaper.com
5 [Turn over
16
CHIJ Katong Convent Mid-Year Exams 2018 Science (Chemistry) 5078/03 Sec 4E/5N(A)
6 Fig. 6.1 describes some of the properties and reactions of several substances.
P
black solid
neutralise with
solution Q
R
blue aqueous
solution
add dilute nitric
acid and aqueous
add aqueous
silver nitrate
sodium hydroxide
S white precipitate
blue precipitate
Fig. 6.1
P …………………………………………
Q …………………………………………
R …………………………………………
S …………………………………………...... [4]
(b) Write the ionic equation for the reaction of R with aqueous silver nitrate.
............................................................................................................................................... [1]
[Total: 5]
www.KiasuExamPaper.com
6 [Turn over
17
CHIJ Katong Convent Mid-Year Exams 2018 Science (Chemistry) 5078/03 Sec 4E/5N(A)
7 (a) Lithium, sodium and potassium belong to Group I of the Periodic Table.
Table 7.1 shows the observations when these three metals react with water.
Table 7.1
(i) Describe and explain the reactivity of Group I metals down the group.
………………………………………………….…………………………………………………..
...……………………………………………………………………………………………………
...……………………………………………………………………………………………………
...……………………………………………………………………………………………………
.…………………………………………………………………………………...…….…….....[3]
...……………………………………………………………………………………………………
.…………………………………………………………………………………...…….…….....[1]
(iii) Name the gas evolved when Group I metals react with water.
.…………………………………………………………………………………...…….…….....[1]
...……………………………………………………………………………………………………
.…………………………………………………………………………………...…….…….....[1]
(ii) Using your knowledge of electronic structures, explain why elements in Group 0 are
unreactive.
...……………………………………………………………………………………………………
...……………………………………………………………………………………………………
.…………………………………………………………………………………...…….…….....[1]
[Total: 7]
www.KiasuExamPaper.com
7 [Turn over
18
CHIJ Katong Convent Mid-Year Exams 2018 Science (Chemistry) 5078/03 Sec 4E/5N(A)
8 The petrol burnt in car engines react with air to form a mixture of gases.
Table 8.1 shows the composition of the mixture of all the gases coming from car exhaust fumes.
Table 8.1
(b) The amount of carbon dioxide emitted by vehicles contributes to the increasing concentration
of the gas in the atmosphere.
...........……………………………………………………………………………………………………
...........……………………………………………………………………………………………………
...........……………………………………………………………………………………………………
.…………….…………………………………………………………………………...…….…….....[2]
...........……………………………………………………………………………………………………
.…………….…………………………………………………………………………...…….…….....[1]
www.KiasuExamPaper.com
8 [Turn over
19
CHIJ Katong Convent Mid-Year Exams 2018 Science (Chemistry) 5078/03 Sec 4E/5N(A)
8 (d) Water is one of the major by-products in the combustion of petrol in vehicles.
Draw a ‘dot and cross’ diagram of water, showing only the arrangement of electrons on the
valence shells.
[2]
(e) The combustion of petrol is exothermic.
.…….…………………….……………………………………………………...…….……..... [1]
.…….…………………….……………………………………………………...…….……..... [1]
[Total: 8]
www.KiasuExamPaper.com
9 [Turn over
20
CHIJ Katong Convent Mid-Year Exams 2018 Science (Chemistry) 5078/03 Sec 4E/5N(A)
9 Magnesium sulfate is formed from the reaction between a metal, M and an acid, N.
M ……………………………………….
N ....................................................... [2]
(b) Write the balanced chemical equation for the reaction between M and N.
.................................................................................................................................................... [1]
(c) Describe how pure crystals of magnesium sulfate can be prepared using metal M and acid N.
.........................................................................................................................................................
.........................................................................................................................................................
.........................................................................................................................................................
.........................................................................................................................................................
.........................................................................................................................................................
.........................................................................................................................................................
.........................................................................................................................................................
.................................................................................................................................................... [4]
(d) Magnesium sulfate can also be prepared using acid N and another substance.
.................................................................................................................................................... [1]
www.KiasuExamPaper.com
10 [Turn over
21
CHIJ Katong Convent Mid-Year Exams 2018 Science (Chemistry) 5078/03 Sec 4E/5N(A)
9 (e) The labels on two bottles, one containing acid N and the other containing aqueous ammonia, were
missing.
(i) Briefly describe a method you would use to distinguish between the two solutions.
.................................................................................................................................................
............................................................................................................................................ [1]
(ii) State the result you would expect for acid N using the method described in (e)(i).
.................................................................................................................................................
.............................................................................................................................................[1]
[Total: 10]
10 Iron is a metal that is commonly used in the construction of ships and bridges.
(a) Iron is extracted from haematite using carbon in a blast furnace. Impurities from the iron are
removed using limestone.
Describe how limestone is used to remove impurities from iron and include suitable chemical
equations in your answer.
.........................................................................................................................................................
.........................................................................................................................................................
.........................................................................................................................................................
.........................................................................................................................................................
.........................................................................................................................................................
.........................................................................................................................................................
.........................................................................................................................................................
.................................................................................................................................................... [4]
(b) When iron is exposed to the environment for some time, it starts to rust.
.................................................................................................................................................
............................................................................................................................................ [1]
www.KiasuExamPaper.com
11 [Turn over
22
CHIJ Katong Convent Mid-Year Exams 2018 Science (Chemistry) 5078/03 Sec 4E/5N(A)
10 (b) (ii) Some ships that are made of iron prevent rusting by attaching blocks of zinc to its surface.
After some time, it was observed the block of zinc corroded instead of iron.
Explain how attaching blocks of zinc help to prevent the ship from rusting.
.................................................................................................................................................
............................................................................................................................................. [1]
(iii) Predict what happens when blocks of silver metal are attached to the iron surface of the ship
instead of zinc.
............................................................................................................................................. [1]
(iv) It was observed that ships in the sea tend to corrode more quickly than bridges.
.................................................................................................................................................
............................................................................................................................................. [1]
(c) In addition to the production of iron using the blast furnace, iron is also obtained through recycling.
1.......................................................................................................................................................
.........................................................................................................................................................
2.......................................................................................................................................................
.................................................................................................................................................... [2]
[Total: 10]
www.KiasuExamPaper.com
12 [Turn over
23
CHIJ Katong Convent Mid-Year Exams 2018 Science (Chemistry) 5078/03 Sec 4E/5N(A)
11 Egg shells are made up mainly of calcium carbonate. A pupil carried out an experiment to react egg
shells with excess dilute hydrochloric acid. The gas that was produced was measured at a regular time
interval to investigate the speed of the reaction.
.................................................................................................................................................. [1]
(b) Complete the diagram in Fig. 11.1 to show the apparatus which could be used to measure the
volume of gas produced.
Table 11.1
(i) Plot the results on Fig. 11.2 and draw a smooth curve through the points.
volume/ cm3
60
50
40
30
20
10
time/ s
0
0 20 40 60 80 100 120 140 160 180 200 220 240
www.KiasuExamPaper.com
13 [Turn over
24
CHIJ Katong Convent Mid-Year Exams 2018 Science (Chemistry) 5078/03 Sec 4E/5N(A)
11 (c) (ii) Explain why no further measurements were taken after 220 seconds.
.................................................................................................................................................
............................................................................................................................................ [1]
(iii) Using the graph drawn in (c)(i), estimate the volume of gas evolved for the first 100 seconds.
............................................................................................................................................. [1]
(iv) Calculate the average speed of reaction in cm3/ s for the first 10 seconds of the reaction.
final volume - initial volume
(Average speed = )
duration concerned
[Total: 10]
www.KiasuExamPaper.com
14 [Turn over
25
CHIJ Katong Convent Mid-Year Exams 2018 Science (Chemistry) 5078/03 Sec 4E/5N(A)
Data Sheet
www.KiasuExamPaper.com
15 [Turn over
26
CHIJ Katong Convent Mid-Year Exams 2018 Science (Chemistry) 5078/03 Sec 4E/5N(A)
www.KiasuExamPaper.com
16 [Turn over
27
www.KiasuExamPaper.com
28
CHIJ Katong Convent
Answer scheme
Paper 1
1 2 3 4 5 6 7 8 9 10
B A D C D D C D C B
11 12 13 14 15 16 17 18 19 20
A D C A B A C A D B
Paper 3
Section A
Qn Answers
1
classification atoms
atoms s
substance (element, compound ffo
oun
foundnd within
withhin
n the
the
or mixture) substa
anc
nce
substance e
hydrogen sulfide compound d hydrogen,
hydr
hy d og
o en, sulfur
sulf
lfur
lf ur
brass mixture
mixturre c
co
copper,
opp
pper
er, zi
er zinc
nc
calc ciu
um, carbon,
calcium, car
arbo
ar bon
bo n,
limestone compound
ox en
oxygen n
2a Fe(CO) 5
2b(ii)
numb
nu mber
numberer o
off nu
number of number of
prot
pr oto
ons
protonss neutrons electrons
ହ
ଶ ݁ܨଶଶା
ା 2
26 30 24
ଵ
଼ ܱ ଶଶି
ି 8 8 10
2c(ii) As the isotopes have the same number of valence electrons, they possess the same
chemical properties.
www.KiasuExamPaper.com
1
29
3c The colourless solution turns blue
OR
A pink/ brown/ reddish-brown solid is formed.
4a(ii) burette
7ai
7a
ai The reactivity
ty of
of Group
Grou
Gr o p I me
meta
metals
tals
ls increases down the group.
Down thhe gr
the grou
o p, tthere
group, here
he re are mo
are m re filled electron shells between the nucleus and the
more
valence electron.
electron
on.
Hence
ce
e, there
Hence, ther
th e e is a gr
er g eater tendency to lose the valence electron to attain the noble
greater
gas electronic
e ec
el ectr
tron
onic
ic configuration.
configuration.
7aiii
7 iii Hydrogen
H d gas
(any one)
www.KiasuExamPaper.com
2
30
7bii They have fully-filled valence electron shells and already achieved a stable noble
gas electronic configuration.
8a nitrogen/ N 2
increase
ed flooding
This results in ice caps melting (or rise in sea levels) / increased floo
fl ooding /
desertification / increased death of corals.
8d
Key
K
Ke
ey
electron
: eel
lectron from
n fr
ro m O
ro
X : electro on fro
on
electron ro
Section B
Qn Answers
9a M: magne
magnesium
esi
s um
N ssulfuric
N: u fu
ul uri
ricc ac
acid
id
9b Mg + H 2 SO 4 Æ MgSO 4 + H 2
2. Filter to obtain the filtrate, which is magnesum sulfate solution, and remove the
excess magnesium metal residue.
5. Filter to collect the crystals. Wash the crystals with a little cold distilled water to
remove impurities and dry between sheets of filter paper.
9ei Add a few drops of universal indicator solution into each solution. OR
Dip a piece of red and blue litmus paper into each solution.
The red litmus paper will remain red and the blue litmus paper will turn red.
10a Limestone is first decomposed by heat to produce carbon dioxide and calcium
oxide.
CaCO 3 (s) J CaO (s) + CO 2 (g)
www.KiasuExamPaper.com
3
31
Calcium oxide reacts with the impurities from iron, which is sand, to form molten
slag.
CaO (s) + SiO 2 (s) J CaSiO 3 (l)
10bi Paint serves as a protective layer that prevents iron from coming into contact with
water and oxygen.
10biv The presence of sodium chloride in seawater results in the increase of the speed of
rusting.
(any two)
11c(i)
11c(ii) All the egg shell (calcium carbonate) had been used up.
www.KiasuExamPaper.com
4
32
11c(iv) Based on students’ graph
௩௨ ௧ ଵ ௦ ି ௩௨ ௧ ௦
average speed =
ଵ ௦
www.KiasuExamPaper.com
5
33
www.KiasuExamPaper.com
34
Name Reg. No Class
4EX/5NA
MAYFLOWER SECONDARY SCHOOL MAYFLOWER SECONDARY SCHOOL MAYFLOWER SECONDARY SCHOOL MAYFLOWER SECONDARY SCHOOL
MAYFLOWER SECONDARY SCHOOL MAYFLOWER SECONDARY SCHOOL MAYFLOWER SECONDARY SCHOOL MAYFLOWER SECONDARY SCHOOL
MAYFLOWER SECONDARY SCHOOL MAYFLOWER SECONDARY SCHOOL MAYFLOWER SECONDARY SCHOOL MAYFLOWER SECONDARY SCHOOL
MAYFLOWER SECONDARY SCHOOL MAYFLOWER SECONDARY SCHOOL MAYFLOWER SECONDARY SCHOOL MAYFLOWER SECONDARY SCHOOL
MAYFLOWER SECONDARY SCHOOL MAYFLOWER SECONDARY SCHOOL MAYFLOWER SECONDARY SCHOOL MAYFLOWER SECONDARY SCHOOL
MAYFLOWER SECONDARY SCHOOL MAYFLOWER SECONDARY SCHOOL MAYFLOWER SECONDARY SCHOOL MAYFLOWER SECONDARY SCHOOL
MAYFLOWER SECONDARY SCHOOL MAYFLOWER SECONDARY SCHOOL MAYFLOWER SECONDARY SCHOOL MAYFLOWER SECONDARY SCHOOL
MAYFLOWER SECONDARY SCHOOL MAYFLOWER SECONDARY SCHOOL MAYFLOWER SECONDARY SCHOOL MAYFLOWER SECONDARY SCHOOL
MAYFLOWER SECONDARY SCHOOL MAYFLOWER SECONDARY SCHOOL MAYFLOWER SECONDARY SCHOOL MAYFLOWER SECONDARY SCHOOL
MAYFLOWER SECONDARY SCHOOL MAYFLOWER SECONDARY SCHOOL MAYFLOWER SECONDARY SCHOOL MAYFLOWER SECONDARY SCHOOL
Additional Materials:
Electronic calculator
OTAS Answer Sheet
INSTRUCTIONS TO CANDIDATES:
Do not open this booklet until you are told to do so.
Write your name, index number and class in the spaces at the top of this page and on any separate
answer paper used.
Write in soft pencil.
You may use a soft pencil for any diagrams, graphs, tables or rough working.
Do not use staples, paper clips, highlighters, glue or correction fluid.
At the end of the examination, hand in your OTAS sheet and question paper separately.
www.KiasuExamPaper.com
35
2
21 A student follows the rate of the reaction between marble chips, CaCO 3 , and
dilute hydrochloric acid, by measuring the amount products produced or the
amount of reactants reacted.
A 1 and 2
B 2 and 4
C 1, 2 and 4
D All of the above
www.KiasuExamPaper.com
36
3
Which part of the cooling curve below shows that both the gas and liquid exist
together?
Temp /°C
B
D
Time /min
23 Four samples are spotted onto chromatography paper. It is known that one of
these samples is pure compound Q. A separate sample of pure compound Q is
also spotted onto the paper. The paper is placed in a solvent.
R Q R
www.KiasuExamPaper.com
38
5
What is the maximum volume of nitrogen dioxide that could be obtained when
1 dm3 of nitrogen monoxide reacts with 2 dm3 of oxygen?
A 1.0 dm3
B 2.0 dm3
C 3.0 dm3
D 4.0 dm3
A pH 2
B pH 7
C pH 10
D pH 14
31 The table shows the results of adding dilute nitric acid and aqueous sodium
hydroxide to four oxides.
A reaction reaction
B reaction no reaction
C no reaction reaction
D no reaction no reaction
33 Which reagent can be used to react with dilute hydrochloric acid to prepare
silver chloride?
A solid silver
B solid silver oxide
C solid silver carbonate
D aqueous silver nitrate
34 The results of experiments involving four metals, W, X, Y and Z, and their ions
are shown.
What is the order of reactivity of the four metals, most reactive to least
reactive?
35 Element Z is in the same group of the Periodic Table as bromine but has a
lower boiling point.
www.KiasuExamPaper.com
40
7
In all four experiments the same volume of nitric acid was used.
Its concentration, or temperature, or both concentration and temperature, were
changed.
39 The inside of a tube is coated with iron filings. The tube is placed in a trough of
Water as shown.
Which diagram represents the likely appearance of the apparatus after one
week?
A carbon monoxide
B methane
C oxides of nitrogen
D sulfur dioxide
-- End of paper 1 --
www.KiasuExamPaper.com
42
9
43
www.KiasuExamPaper.com
Name Reg. No Class
4EX/5NA
MAYFLOWER SECONDARY SCHOOL MAYFLOWER SECONDARY SCHOOL MAYFLOWER SECONDARY SCHOOL MAYFLOWER SECONDARY SCHOOL
MAYFLOWER SECONDARY SCHOOL MAYFLOWER SECONDARY SCHOOL MAYFLOWER SECONDARY SCHOOL MAYFLOWER SECONDARY SCHOOL
MAYFLOWER SECONDARY SCHOOL MAYFLOWER SECONDARY SCHOOL MAYFLOWER SECONDARY SCHOOL MAYFLOWER SECONDARY SCHOOL
MAYFLOWER SECONDARY SCHOOL MAYFLOWER SECONDARY SCHOOL MAYFLOWER SECONDARY SCHOOL MAYFLOWER SECONDARY SCHOOL
MAYFLOWER SECONDARY SCHOOL MAYFLOWER SECONDARY SCHOOL MAYFLOWER SECONDARY SCHOOL MAYFLOWER SECONDARY SCHOOL
MAYFLOWER SECONDARY SCHOOL MAYFLOWER SECONDARY SCHOOL MAYFLOWER SECONDARY SCHOOL MAYFLOWER SECONDARY SCHOOL
MAYFLOWER SECONDARY SCHOOL MAYFLOWER SECONDARY SCHOOL MAYFLOWER SECONDARY SCHOOL MAYFLOWER SECONDARY SCHOOL
MAYFLOWER SECONDARY SCHOOL MAYFLOWER SECONDARY SCHOOL MAYFLOWER SECONDARY SCHOOL MAYFLOWER SECONDARY SCHOOL
MAYFLOWER SECONDARY SCHOOL MAYFLOWER SECONDARY SCHOOL MAYFLOWER SECONDARY SCHOOL MAYFLOWER SECONDARY SCHOOL
MAYFLOWER SECONDARY SCHOOL MAYFLOWER SECONDARY SCHOOL MAYFLOWER SECONDARY SCHOOL MAYFLOWER SECONDARY SCHOOL
INSTRUCTIONS TO CANDIDATES:
Do not open this booklet until you are told to do so. FOR EXAMINER’S USE
Write your name, index number and class in the spaces at the top of Section Marks
this page and on any separate answer paper used.
Paper 1
Write in dark blue or black pen on both sides of the paper. You may / 20
MCQ
use a soft pencil for any diagrams, graphs or tables or rough working.
Paper 3
Do not use staples, paper clips, highlighters, glue or correction fluid. / 45
Section A
Paper 3
/ 20
Section B
Section A Paper 5 /15
Answer all questions.
Write your answers in the spaces provided on the question paper. Total / 100
Section B
Answer all questions on the spaces provided.
Answers any two questions out of the three questions given.
At the end of the examination, fasten all your work securely together.
The number of marks is given in brackets [ ] at the end of each question or part question.
A copy of the Periodic Table is printed on page 13.
The use of an approved scientific calculator is expected, where appropriate.
www.KiasuExamPaper.com
44
2
Paper 3
1 The apparatus shown in Fig 1.1 can be used to separate a mixture of 3 liquids, A, B and
water.
A has a boiling point of 50 °C while B has a boiling
g point of 78 °C.
Thermometer
condenser
Mixture
Fig. 1.1
……………………………………………………………………………………………..[1]
……………………………………………………………………………………………..[1]
(c) Predict the temperature of the thermometer when the first distillate appears in the
beaker.
Explain why.
………………………………………………………………………………………………...
………………………………………………………………………………………………...
……………………………………………………………………………………………..[2]
www.KiasuExamPaper.com
45
3
2 Table 2.1 shows the number of protons, electrons and neutrons of five particles Q to V.
Table 2.1
Q 5 5 4
R 7 7 10
S 8 8 8
T 9 11 9
U 10 10 10
V 16 16 16
Which of the particles, Q to V in Table 2.1, fit each of the following descriptions?
………………………………………………………………………………………………...
………………………………………………………………………………………………...
………………………………………………………………………………………………...
……………………………………………………………………………………………..[2]
A
B
Fig. 3.1
……………………………………………………………………………………...[1]
(ii) With the aid of a chemical equation, describe how your answer in 3(a)(i)
reduces haematite to molten iron.
………………………………………………………………………………………...
………………………………………………………………………………………...
………………………………………………………………………………………..
……………………………………………………………………………………..[3]
(iii) Besides haematite, name the other 2 raw materials that are added to the
Blast Furnace.
……………………………………………………………………………………..[2]
(b) Name product A and state its usefulness as a substance floating above product B.
………………………………………………………………………………………………...
……………………………………………………………………………………………..[2]
www.KiasuExamPaper.com
47
5
Stainless steel can be made by adding elements such as chromium and nickel to
iron to improve its strength.
……………………………………………………………………………………...[1]
(ii) Explain, in terms of the arrangement of atoms, why stainless steel is harder
than pure iron.
………………………………………………………………………………………...
……………………………………………………………………………….............
……………………………………………………………………………….............
……………………………………………………………………………………..[2]
4 A student titrates 25.0 cm3 an alkali of metal X, XOH, with sulfuric acid.
He realizes that 20.0 cm3 of 0.2 mol/dm3 of sulfuric acid is required to neutralize the acid
fully.
2XOH + H 2 SO 4 Æ X 2 SO 4 + 2H 2 O
(a) Name an indicator that can determine the endpoint of the reaction and describe the
colour change seen.
………………………………………………………………………………………………...
……………………………………………………………………………………………..[2]
(b) (i) Calculate the number of moles present in 20.0 cm3 of the sulfuric acid used.
(iii) If the concentration of XOH used is 12.8 g/dm3, calculate the relative mass
of XOH and, hence, determine the identity of X.
Name the gas formed and describe how to test for its identity.
………………………………………………………………………………………..............
………………………………………………………………………………………..............
…………………………………………………………………………………….............[2]
www.KiasuExamPaper.com
49
7
For example, it can form lithium hydride with lithium and also ammonia with nitrogen.
……………………………………………………………………………………………..[1]
(b) (i) Draw the dot-and-cross diagram to show the arrangement of valence
electrons found in lithium hydride and ammonia in the space below.
Lithium hydride:
[2]
Ammonia:
[2]
(ii) Explain, in terms of bonding, why lithium hydride exist as a solid while
ammonia exist as a gas at room temperature.
………………………………………………………………………………………...
………………………………………………………………………………………...
………………………………………………………………………………………...
………………………………………………………………………………………...
……………………………………………………………………………………...[3]
nitric acid
metal A green solution B + gas C
aqueous
sodium
hydroxide
Fig 6.1
A ……………………………
B ……………………………
C ……………………………
D ……………………………
E ……………………………
F ……………………………
[6]
(b) Describe how to test for gas C that is formed in the above reactions.
………………………………………………………………………………………………..
……………………………………………………………………………………………..[1]
www.KiasuExamPaper.com
51
9
7 (a) (i) Name an element from Period 3 and explain how the electronic structure of this
element can be used to determine the group the element belongs.
……………………………………………………………………………………………
……………………………………………………………………………………………
……………………………………………………………………………………………
……………………………………………………………………………………………
…………………………………………………………………………………………[3]
(ii) Moving from Group I to Group VII across period 3, the character of the
elements change.
……………………………………………………………………………………………
……………………………………………………………………………………………
……………………………………………………………………………………………
……………………………………………………………………………………………
…………………………………………………………………………………………[3]
(b) The element with an atomic number of 87 is extremely rare and only about 30 g exist
throughout the Earth crust.
…………………………………………………………………………………………………...
…………………………………………………………………………………………………...
…………………………………………………………………………………………………...
…………………………………………………………………………………………………...
…………………………………………………………………………………………………...
…………………………………………………………………………………………………...
………………………………………………………………………………………………..[4]
8 Coal contains sulfur. When coal is burnt at power stations in an excess of oxygen, sulfur
dioxide is formed according to the reaction shown below.
S + O 2 Æ SO 2
……………………………………………………………………………………………
…………………………………………………………………………………………[1]
(ii) Find the mass of sulfur burnt if 320 dm3 of sulfur dioxide is formed at room
temperature and pressure.
[3]
(iii) Describe how the release of sulfur dioxide can indirectly cause damage to
buildings made of limestone.
……………………………………………………………………………………………
……………………………………………………………………………………………
…………………………………………………………………………………………[2]
(b) Two pollutants can be produced in the internal combustion engines of automobiles.
Name the pollutants and describe how they are produced in the engines of
automobiles.
…………………………………………………………………………………………………...
…………………………………………………………………………………………………...
…………………………………………………………………………………………………...
…………………………………………………………………………………………………...
…………………………………………………………………………………………………...
……………………………………………………………………………………….............[4]
www.KiasuExamPaper.com
53
11
9 (a) Explain, in terms of collision theory, how the temperature of reactants affect the
speed of reaction.
…………………………………………………………………………………………………………….
…………………………………………………………………………………………………………….
…………………………………………………………………………………………………………[2]
(b) A student wants to investigate the rate of reaction involving particle size.
Given that he has magnesium strips and magnesium powder with some hydrochloric
acid, describe how he can conduct a laboratory experiment to do his investigation.
Your description should include the measurement obtained to measure the rate of
reaction.
…………………………………………………………………………………………………...
…………………………………………………………………………………………………...
…………………………………………………………………………………………………...
…………………………………………………………………………………………………...
…………………………………………………………………………………………………...
…………………………………………………………………………………………………...
…………………………………………………………………………………………………...
…………………………………………………………………………………………………...
………………………………………………………………………………………………..[5]
(c) Magnesium can also react with copper(II) sulfate as shown below.
Mg + CuSO 4 Æ MgSO 4 + Cu
…………………………………………………………………………………………[1]
……………………………………………………………………………………………
……………………………………………………………………………………………
…………………………………………………………………………………………[2]
-- End of section B --
-- End of paper –
Blank Page
www.KiasuExamPaper.com
55
56
13
www.KiasuExamPaper.com
57
www.KiasuExamPaper.com
Name Reg. No Class
4EX5NA
FLOWER SECONDARY SCHOOL MAYFLOWER SECONDARY SCHOOL MAYFLOWER SECONDARY SCHOOL MAYFLOWER SECONDARY SCHOOL
FLOWER SECONDARY SCHOOL MAYFLOWER SECONDARY SCHOOL MAYFLOWER SECONDARY SCHOOL MAYFLOWER SECONDARY SCHOOL
FLOWER SECONDARY SCHOOL MAYFLOWER SECONDARY SCHOOL MAYFLOWER SECONDARY SCHOOL MAYFLOWER SECONDARY SCHOOL
FLOWER SECONDARY SCHOOL MAYFLOWER SECONDARY SCHOOL MAYFLOWER SECONDARY SCHOOL MAYFLOWER SECONDARY SCHOOL
FLOWER SECONDARY SCHOOL MAYFLOWER SECONDARY SCHOOL MAYFLOWER SECONDARY SCHOOL MAYFLOWER SECONDARY SCHOOL
FLOWER SECONDARY SCHOOL MAYFLOWER SECONDARY SCHOOL MAYFLOWER SECONDARY SCHOOL MAYFLOWER SECONDARY SCHOOL
FLOWER SECONDARY SCHOOL MAYFLOWER SECONDARY SCHOOL MAYFLOWER SECONDARY SCHOOL MAYFLOWER SECONDARY SCHOOL
Sci (Chem)
( ) 5076 /5078
[65
65 marks]
SEMESTRAL A SSMENT One
ASSESSMENT
SSESS
May 2018
ANSWER SCHE
SCHEME
EME
58
2
Paper 1 (20m)
21 22 23 24 25 26 27 28 29 30
B B D D C A B A D A
31 32 33 34 35 36 37 38 39 40
A C D C A C A D C D
Paper 2
Section A ((45m))
2 (a) (i) S 1
(ii) Q 1
(iii) U 1
(iv) V and S 1
(b) It is fluorine. 1
Both have 9 protons, however,
Fluorine has 10 neutrons while
whi
hile
l T has
hass 11 neut
utro
rons
ns..
neutrons. 1
3 (a) (i) ca
carbon
arb
r on monoxide 1
(ii) Fe 2 O 3 + 3CO Æ 2F 2Fe
Fe + 3CO3CO 2 1
Fe 2 O 3 loses oxygen n tto
o carbon
ca
arb
rbon
on monoxide,
mon
o oxxide, 1
And is thus reduced d to fform
orm iron /
or 1
The oxid id
dat
atio
oxidationion state
stat
atee of FFee de
d creases from +3 in haematite
decreases
in iron.
to 0 in ir
(iii) Limestone
Limeston ne and
a d
an 1
coke.
co
okee. 1
4 (a) 1
Universal indicator.
1
There will be a colour change from purple to green.
[Turn over
www.KiasuExamPaper.com
59
3
1 mark for
no inn
inner
shel
shell
electro
electrons
for both
bo
H N H
(ii) Si
Sinc
Since
nce e lilithium hydride consist of strong electrostatic forces of 1
attraction between positive and negative ions while ammonia
consists of weak intermolecular forces between ammonia 1
molecules.
l l
And because much more energy is required to overcome the 1
forces of attraction in lithium hydride compared to ammonia,
Therefore, lithium hydride has a much higher melting and boiling
point, hence it exist as a solid while ammonia exist as a gas
under room temperature.(ERC)
[Turn over
www.KiasuExamPaper.com
60
4
6 (a) A: iron 1
B: iron(II) nitrate 1
C: hydrogen gas 1
D: iron(II) hydroxide 1
E: iron(III) hydroxide 1
F: iron(III) chloride 1
(b) Test the gas evolved using a burning / lighted splint. It should extinguish 1
with a pop sound
Section
ction B (20m)
8 (a) (i) Sulfur gains oxygen to form sulfur dioxide / the oxidation state of 1
sulfur increases from 0 to +2.
+2
(ii) Mole of sulfur dioxide = 320 / 24 = 13.33 mol 1
Mole ratio of SO 2 : S = 1:1 = 13.33:13.33 1
Mass of sulfur burnt = 13.33 * 32 = 426.6 = 427g 1
(iii) sulfur dioxide can react with the water to form sulfurous acid. 1
Sulfurous acid oxidises in the air to sulfuric acid which forms acid 1
rain which can damage buildings made of limestone.
[Turn over
www.KiasuExamPaper.com
61
5
9 (a) The smaller the particle size, the larger the sur surface
urrffa
urfa ac
ceeaarea
rea ffo
re for
or re
rreaction
eac
actition
tion tto
o 1
occur.
This increases the frequency of collisions between rea actant particles,
reactant parrticles,
pa s, 1
resulting in a faster reaction.
(b) Add a fixed mass of magnesium strip to hydrochloric
hydrochl h oric acid off 1
fixed concentration.
Collect the volume of hydrogen n gas s collected
coollected using
usin
us ing
g a gas
g s syringe and
ga 1
measure the volume of hydrogen en gas
gas collected
colleect
cted
ed atat regular
regu
regular time intervals 1
seco
c nds).
(eg. 30 seconds).
Record the values collected and annd plot
p ott a graph
pl gra
raph
ph ofo volume of hydrogen gas 1
collected against time.
Repeat the experiment using g magnesium
ma agn
gnesiu m powder instead of magnesium
ium 1
Comparre the
ribbon. Compare the sl
s op
pes of
slopes of the
the graph obtained for both ribbon and
powder to investigate
in
nve
vest
s ig
igate the
th
he rate
ra
ate of
of reaction.
[Turn over
www.KiasuExamPaper.com
62
www.KiasuExamPaper.com
63
PASIR RIS CREST SECONDARY SCHOOL
Mid-Year Examination
Secondary Four Express and Five Normal Academic
CANDIDATE
NAME
INDEX
CLASS /
NUMBER
Write your name, class and register number in the spaces provided.
Write in soft pencil.
Do not use staples, paper clips, glue or correction fluid.
Each correct answer will score one mark. A mark will not be deducted for a wrong answer.
Any rough working should be done in this booklet.
A copy of the Data Sheet is printed on page 8.
A copy of the Periodic Table is printed on page 9.
The use of an approved scientific calculator is expected, where appropriate.
40
Parent’s Signature
www.KiasuExamPaper.com
1
64
Multiple Choice Questions (40 marks)
Answer all questions.
1 A student mixes 25 cm3 samples of acid solution with different volumes of alkali solution.
At every 30 seconds, the student measures the change in temperature.
Which piece of apparatus is not needed?
A gas syringe
B measuring cylinder
C thermometer
D stop watch
3 The table shows the melting and boiling points of four substances.
Which of the following substances contains particles that are sliding past each other at room
temperature (25 ºC)?
A – 110 – 55
B – 20 15
C 0 100
D 744 1214
www.KiasuExamPaper.com
2
65
4 Aqueous sodium hydroxide is added to aqueous salt Z and a white precipitate formed. The white
precipitate dissolved when excess sodium hydroxide is added.
When this reaction was completed, aluminium foil is added to the solution. The gas given off turned
damp red litmus blue.
What is aqueous salt Z?
A calcium nitrate
B lead(II) sulfate
C zinc nitrate
D zinc sulfate
21
19
9
F– 11
Na+
Which of the following statements is correct?
A Both the ions contain the same number of electrons.
B Both the ions contain the same number of protons.
C The fluoride ion contains more electrons than the sodium ion.
D The sodium ion contains more neutrons than the fluoride ion.
www.KiasuExamPaper.com
3
66
8 50 cm3 of nitrogen gas reacts with 50 cm3 of oxygen gas to produce nitrogen dioxide. The chemical
equation for the reaction is given below:
N 2 (g) + 2 O 2 (g) Æ 2 NO 2 (g)
What are the volumes of the gases remaining at room temperature and pressure?
A 0 0 100
B 0 25 50
C 25 0 50
D 25 25 50
9 20 g of magnesium oxide, MgO, reacts completely with 500 cm3 of dilute nitric acid.
The chemical equation of the reaction is as follows:
MgO (s) + 2 HNO 3 (aq) Æ Mg(NO 3 ) 2 (aq) + H 2 O (l)
What is the concentration of the acid used?
[relative atomic masses, A r : O, 16; Mg, 24]
A 0.002 mol/dm3
B 0.008 mol/dm3
C 2 mol/dm3
D 8 mol/dm3
10 Which substance below will not react with aqueous potassium hydroxide but will react with dilute
hydrochloric acid to form a salt and water?
A aluminium oxide
B carbon monoxide
C copper(II) oxide
D nitrogen dioxide
11 Which pair of reagents can be best used to prepare insoluble magnesium carbonate?
reagent 1 reagent 2
www.KiasuExamPaper.com
4
67
12 Which of the following reactions will have the slowest rate of reaction?
30 ºC 30 ºC
20 ºC 20 ºC
www.KiasuExamPaper.com
5
68
16 The pie-chart shows the composition of pure air.
H
F G H
A 1 only
B 1 and 2 only
C 1, 3 and 4 only
D 1, 2, 3 and 4
www.KiasuExamPaper.com
6
69
19 A metal X reacts as follows:
By comparing X with calcium and silver, which of the following shows the correct order of reactivity of
the metals, starting with the least reactive?
A calcium, silver, X
B calcium, X, silver
C silver, X, calcium
D X, calcium, silver
20 An experiment was set up as shown below to investigate the rate of rusting under different conditions.
cork cork
1 2 3 4
Which of the following predicts the order of the test-tubes in which rust would first appear?
A 1, 2, 3, 4
B 1, 3, 2, 4
C 4, 2, 3, 1
D 4, 3, 2, 1
End of Paper
www.KiasuExamPaper.com
7
70
Data Sheet
www.KiasuExamPaper.com
8
71
72
www.KiasuExamPaper.com
9
PASIR RIS CREST SECONDARY SCHOOL
Mid-Year Examination
Secondary Four Express and Five Normal Academic
CANDIDATE
NAME
INDEX
CLASS /
NUMBER
Write your name, class and register number in the spaces above.
Write in dark blue or black pen.
You may use an HB pencil for any diagrams, graphs, tables or rough working.
Do not use staples, paper clips, highlighters, glue or correction fluid.
Section A
Answer all questions.
Write your answers in the spaces provided on the question paper.
Section B
Answer any two questions.
Write your answers in the spaces provided on the question paper.
The number of marks is given in the brackets [ ] at the end of each question or part question.
65
Parent’s Signature
www.KiasuExamPaper.com
1
73
Section A [45 marks]
Answer all the questions in the spaces provided.
[3]
[Total: 3 marks]
2 The diagrams N, P, Q, R, S and T in Fig 2.1 represent the particles in different substances.
N P Q
R S T
Fig 2.1
................…………………………… [1]
(b) Which of the following above best represents a mixture containing fluorine and chlorine
gases?
................…………………………… [1]
(c) Which of the following above best represents air?
................…………………………… [1]
(d) Which of the following above best represents neon gas?
................…………………………… [1]
[Total: 4 marks]
www.KiasuExamPaper.com
2
74
3 The atomic structures of atoms W, X, Y and Z are shown below. The elements are found in
Period 3 of the Periodic Table. The letters do not represent the elements and only the valence
electrons of the elements are shown.
W X Y Z
(a) State and explain which group does atom Z belongs to in the Periodic Table.
................……………………………………………………………………………………….
................……………………………………………………………………………………….
[2]
(b) (i) Write the chemical formula of the compound formed between atoms W and X.
.......……………………………………………………………………………………….
[1]
(ii) The compound formed between W and X has a melting point of 1100 ºC. In terms of
structure and bonding of the compound formed, explain why it has a high melting point.
.......……………………………………………………………………………………….
.......……………………………………………………………………………………….
.......……………………………………………………………………………………….
[2]
(c) Explain why atom Y is the least chemically reactive as compared to the other atoms.
................……………………………………………………………………………………….
................……………………………………………………………………………………….
................……………………………………………………………………………………….
[1]
[Total: 6 marks]
www.KiasuExamPaper.com
3
75
4 The Blast furnace reaction is an industrial process used to obtain iron from its ore. The iron
obtained is usually used to produce stainless steel, an alloy, which is harder and stronger than
pure iron. Stainless steel is an important material in construction building.
(a) (i) Define the term, alloy.
……………………………………………………………………………………………………..
[1]
(ii) Apart from its hardness and strength, state another advantage of using stainless steel
as an industrial material.
……………………………………………………………………………………………………..
[1]
(b) Carbon, also known as coke, is added to the Blast furnace reaction for the extraction of iron.
The chemical equation for this reaction is given below.
2 Fe 2 O 3 (s) + 3 C (s) Æ 4 Fe (l) + 3 CO 2 (g)
Given 30% of iron(III) oxide, Fe 2 O 3 , is present in 1000 kg of haematite used, calculate the
mass of carbon required for the extraction of iron.
[relative atomic masses, A r : C, 12; O, 16; Fe, 56]
www.KiasuExamPaper.com
4
76
(c) Silicon dioxide, SiO 2 , is an impurity produced in Blast furnace.
Explain how silicon dioxide is removed from the Blast furnace.
……………………………………………………………………………………………………………
……………………………………………………………………………………………………………
……………………………………………………………………………………………………………
[2]
(d) During the production of iron, sulfur dioxide gas is produced. Explain why sulfur dioxide gas
produced pose an environmental threat to water bodies.
……………………………………………………………………………………………………………
……………………………………………………………………………………………………………
……………………………………………………………………………………………………………
[2]
[Total: 9 marks]
www.KiasuExamPaper.com
5
77
5 Chlorine gas, a member of the halogens, is an element in Group VII of the Periodic Table.
(a) State two physical properties of chlorine, other than existing as a gas at room temperature
and pressure.
……………………………………………………………………………………………………………
……………………………………………………………………………………………………………
[2]
(b) Explain, using its electronic structure, why chlorine is found in Period 3 of the Periodic Table.
……………………………………………………………………………………………………………
……………………………………………………………………………………………………………
[2]
(c) Chlorine gas reacts vigorously with hot zinc metal to produce solid zinc chloride.
Construct a balanced chemical equation, including state symbols, for the reaction.
……………………………………………………………………………………………………………
[2]
(d) When chlorine gas is bubbled into aqueous potassium bromide, potassium chloride and
bromine solution is obtained. Explain why this reaction occurs.
……………………………………………………………………………………………………………
……………………………………………………………………………………………………………
……………………………………………………………………………………………………………
[2]
[Total: 8 marks]
www.KiasuExamPaper.com
6
78
6 (a) Metals A, B and C are placed in salt solutions as shown in the table.
A no reaction C displaced
B A displaced C displaced
C no reaction no reaction
Arrange the reactivity of the metals, starting with the least reactive metal.
……........................……………………………………………………………………………
[1]
(b) Explain why carbon can be used to obtain zinc from zinc oxide but not to obtain sodium from
sodium oxide.
……........................……………………………………………………………………………
……........................……………………………………………………………………………
……........................……………………………………………………………………………
[2]
(c) Sodium metal is kept in oil to prevent it from corrosion.
Explain how the oil prevents the sodium metal from corrosion, stating clearly the conditions
that cause the corrosion of sodium.
.................................…………………………………………………………………………..
……........................……………………………………………………………………………
…….........................……………………………………………………………………………
[3]
[Total: 6 marks]
www.KiasuExamPaper.com
7
79
7 The figure below describes the reactions between colourless solution A and grey solid B.
A ………………………………………………….
B ………………………………………………….
C ………………………………………………….
D ………………………………………………….
E ………………………………………………….
F ………………………………………………….
[6]
(b) Construct a balanced ionic equation for the formation of precipitate E. State symbols are not
required.
……………………………………………………………………………………………………………
[2]
(c) Explain why grey solid B cannot be a metal carbonate.
……………………………………………………………………………………………………………
……………………………………………………………………………………………………………
[1]
[Total: 9 marks]
www.KiasuExamPaper.com
8
80
Section B [20 marks]
8 Nitrogen, oxygen and argon gases can be extracted from compressed liquefied air (mixture of
miscible liquids) at –200 ºC.
(a) (i) State the separation method used to obtain the gases separately at –200 ºC.
..…………………………………………………………………………………………...
[1]
(ii) Describe the changes in movement of the air particles as it is compressed and cooled
from room temperature to –200 ºC.
..…………………………………………………………………………………………...
..…………………………………………………………………………………………...
[1]
(b) Oxygen is a reactive non-metal.
Describe, in terms of the number of electrons gained, lost or shared, what happens when
(i) an oxygen atom combines with magnesium atom(s).
..…………………………………………………………………………………………..
..…………………………………………………………………………………………..
[2]
(ii) an oxygen atom combines with fluorine atom(s).
..…………………………………………………………………………………………...
..…………………………………………………………………………………………...
[2]
www.KiasuExamPaper.com
9
81
(c) (i) Draw a ‘dot-and-cross’ diagram for nitrogen gas.
[2]
(ii) Draw a ‘dot-and-cross’ diagram for oxygen gas.
[2]
[Total: 10 marks]
www.KiasuExamPaper.com
10
82
9 (a) State two physical properties of copper metal.
Property 1: ………………………………………………………………………………………
Property 2: ………………………………………………………………………………………
[2]
(b) Describe a way to prepare a pure sample of copper(II) sulfate crystals, from copper metal.
Use the following information to help you
x copper does not react with dilute acids
x copper burns in oxygen to form a black solid, which is copper(II) oxide
x copper(II) oxide is insoluble in water
x copper(II) sulfate is soluble in water
……………………………………………………………………………………………………………
……………………………………………………………………………………………………………
……………………………………………………………………………………………………………
……………………………………………………………………………………………………………
……………………………………………………………………………………………………………
……………………………………………………………………………………………………………
[4]
(c) 10 g of copper(II) carbonate lumps were reacted with excess 1.0 mol/dm3 hydrochloric acid
and the carbon dioxide gas produced was collected. The experiment was repeated again but
using excess 2.0 mol/dm3 hydrochloric acid. The graph of the data collected is plotted and
shown below.
volume of
carbon dioxide experiment 1
/ cm3
experiment 2
time / min
experiment 1: 10 g of copper(II) carbonate lumps with excess 2.0 mol/dm3 hydrochloric acid
experiment 2: 10 g of copper(II) carbonate lumps with excess 1.0 mol/dm3 hydrochloric acid
(i) State why the production of carbon dioxide gas stopped after a period of time.
.…………………………………………………………………………………………………….
[1]
www.KiasuExamPaper.com
11
83
(ii) Use your knowledge of reacting particles to explain why a higher concentration of acid
results in a faster rate of reaction.
.…………………………………………………………………………………………………….
.…………………………………………………………………………………………………….
.…………………………………………………………………………………………………….
[2]
(iii) The experiment is repeated using 5 g of powdered copper(II) carbonate and excess
2.0 mol/dm3 hydrochloric acid. Add to Fig. 9.1 the graph you would expect. The original
graphs are already included. Label the new graph as 3.
volume of
carbon dioxide experiment 1
/ cm3
experiment 2
time / min
[1]
Fig. 9.1
[Total: 10 marks]
www.KiasuExamPaper.com
12
84
10 (a) The diagram below shows the atomic radius of the first 20 elements in the Periodic Table.
Atomic Radius
Element Symbol
(i) Use the diagram above to describe the change in atomic radius across the Period and
down the Group.
……………………………………………………………………………………………………..
……………………………………………………………………………………………………..
[2]
(ii) Describe the change in the character of the elements across Period 3 and how it
affects the respective oxides formed.
……………………………………………………………………………………………………..
……………………………………………………………………………………………………..
[2]
(b) Lithium, potassium and sodium are Group I elements.
State one physical property trend and one chemical property trend of these elements.
www.KiasuExamPaper.com
13
85
(c) Describe a laboratory investigation that can be used to justify the relative positions of iron,
magnesium and silver in the reactivity series. You may include a diagram if it helps you to
answer the question.
Diagram
……………………………………………………………………………………………………………
……………………………………………………………………………………………………………
……………………………………………………………………………………………………………
……………………………………………………………………………………………………………
……………………………………………………………………………………………………………
……………………………………………………………………………………………………………
……………………………………………………………………………………………………………
……………………………………………………………………………………………………………
……………………………………………………………………………………………………………
……………………………………………………………………………………………………………
[4]
[Total: 10 marks]
End of Paper
www.KiasuExamPaper.com
14
86
Data Sheet
www.KiasuExamPaper.com
15
87
88
16
www.KiasuExamPaper.com
89
www.KiasuExamPaper.com
Secondary 4 Express and 5 Normal Academic Science(Chemistry)
Mid-Year Examination
Mark Scheme
Ma
Marks
Qn no. Answer Scheme
Alloc
Allocated
1 A 6 B 11 D 16 B [1] each
ea
2 D 7 D 12 D 17 A 20 m max
3 C 8 C 13 B 18
8 D
4 C 9 C 14 A 19 C
5 A 10 C 15 A 20 D
1 purpose e of
name of substance
su
sub
bs
sta
tance
ncce
n
oxide
calcium oxid
id calcium
de / c
ca
a
alc
lciu
lc hydroxide
ium hy
h
hydr
ydr
d oxid
de /
reducing the acidity in soil
lime / slaked lime / calcium
calcium
m carbonate
c rbon
ca onat
on ate
at
testing for presence of
limewater / calcium hydroxide
hy
ydr
droxide
carbon dioxide gas
testing for presence of ac
acidified silver
silv
si lver
er nitrate
nitrate / acidified
chloride ions in water le
ead
ad(I
(II) nitrate
lead(II) nit
itra
rate
te / acidified
acid
ac id
dified silver sulfate
[3]
1m each
Reject: chemical formula of
of substances
su
ubs
bsta
tanc
nces
es Total: 3
Total
2(a) S [1]
2(b) N [1]
2(c) Q [1]
2(d) P [1]
Total: 4
3(b)(ii) The compound formed has a giant lattice structure [1]. Thus, large
amount of energy is needed to overcome strong electrostatic forces of
attraction between the oppositely charged ions [1]. [2]
www.KiasuExamPaper.com
90
3(c) It has eight valence electrons / a completely filled valence shell /
does not need to take in, give out or share electrons with other
elements. [1]
Total: 6
4(a)(i) An alloy is a mixture containing at least one metal with other elements
/ substances. [1]
Mole ratio: 2 Fe 2 O 3 : 3 C
1875 : 2812.5
Note:
1. Allow ECF for wrong answer.
2. –1 if no/wrong units written for final answer.
Total: 9
5(b)) Chlorine has an electronic structure of 2.8.7 [1], hence it contains 3 [2]
electrons shells [1] filled with electrons. Therefore, it is in period 3.
5(d) Chlorine is more reactive than bromine [1]. Hence, it can displace
bromine [1] to form potassium chloride and bromine. [2]
Total: 8
www.KiasuExamPaper.com
91
6(a)
a) C, A, B (only answer) [1]
b)
6(b) Carbon is more reactive than zinc [1], but less reactive than an sodium
sod
odium
[1]. Hence it displaces zinc from zinc oxide but not sodium
um ffrom
rom
rom sodium
so
oxide. [2]
6(c)
c) By keeping sodium in oil, the oil creates a physical
al barrier
baarrrriie
ie [[1]
err [11] that
tha
at
at
prevents the surface of sodium metal to come in n contact
co
on
ntta
act
ct with
witith ox
o
oxygen
ygen gas
gas
[1] and water / water vapour [1], which cause
causes
es corrosion.
corr
corros
osio
ion. [3]
Total: 6
7(a)
a) A – nitric acid or HNO 3
B – iron metal or Fe
C – hydrogen e gas or H 2
D – iron(II) nitrate or Fe(NO 3 ) 2
E – iron(II) hydroxide or Fe(OH)
Fe(OH
OH)) 2
F – ammonia or NH 3
1m each; acaccept
cce
c ptt ch
chemic
chemical
ical
al formula
for
ormumula
l [6]
b)
7(b) Fe2+ + 2 OH– Æ Fe(OH)
Fe(O
Fe (OH)
(OH2
H)
1m – correct
corr
rrec
ectt equation;
equa
equati
tion
n; 1m – balanced equation [2]
c)
7(c) A metal carbonate
car
arbo
bona
nate
te will produce carbon dioxide gas, instead of hydrogen
gas. OR A m metal
etal carbonate does not produce hydrogen gas when [1]
reacted with acid.
Total: 9
www.KiasuExamPaper.com
92
8(a)(i) Fractional distillation [1]
8(b)(i) Each oxygen atom gains two electrons from magnesium [1], forming
oxide ion. Each magnesium atom loses/transfers two electrons [1] to
oxygen, forming magnesium ion. [2]
)( )
8(b)(ii) yg atom shares two valence electrons [[1]] with two fluorine
Each oxygen
atoms [1]. [2]
c)(i)
8(c)(i)
O O
[2]
c)(ii)
8(c)(ii)
N N
[2]
Note: 1m deducted
ded
educ
cte
ted
d if students only shows valence electrons
Total: 10
1
www.KiasuExamPaper.com
93
9(a) High density / High melting and boiling points / conducts electricity /
conducts heat / malleable / ductile / shiny surface / Solid at room
temperature / Pink/brown solid [2]
[Any two]
c)(i)
9(c)(i) Copper(II) carbonate is used up. [1]
c)(ii)
9(c)(ii) At a higher concentration, there are more reactant particles
particle es pe
perr un
unit
it
it
volume [1]. Hence, the frequency of effective collision
collisions
ns between
betw
be tween
tw
particles increases [1], leading to a faster rate of reaction.
reacttio
on.
n [2]
Total: 10
1
10(a)(i)
a)(i) Atomic radius increases down the group [1]and de
d
decreases
creases across the
period [1]. [2]
10(b)
0(b) Physical prop
per
erty
ty trend:
property trend
nd:: melting
melt
meltin
ing
g or boiling point decreases / density
increases [1]
Chemicall property
p op
pr oper
e ty
y trend:
tre
rend
n : chemical reactivity increases [1] [2]
www.KiasuExamPaper.com
94
10(c)
0(c) Reaction condition [1]: state the use of either water / steam / dilute
dilu
lutte acids
lu aci
ci
cids
OR
Total: 10
1
www.KiasuExamPaper.com
95
www.KiasuExamPaper.com
96
1
SECONDARY 4 EXPRESS
MID-YEAR EXAMINATION 2018
Write your name, class and register number on all the work you hand in.
Write in dark blue or black pen.
You may use a soft pencil for any diagrams, graphs, tables or rough working.
Do not use staples, paper clips, glue or correction fluid.
Section A
There are ten questions in this section. Answer all questions.
For each question there are four possible answers, A, B, C and D.
Choose the one you consider correct and record your choice in the boxes provided on page 4.
Section B & C
Answer all questions in the spaces provided.
The number of marks is given in brackets [ ] at the end of each question or part question.
A copy of the Periodic Table is printed on page 8.
Section Marks
A /10
B /20
C /10
Total /40
www.KiasuExamPaper.com
97
2
Which two methods of separation should be used in sequence to obtain samples of liquid J first
before solid potassium chloride?
method 1 method 2
B evaporation sublimation
C distillation filtration
D filtration evaporation
A B
C D
1 2 3 4
A ammonia sodium chloride methane water
www.KiasuExamPaper.com
98
3
metal heat
carbonate metal oxide X + carbon dioxide
W
dilute nitric
acid
25 The diagrams show a series of experiments carried out using chlorine water and bromine water.
A combustion B freezing
C photosynthesis D respiration
www.KiasuExamPaper.com
99
4
Which of the following shows the correct change in oxidation states of sulfur in each stage of the
process?
S SO 2 SO 3 H 2 SO 4
A 0 +2 +6 +8
B 0 +4 +6 +6
C +2 0 +6 +6
D +6 +6 +2 0
A 1 and 2 only
B 2 and 3 only
C 1 and 3 only
D 1, 2 and 3
Which solid, when placed between P and Q, would cause the light bulb L to light up?
A copper
solid between
B hydrogen fluoride electrical contacts
C sodium chloride
D sulphur
www.KiasuExamPaper.com
100
5
30 In the preparation of salts, which of the following would require the use of a burette and pipette?
gas X
potassium
hydroxide pellets
as drying agent
heat
32 Which ionic equation represents the neutralisation of dilute sulfuric acid with aqueous sodium
hydroxide?
A H+ + OH– o H 2 O
B NaOH + H+ o Na+ + H 2 O
C H 2 SO 4 + 2OH– o SO 4 2– + 2H 2 O
D SO 4 2– + 2Na+ o Na 2 SO 4
www.KiasuExamPaper.com
101
6
temperature/ °C
time/ min
Which of the following options correctly describes the changes that occur between points C to D?
attractive forces
separation of particles energy of particles
between particles
A decreases increases decreases
A Group I
B Group II
C Group V
D Group VI
www.KiasuExamPaper.com
102
7
36 Some zinc carbonate was reacted with excess dilute nitric acid.
The graph shows the volume of carbon dioxide gas evolved at 20 second intervals until the
reaction has finished. Graph 1 shows the results obtained from this reaction.
graph 1
graph 2
What volume of 1.0 mol/dm3 hydrochloric acid is needed to react completely with 1.0 g of calcium
carbonate (M r = 100)?
A 10 cm3 B 20 cm3
38 Which of the following substances is not present in the reaction during the extraction of iron?
www.KiasuExamPaper.com
103
8
B K L J
C K J L
D L J K
End of Paper
www.KiasuExamPaper.com
104
9
105
www.KiasuExamPaper.com
Pasir Ris Secondary School
Name Class Register Number
Write your name, class and register number on all the work you hand in.
You may use a soft pencil for any diagrams, graphs, tables or rough working.
Write in dark blue or black pen.
Do not use staples, paper clips, glue or correction fluid.
Section A
Answer all questions.
Write your answers in the spaces provided on the question paper.
For Examiner’s Use
Section B Section A
Answer any two questions.
Write your answers in the spaces provided on the question paper.
At the end of the examination, fasten all your work securely together.
The number of marks is given in brackets [ ] at the end of each question or part question.
www.KiasuExamPaper.com
106
PRSS_2018_MYE_S4E5NA_SCI_5076_P3
Section A
1 (a) Use three words from the box below to describe each substance in Table 1.1. For
Examiner’s
The words can be used once, more than once, or not at all. Use
Table 1.1
1 ……………………………
A 2 ……………………………
3 .………….………………..
1 ……………………………
B 2 ……………………………
3 .………….………………..
1 ……………………………
C 2 ……………………………
3 .………….………………..
[3]
(b) (i) Explain why substance A will conduct electricity when dissolved in water.
……………………………………………………………………………………….……... [1]
…………………………………………………………………………………….………... [1]
2
www.KiasuExamPaper.com
107
PRSS_2018_MYE_S4E5NA_SCI_5076_P3
2 Spots of different coloured dyes were placed along a pencil line on a sheet of chromatography For
Examiner’s
pencil line
Fig. 2.1
(a) Which physical property allows chromatography to separate components of the dyes?
………………………………………………………………………………………….….……..…. [1]
(b) Based on Fig. 2.1, what can be deduced about the components of the black dye?
……………………………………………………………………………………….….…..………. [1]
(c) Suggest why the start line was drawn in pencil line and not in ink for this experiment?
……………………………………………………………………………………….…...………….
………………………………………………………………………………………..……..………. [1]
3
www.KiasuExamPaper.com
108
PRSS_2018_MYE_S4E5NA_SCI_5076_P3
3 Hydrogen bromide has a melting point of –87 oC and a boiling point of –67 oC. For
Examiner’s
Use
(a) Draw a ‘dot and cross’ diagram to show the arrangement of electrons in a molecule of
hydrogen bromide. Show only the outer shell electrons.
[2]
(b) Hydrogen bromide dissolves in water to form an acidic solution which is colourless.
(i) Give the formula of the ion which causes the acidity.
…………………………………………………………………..……………….….……… [1]
(ii) Describe what is seen when chlorine gas is bubbled through the solution.
…………………………………………………..…………………….…………….………
……………………………………………………………..………….………….………… [1]
(iii) Construct an ionic equation, including state symbols, for the reaction you have
described in (ii).
………………………………………………..…………………..………………………… [2]
4
www.KiasuExamPaper.com
109
PRSS_2018_MYE_S4E5NA_SCI_5076_P3
4 Zinc blende is an ore that contains mainly zinc sulfide (ZnS). The extraction of zinc from its ore For
Examiner’s
happens in the blast furnace. Use
The ore of zinc blende is roasted in air (oxygen) to form zinc oxide which is then reduced with
carbon monoxide in the blast furnace, similar to the extraction of iron from haematite.
ZnO + CO o Zn + CO 2
(a) State which substance is reduced and give a reason for your answer.
(b) Zinc produced by the blast furnace is often alloyed to increase its hardness and strength.
Brass is an alloy of zinc and copper.
(i) Draw the structure of brass in the box provided in Fig. 4.1.
(ii) With reference to your drawing in Fig. 4.1, explain why brass is harder and stronger
compared to pure zinc.
………………………………………………………………………………………...………
...………………………………………………………………………………………………
…...…………………………………………………………………………………………… [2]
5
www.KiasuExamPaper.com
110
PRSS_2018_MYE_S4E5NA_SCI_5076_P3
5 The reaction between copper(II) oxide and hydrogen can be represented by the equation as For
Examiner’s
shown. Use
(a) (i) Calculate the number of moles of copper(II) oxide used in the reaction.
[1]
(ii) Hence, determine the number of moles of hydrogen gas is required for all the
copper(II) oxide to be used up in the reaction.
[1]
(b) It is also known that 165 cm3 of hydrogen gas was used in the reaction.
(i) Using your answer from (a), determine the limiting reagent.
Explain your answer clearly by showing all relevant calculations.
[3]
(ii) Hence or otherwise, calculate the mass of water vapour produced at the end of the
reaction.
[2]
6
www.KiasuExamPaper.com
111
PRSS_2018_MYE_S4E5NA_SCI_5076_P3
V X
W Z
Fig. 6.1
For each of the following statements, decide whether the statement is true or false and state a
reason for your decision.
.………………………………………………………………………………………………….…..
………………………………………………………………………...……………………………. [1]
.………………………………………………………………………………………………….…..
………………………………………………………………………………………………………. [1]
.………………………………………………………………………………………………….…..
………………………………………...……………………………………………………………. [1]
.………………………………………………………………………..……………………..….…..
…………………………………………………………………………..………………….………. [1]
7
www.KiasuExamPaper.com
112
PRSS_2018_MYE_S4E5NA_SCI_5076_P3
7 Study the
Study the flowchart
flowchart below
in Fig.and
7.1 answer
and answer the following
the questions thatquestions.
follow. For
Examiner’s
Use
colourless gas D
that extinguishes colourless
+ solution E
lighted splint with a
‘pop’ sound
add aqueous
silver nitrate
white precipitate F
Fig. 7.1
A …………………………………………...
B …………………………………………...
C …………………………………………...
D …………………………………………...
E …………………………………………...
F …………………………………………... [6]
(b) Write a balanced chemical equation for any one of the reactions described in Fig. 7.1.
………………………………………………………………………………………...……….……. [2]
8
www.KiasuExamPaper.com
113
PRSS_2018_MYE_S4E5NA_SCI_5076_P3
8 (a) A chemical company makes salts for use in industries. Table 8.1 shows some names and For
Examiner’s
formulae of salts with the names of the acids and other compounds used to make them. Use
Table 8.1
sodium sulfate Na 2 SO 4
(b) Fig. 8.2 shows a rusted car. However, not all the parts have rusted. The areas that have
not rusted are either painted or have plastic coatings.
painted
rusted areas
plastic coating
Fig. 8.2
Explain how the paint and plastic coating can slow down rusting.
..............................................................................................................................................
………………………………………………………………………………………………………. [2]
(c) Harmful gases released into the atmosphere can form acid rain which speeds up rusting.
Name one such gas which causes acid rain and state its source.
………………………………………………………………………………………………………. [2]
9
www.KiasuExamPaper.com
114
PRSS_2018_MYE_S4E5NA_SCI_5076_P3
Section B For
Examiner’s
Use
Chlorine ranks among the top ten chemicals produced today. Chlorine is produced by
passing an electric current through a concentrated solution of sodium chloride or
through molten sodium chloride. This process is one of the most important commercial
processes in industry. Chlorine, in one form or another, is added to most swimming
pools, spas, and public water supplies because it kills bacteria that cause disease. Many
people also use chlorine to bleach their clothes. Large paper and pulp mills use chlorine
to bleach their products.
Chlorine is very reactive. The reaction between chlorine and other elements can often
be vigorous. For example, chlorine reacts explosively with hydrogen to form hydrogen
chloride.
………………………………………………………………………………………………………. [1]
(b) The chemical symbols of the two chlorine isotopes are shown below.
………………………………………………………………………………………………….……
……………………………………………………………………………………………………….
………………………………………………………………………………….……………..……. [2]
10
www.KiasuExamPaper.com
115
PRSS_2018_MYE_S4E5NA_SCI_5076_P3
(i) Complete Table 9.1 which gives information about the two ions in magnesium
chloride.
Table 9.1
chloride ion 17 18
[2]
(ii) Draw a ‘dot’ and cross diagram to show the arrangement of electrons in magnesium
chloride. Show only outer shell electrons.
[2]
(d) Chlorine can react with hydrogen to form hydrogen chloride. Hydrogen chloride is a gas at
room temperature.
……………………………………………………………………………………….……..
……………………………………………………………………………………….…….. [1]
……………………………………………………………………………………………....
…………………………………………………………………………………………..…..
…………………………………………………………………………………………..…..
…….………………………………………………………………………………….……..
……………………………………………………………………………………….……... [2]
11
www.KiasuExamPaper.com
116
PRSS_2018_MYE_S4E5NA_SCI_5076_P3
10 (a) Hydrochloric acid is used for rust removal while sodium hydroxide is used in detergents. For
Examiner’s
Use
(i) State the colour of Universal Indicator in dilute hydrochloric acid and in aqueous
sodium hydroxide.
……………………………………………………………………………………………….
………………………………………………………………………………………………. [2]
(ii) Explain briefly, in terms of ions in solution, the reason for the difference in acidity
and alkalinity of hydrochloric acid and sodium hydroxide solutions.
……………………………………………………………………………………………….
………………………………………………………………………………………………. [2]
(iii) The reaction between hydrochloric acid and magnesium metal produces a soluble
salt, magnesium chloride.
Describe the steps to obtain a pure sample of magnesium chloride from the reaction.
…………………………………………………………………………….…………………
……………………………………………………………………………………………….
……………………………………………………………………………………………….
……………………………………………………………………………………………….
……………………………………………………………………………………………….
………………………………………………………………………………………………. [4]
(b) In an experiment, 20.0 cm3 of 1.50 mol/dm3 sodium hydroxide exactly neutralised 25.0 cm3
of hydrochloric acid. Using the chemical equation provided for the reaction, calculate the
concentration of the hydrochloric acid used.
[2]
12
www.KiasuExamPaper.com
117
PRSS_2018_MYE_S4E5NA_SCI_5076_P3
11 (a) Fig. 11.1 shows the speed of reaction between calcium carbonate and hydrochloric acid For
Examiner’s
in two different experiments. Us
time / s
Fig. 11.1
(i) Based on the graphs, compare the speed of reaction for the two experiments.
……………………………………………………………………………………………….
………………………………………………………………………………………………. [1]
(ii) Use your knowledge of reacting particles to explain why the particle size of calcium
carbonate affects the speed of reaction.
……………………………………………………………………………………………….
……………………………………………………………………………………………….
……………………………………………………………………………………………….
………………………………………………………………………………………………. [2]
(iii) Write a balanced chemical equation, including state symbols, for the reaction
between calcium carbonate and hydrochloric acid.
.……………………………………………………………………………………………… [2]
13
www.KiasuExamPaper.com
118
PRSS_2018_MYE_S4E5NA_SCI_5076_P3
(b) Sketch on Fig. 11.1 the speed of reaction for 5 g of powdered calcium carbonate. [1]
Label this ‘Experiment 3’.
(c) The temperature of the mixtures increased during the reaction in both experiments 1 and
2.
………………………………………………………………………………………………. [1]
(ii) Explain in terms of bond breaking and bond forming for your answer in c (i).
……………………………………………………………………………………………….
……………………………………………………………………………………………….
……………………………………………………………………………………………….
……………………………………………………………………………………………….
……………………………………………………………………………………………….
………………………………………………………………………………………………. [2]
(iii) Suggest a method that can be used to accurately determine that all the acid has
been used up during the reaction.
.……………………………………………………………………………………………… [1]
End of Paper
14
www.KiasuExamPaper.com
119
PRSS_2018_MYE_S4E5NA_SCI_5076_P3
DATA SHEET
15
www.KiasuExamPaper.com
120
PRSS_2018_MYE_S4E5NA_SCI_5076_P3
16
121
16
www.KiasuExamPaper.com
www.KiasuExamPaper.com
122
PRSS_2018_MYE_S4E_SCI_5076_P3
9
www.KiasuExamPaper.com
123
Pasir Ris Secondary School
SECONDARY 4 EXPRESS
MID-YEAR EXAMINATION 2018
MARKING SCHEME
www.KiasuExamPaper.com
124
PRSS_2018_MYE_S4E_SCI_5076_P3
2
www.KiasuExamPaper.com
125
PRSS_2018_MYE_S4E_SCI_5076_P3
(b)
(i) H+ Students wrote equations of HBr or H- . [1]
(ii) Colourless solutions starts to turn Students described the displacement [1]
reddish - brown reaction itself rather than colour
observations. Some stated yellow instead
of reddish brown.
(iii) Cl 2 (g) + 2Br- (aq) o Very poorly done. 98% could not do this [2]
2Cl- (aq) + Br 2 (aq) question and could not balance equation.
Need to revisit this topic.
[1] – correct chemical
formula/ions
[1] – correct state symbols
(2nd mark is only awarded if the
1st mark is given)
[Total:
[Total
[Tot al:: 6 marks]
marks
4 (a) Substance reduced: ZnO has been substance reduced:
redu
re duce most
ed: mos ostt incorrectly
inco
in corr
rreectly state
reduced [1] as just Zn.
Zn n.
Reason: ZnO has lost an oxygen atom to Reason:
Reas so
onn: students
sttud
s ud
den
ents
en s are able
abl
ble explain the [2]
e to explain
form Zn / oxidation number of Zn has loss
lo s of
oss of oxygen
ox
o xxyg
yg
y g en tto
gen
en identify
o id
iden
enti
en tify
ti fy tthe
he substance
decreased from +2 in ZnO to 0 in Zn. [1] reduced.
reduceedd.. However,
Ho owwever their phrasing
er, th
er phra
ph ras
ra sing is wrong
using oxygen hass b been
een reduced
redu
re duced from zinc
oxide.
oxid
de.
(b) (i)
most
mo
m st students
stu
t dents who made mistakes drew
orde
or
o d rly arranged atoms or did not
orderly no
differentiate the size of the atoms
di
d
enough.
e
tthe size of the atoms enough. Labelling [1]
might help.
m
(ii) The
The different
diff
di ffer entt sized atoms disrupts
eren Most fail to get the full marks by either
the
th e orderly
derrly arrangement [1]
orrde omitting different size disrupts orderly
of pure
ure metal. This makes it
pur arrangement. [2]
harder for the layers to slide over
one another [1] thereby making it
harder.
[Total: 5 marks]
3
www.KiasuExamPaper.com
126
PRSS_2018_MYE_S4E_SCI_5076_P3
4
www.KiasuExamPaper.com
127
PRSS_2018_MYE_S4E_SCI_5076_P3
F as above abo
boove
ve
(b) 2AgNO 3 (aq) + ZnCl 2 (aq) o 2AgCl (s) + Most t students
s t
tuu de
d e n
ntts who
wh
w could
ho coul not do the above [2]
ld not
Zn(NO 3 ) 2 (aq) could
co
o ul
u l d not
no
n o t write
w
wrri
it
ite
t e a balanced
b
baa lanc
la nced
nced equation.
equ ation. Some
quat
BaCl 2 (aq)+ H 2 SO 4 (aq) o BaSO 4 (s) + alsoo wrote
wrro te nonsensical
ote nonse i al rresponse
enssic espo
es ponse as the
po
2HCl (aq) reaction cannotcannot go o through.
thr
hrough
hr gh
h.
2HCl (aq) + Zn (s) o ZnCl 2 (aq) + H 2 (g)
marks]
[Total: 8 marks
8 (a)
name of salt formula
formul a of name
ula off ac
me o acid used
id u sed name of the other
salt to make
mak
ake ssalt
e sa lt compound used to make
salt
(b) The paint and plastic coating acts as a Most students could identify why the paint can
barrier [1] to be used to prevent rusting but quite a large [2]
number did not state how it acts as a
protective layer/ barrier from the reactants.
5
www.KiasuExamPaper.com
128
PRSS_2018_MYE_S4E_SCI_5076_P3
(c) Nitrogen dioxide – motor vehicles Most correctly stated the gases SO2 but CO [2]
Sulfur dioxide – factories / coal/ was another incorrect response.
volcanic eruptions
[Total: 7 marks]
9 (a) Solution of sodium chloride and Many stated swimming pool, tap but the [1]
seawater water was only inferred not mentioned.
(b) Both have same number of protons, 17. Many students correctly stated the the same [1]
They have different number of number in proton but did not elaborate on the [1]
neutrons, Cl-35 has 18 neutrons while difference in the number of neutron through
Cl-17 has 20 neutrons.
calculation to show how they knew the
neutron was different.
(c) (i) Table 8.1
charges
char
arge
g s [1], electrons [1]
ge [2]
Most failed to draw the proper charges and wrongly indicated the outermost shell
for magnesium with 2 electrons.
6
www.KiasuExamPaper.com
129
PRSS_2018_MYE_S4E_SCI_5076_P3
(d) (i)
Hydrogen and chlorine share a Most wrongly stated by just stating it has [1]
pair of electrons between them. covalent bonds without describing further.
(ii) Magnesium chloride is a solid at Most students failed to state everything to get
room temperature as a large full marks. Many confused between structure [1]
amount of energy is required to and bonding. Structure describes how the
overcome the strong electrostatic
particles are packed and its movement and
forces of attraction between
oppositely charged ions. arrangement.
[1]
Hydrogen chloride is a gas at room
temperature as only a small
amount of energy is required to
overcome the weak intermolecular
forces of attraction between
molecules.
[Total
[Tot
[T otal
ot al : 10
10 marks]
marks
10 (a) (i) Universal indicator in hydrochloric Orange and b bluel e are
lu arre synonymous
syno
ony
nymo
mous
mo us for weak [2]
acid is red while it is purple in acid and alkalis
allk
ka
alliis
s
sodium hydroxide.
Reject orange/yellow for
hydrochloric acid and blue for
sodium hydroxide
(ii) There are more H+ ions than OH- Acidss have both types ty
ype
pes of ions only that there [2]
ions in acid. [1] are more off one ty
re o type
ype tthan
han the other.
There are more OH- th than
han H+ io
ions
ons The converse
e co
conv
n er
nverse is true.
in alkaline solution
ns. [1]
solutions. 1]
[Total: 10 marks]
11 (a) (i) Experiment 1 has a faster rate of Steeper gradient indicates a faster rate of [1]
reaction than experiment 2. / reaction.
Experiment 1 took a faster time to
complete than experiment 2.
(ii) Powdered calcium carbonate has Most omitted to state which particle was the
a larger surface area to volume smaller one and assumed the reader to [2]
7
www.KiasuExamPaper.com
130
PRSS_2018_MYE_S4E_SCI_5076_P3
ratio / larger total surface area know. Many used higher probability instead
exposed to collisions. [1] of frequency.
Results in higher frequency of
effective collisions [1],
[ ], thus
greater speed of reaction.
(iii) CaCO 3 (s) + 2HCl(aq) o Most could not recall reactions rea
reaction
onss between [2]
CaCl 2 (aq) + CO 2 (g) + H 2 O(l) acid and carbonates and and the e product
product
obtained.
(b) number
Sizeable numbe berr of sstudents
tudent
nts failed
s fa ed to label
aililed labe [1]
te
the correct term. erm
rm..
Students failed
faiiled
lle
ed tto oaappreciate
ppreciat
ate th
at thee half volume
compared
comp par
areed
e d tto
o fiffirst
irrs
irsst graph.
g aph.
gr
Students
Stud
ud
deennttss did
id not
did no ott follow
fo
olllo
loww the
the reaction
reac
re tion speed of
actti
ac tio o
the firstt graph.
g ra
gr apph.h.
End of Paper
8
www.KiasuExamPaper.com
131
www.KiasuExamPaper.com
132
West Spring Secondary School
MID-YEAR EXAMINATION 2018
Section B /45
Section C /20
Total /85
133
2
Section A
substance pH
A baking soda 9
B salt 7
C orange juice 4
D vinegar 3
3 Which ionic equation represents the reaction between aqueous potassium hydroxide and dilute sulfuric
acid?
4 A student proposed a few methods to safely prepare a sample of sodium chloride in the laboratory:
A 1 only
B 2 only
C 1 and 2 only
D 1 2 and 3
www.KiasuExamPaper.com
134
3
6 What is the mass of sodium hydroxide present in 500 cm3 of 1.0 mol/dm3 sodium hydroxide solution?
A 0.5 g
B 20 g
C 40 g
D 2 kg
7 In a reaction, 10 cm3 of butene (C 4 H 8 ) was burnt in 80 cm3 of oxygen. The equation for the reaction is
shown:
At the end of the reaction, what is the total volume of gas remaining?
(all volumes are measured at r.t.p.)
A 40 cm3
B 60 cm3
C 80 cm3
D 100 cm3
pollutant source
A carbon monoxide incomplete combustion of petrol in car engines
B nitrogen oxides lightning activity
C sulfur dioxide decomposition of organic matter
D unburned hydrocarbons incomplete combustion of petrol in car engines
9 The data below gives the concentration of various air pollutants, in parts per billion, in four different
cities.
In which city are limestone buildings under the greatest threat from pollution?
www.KiasuExamPaper.com
135
4
12 The reaction between iron(III) ions and iodide ions is represented by the following ionic equation:
Which of the following correctly describes the colour of solution X in the respective samples?
14 The ionic equations below represent the reactions between four metals zinc, iron, X and Y and the
aqueous ions of one of the other listed metals.
Zn + X2+ Æ Zn2+ + X
Zn + Fe2+ Æ Zn2+ + Fe
X + Fe2+ Æ no reaction
Y + Zn2+ Æ Y2+ + Zn
www.KiasuExamPaper.com
136
5
15 Which of the following explains why recycling ensures that metals will be available in the future?
16 In the experiment shown below, steam is passed over heated solid P, which reacts to give gas Q:
P
mineral wool
saturated with Q
water
heat
water
P Q
A copper hydrogen
B potassium oxygen
C silver oxygen
D zinc hydrogen
17 Aqueous sodium hydroxide and aqueous ammonia were added separately to two different aqueous
solutions each containing the same metallic ion. In both cases, a white precipitate was formed which
dissolved when excess sodium hydroxide or ammonia was added.
A Al3+
B Ca2+
C K+
D Zn2+
18 An aqueous solution of compound Z reacts with aqueous sodium hydroxide to form a green precipitate.
A piece of aluminium foil is added to the mixture and heated; a gas that turns damp red litmus paper to
blue is given off.
What is Z?
A ammonium nitrate
B copper(II) nitrate
C iron(II) chloride
D iron(II) nitrate
www.KiasuExamPaper.com
137
6
A B
Temperature / °C Temperature / °C
room room
temperature temperature
Time / min
Time / min
C D
Temperature / °C Temperature / °C
room room
temperature temperature
Answers
1 2 3 4 5 6 7 8 9 10
11 12 13 14 15 16 17 18 19 20
www.KiasuExamPaper.com
138
7
Section B
1 The physical and chemical properties of five unknown oxides are summarised in Table 1.1.
Table 1.1
unknown state at solubility pH of aqueous reacts with dilute reacts with dilute
oxide r.t.p in water solution hydrochloric acid? sodium hydroxide?
C gas soluble 7 no no
(i) non-metallic?
…………………………………………………………………………………………………………….. [2]
…………………………………………………………………………………………………………….. [1]
(iii) amphoteric?
…………………………………………………………………………………………………………….. [1]
…………………………………………………………………………………………………………….. [1]
…………………………………………………………………………………………………………….. [1]
www.KiasuExamPaper.com
139
8
2 Phosphorus is an element that does not react with water, but will react readily in air, forming an oxide.
Fig. 2.1 below shows a piece of phosphorus fastened to a copper wire and left for a few days in the set
up. The water slowly rises up the tube.
phosphorus
copper wire
trough
water
Fig. 2.1
(a) State the gas in air that phosphorus has reacted with.
…………………………………………………………………………………………………………….. [1]
(b) At which mark will the water level approximately be after a few days?
…………………………………………………………………………………………………………….. [1]
(c) State two gases that are left in the tube after a few days.
…………………………………………………………………………………………………………….. [2]
3 When a mixture of aluminium powder and zinc oxide is heated, the mixture burns vigorously with a
bright flame, and may even explode. The reaction is illustrated by the equation below.
………………………………………………………………………………………………………………….
…………………………………………………………………………………………………………….. [2]
(b) State whether zinc oxide is oxidised or reduced. Explain your answer in terms of electron transfer.
………………………………………………………………………………………………………………….
………………………………………………………………………………………………………………….
…………………………………………………………………………………………………………….. [2]
www.KiasuExamPaper.com
140
9
4 A common reaction iron undergoes is rusting. Fig. 3.1 below shows an experiment where some iron
nails have been exposed to different conditions in four test tubes A, B, C and D.
anhydrous
calcium
chloride oil
(drying iron nail
agent) boiled water sodium chloride
solution
tap water
A B C D
Fig. 3.1
(a) In which test tube(s) will the iron nail not rust? Explain your answer.
………………………………………………………………………………………………………………….
………………………………………………………………………………………………………………….
………………………………………………………………………………………………………………….
…………………………………………………………………………………………………………….. [3]
(b) In which test tube will the iron nail rust the fastest?
…………………………………………………………………………………………………………….. [1]
www.KiasuExamPaper.com
141
10
5 The electronic configurations of lithium, sodium and potassium are shown in Table 5.1.
Table 5.1
element symbol proton number electronic configuration
lithium Li 3 2,1
sodium Na 11 2,8,1
potassium K 19 2,8,8,1
(a) Explain why these three elements are in the same group of the Periodic Table.
………………………………………………………………………………………………………………….
…………………………………………………………………………………………………………….. [1]
(b) For one of the metals in Table 5.1, name the products of its reaction with water.
…………………………………………………………………………………………………………….. [2]
(c) Name one other element that is in the same group as the elements in Table 5.1.
…………………………………………………………………………………………………………….. [1]
(d) Describe how the reactivity of the element in (c) would differ from the elements in Table 5.1.
…………………………………………………………………………………………………………….. [1]
www.KiasuExamPaper.com
142
11
6 An unknown metal M forms the nitrate MNO 3 . The compound is stable, but decomposes upon strong
heating.
When a 17.0 g sample of MNO 3 was heated, it decomposed completely according to the equation:
(a) Is the above decomposition reaction exothermic or endothermic one? Explain your answer.
………………………………………………………………………………………………………………….
…………………………………………………………………………………………………………….. [2]
[2]
(c) Calculate the molar mass of MNO 3 , and hence determine the identity of M.
(d) Describe a test you would perform to confirm the identity of the oxygen produced.
………………………………………………………………………………………………………………….
…………………………………………………………………………………………………………….. [2]
www.KiasuExamPaper.com
143
12
7 Group VII and Group 0 are found at the right side of the Periodic Table.
(a) What are the names given to elements in Group VII and Group 0?
…………………………………………………………………………………………………………….. [1]
………………………………………………………………………………………………………………….
…………………………………………………………………………………………………………….. [1]
Describe how she could use two of the substances to perform an experiment to show that
chlorine is more reactive than bromine. Include the observations you would expect her to make,
and construct a balanced chemical equation of the reaction.
………………………………………………………………………………………………………………….
………………………………………………………………………………………………………………….
………………………………………………………………………………………………………………….
………………………………………………………………………………………………………………….
…………………………………………………………………………………………………………….. [3]
www.KiasuExamPaper.com
144
13
8 Fig. 8.1 describes some of the reactions of two unknown substances A and B.
2. add
1. add dil. aqueous
nitric acid barium red solution
nitrate
Fig. 8.1
A: …………………………………………………………….
B: …………………………………………………………….
C: …………………………………………………………….
D: …………………………………………………………….
E: …………………………………………………………….
F: …………………………………………………………….
[6]
(b) Write a balanced chemical equation for any one of the reactions in Fig. 8.1.
…………………………………………………………………………………………………………….. [2]
www.KiasuExamPaper.com
145
14
Section C
9 Iron is extracted from iron ore in the blast furnace, as shown in Fig. 9.1 below.
haematite, coke,
waste gases
limestone
Fig. 9.1
(a) In the furnace, the coke is converted to carbon monoxide. A redox reaction then takes place
between iron(III) oxide in haematite and carbon monoxide to produce iron and carbon dioxide.
(i) Write a balanced chemical equation for the reaction between iron(III) oxide and carbon
monomonoxide.
…………………………………………………………………………………………………………….. [1]
………………………………………………………………………………………………………………….
………………………………………………………………………………………………………………….
…………………………………………………………………………………………………………….. [2]
…………………………………………………………………………………………………………….. [1]
www.KiasuExamPaper.com
146
15
(b) Pure iron from the blast furnace is frequently mixed with other elements to form alloys.
Give one example of this alloy, and explain why it is preferred to pure iron.
………………………………………………………………………………………………………………….
…………………………………………………………………………………………………………….. [2]
(c) Identify substance Y, and explain how it is formed. Include the relevant chemical equation(s) in
your answer.
………………………………………………………………………………………………………………….
………………………………………………………………………………………………………………….
………………………………………………………………………………………………………………….
………………………………………………………………………………………………………………….
…………………………………………………………………………………………………………….. [4]
www.KiasuExamPaper.com
147
16
10 Magnesium nitrate is commonly used as a dehydrating agent; it is also present in some fertilisers.
A student prepared a sample of magnesium nitrate by adding magnesium oxide to 200 cm3 of nitric
acid of an unknown concentration. The equation is shown below:
In this particular reaction, 7.4 g of magnesium nitrate was collected at the end of the reaction.
(a) Given that the nitric acid reacted completely, calculate the moles of nitric acid that reacted and
hence its concentration in mol/dm3.
[3]
(b) Outline an experimental procedure to describe how pure crystals of zinc nitrate may be prepared
using a similar method as above. State clearly the reagents that you use.
………………………………………………………………………………………………………………….
………………………………………………………………………………………………………………….
………………………………………………………………………………………………………………….
………………………………………………………………………………………………………………….
………………………………………………………………………………………………………………….
………………………………………………………………………………………………………………….
………………………………………………………………………………………………………………….
………………………………………………………………………………………………………………….
…………………………………………………………………………………………………………….. [6]
(c) Explain why sodium nitrate cannot be prepared with the method in (b).
………………………………………………………………………………………………………………….
…………………………………………………………………………………………………………….. [1]
End of Paper
www.KiasuExamPaper.com
148
www.KiasuExamPaper.com
149
West Spring Secondary School
Science Department – Mid-Year Exam [2018]
Marking Scheme
1(a)(ii)
ii) Oxide A CAO
CA 1
1(a)(iii)
iii) Oxide B CA
CA
CAO 1
is exothermic.
The reaction is exot
othe
hermic.
CAO 1
3(a) It gives burns vigorously/may
vig
gorously/may explode, signifying that a lot of heat is
OWTTE 1
given out to the surroundings.
Zinc oxide is reduced.
1
3(b) Zn gains 2 electrons from Zn2+ in ZnO to Zn. CAO
1
(Students need to specify no. of electrons to get the mark.)
www.KiasuExamPaper.com
150
4(b) Tube D. CAO 1
2 KBr + Cl 2 Æ 2 KC
KCl + B
Brr 2
1
A: iron
B:
B sulfuric
lf i acidid
8(a) C: iron(II) sulfate 1m each 6
D: hydrogen
E: barium sulfate
F: iron(II) hydroxide
Fe + H 2 SO 4 Æ FeSO 4 + H 2 CAO
FeSO 4 + 2NaOH Æ Na 2 SO 4 + Fe(OH) 2 1m for correct
8(b) formula, 1m for 2
FeSO 4 + Ba(NO 3 ) 2 Æ Fe(NO 3 ) 2 + BaSO 4 balanced
(Any one) equation.
www.KiasuExamPaper.com
151
Section C [20 marks]
Fe 2 O 3 + 3 CO Æ 2 Fe + 3CO 2 1
9(a)(i)
1m for stating
It is a redox reaction as Fe 2 O 3 is reduced to Fe, and CO is oxidised oxidised and
9(a)(ii) to CO 2 . reduced species 2
Fe 2 O 3 loses oxygen while CO gains oxygen. 1m for
explanation
9(a)(iii) CO is the reducing agent. CAO 1
(Stainless) steel is one iron-based alloy.
1
9(b) It is preferred as it is stronger / more corrosion resistant than pure CAO
1
iron.
Y is slag. CAO
CA
AO 1
The limestone added to the furnace decomposes to form calcium 1
oxide and carbon dioxide.
OWTTE
OWTTE
TE
The calcium oxide reacts with acidic impurities / silicon dioxide
e in
n
1
9(c) the haematite to form slag.
(1m can be given for the role of limestone in removing
gaacidic
ciid
c diic
impurities, without mention of its decomposition)
www.KiasuExamPaper.com
152
www.KiasuExamPaper.com
153
Name: Register Number: Class:
1 hour
Additional Material: Multiple Choice Answer Sheet
There are forty questions on this paper. Answer all questions. For each question there are four
possible answers A, B, C, and D.
Choose the one you consider correct and record your choice in soft pencil on the separate
Answer Sheet.
Each correct answer will score one mark. A mark will not be deducted for a wrong answer.
Any rough working should be done in this Question Booklet.
A copy of the Data Sheet is printed on page 16.
A copy of the Periodic Table is given on page 17.
The use of an approved scientific calculator is expected, where appropriate.
www.KiasuExamPaper.com
154
2
A pipette
B burette
C measuring cylinder
D electronic top pan balance
A filtration
B distillation
C crystallisation
D fractional distillation
3 An unknown white substance was found on a murdered victim’s body. A crime scene
investigator conducted a series of tests on the substance.
First, he dissolved the substance in water to form a colourless solution. Next, to a portion of
the solution, he added aqueous sodium hydroxide dropwise. A white precipitate was
formed. The precipitate did not dissolve when excess sodium hydroxide was added.
A zinc nitrate
B zinc sulfate
C calcium nitrate
D calcium sulfate
© BGSS 2018
[Turn Over
www.KiasuExamPaper.com
155
3
5 An element has the electronic configuration, 2.8.4. What is the identity of the element?
A sulfur
B silicon
C carbon
D nitrogen
X Y
A sodium sulfur
B magnesium chlorine
C oxygen carbon
D nitrogen oxygen
7 Using the equation shown below, find the volume of oxygen needed to completely react
with 36 dm3 of CH 4 at room temperature and pressure.
A 24 dm3
B 36 dm3
C 48 dm3
D 72 dm3
© BGSS 2018
[Turn Over
www.KiasuExamPaper.com
156
4
8 In a village, before using a cast iron wok, lemon juice is used to clean the surface of the
wok. This is because
A the lemon juice is acidic and will react with iron surface to produce iron(II) salt.
B the lemon juice is acidic and will react with iron surface to produce iron(III) salt.
C the lemon juice is acidic and will remove the iron(III) oxide that is found on the
surface.
D the lemon juice is acidic and will add on to the iron(III) oxide that is found on the
surface.
9 Aluminium oxide can react with both acid and alkali to form a salt. What type of oxide is it?
A basic
B acidic
C neutral
D amphoteric
10 What result is obtained when liquid bromine is added to aqueous potassium chloride?
A A brown vapour forms.
B A silvery solid is formed.
C A green gas is produced.
D There is no visible reaction.
A Element Z sinks to the bottom of the container and no bubbles was seen.
B Element Z floats on the surface of the water and some bubbles were seen.
C Element Z sinks to the bottom of the container and effervescence was seen.
D Element Z darts around the surface of the water, effervescence and sparks
were seen.
© BGSS 2018
[Turn Over
www.KiasuExamPaper.com
157
5
12
beaker 1 beaker 2 beaker 3
The diagrams above show the results obtained when three different metallic discs of the
same shape and size were dropped into dilute hydrochloric acid separately.
Which statement(s) can be deduced from its position in the reactivity series?
A I only
B I and III
C II and III
D I, II and III
14 Carbon monoxide, sulfur dioxide and oxides of nitrogen are all common pollutants of air.
Which pollutant is shown with its correct source and its adverse effect on the environment?
© BGSS 2018
[Turn Over
www.KiasuExamPaper.com
158
6
15 Ammonium chloride was added to distilled water. Which graph correctly shows the change
in temperature?
A temperature/oC B temperature/oC
time/s time/s
C temperature/oC D temperature/oC
time/s
time/s
16 Acidified potassium manganate(VII) can be used to detect the presence of ethanol vapour
in the breath of a person who has consumed alcohol.
If ethanol is present, a colour change is observed. What is the colour change observed?
A colourless to brown
B purple to colourless
C colourless to purple
D brown to colourless
© BGSS 2018
[Turn Over
www.KiasuExamPaper.com
159
7
mass of beaker/g
time/min
A I only
B II only
C I and II only
D II and III only
A B
C D
© BGSS 2018
[Turn Over
www.KiasuExamPaper.com
160
8
A B
C D
20 What is the structure of the product formed when ethene gas is passed through aqueous
bromine?
A B
C D
© BGSS 2018
[Turn Over
www.KiasuExamPaper.com
161
Name: Register Number: Class:
4E 4E
5N Preliminary Examination 2018
5N
5076/03
SCIENCE (CHEMISTRY)
5078/03
Paper 3 Theory 15 August 2018
1 hour 15 minutes
Candidates answer on the Question Paper.
Write your name, register number and class on all the work you hand in.
You may use an HB pencil for any diagrams, graphs, tables or rough working.
Write in dark blue or black pen.
Do not use staples, paper clips, glue or correction fluid.
Section A
Answer all questions.
Write your answers in the spaces provided on the question paper.
Section B
Answer any two questions.
Write your answers in the spaces provided on the question paper.
At the end of the examination, fasten all your work securely together.
The number of marks is given in brackets [ ] at the end of each question or part question.
Section A
+ + +
+ + +
+ + +
+ + +
+ + +
+ + +
A B C
+
+
+
+
D E F
Fig 1.1
………………………………………..……………………………………………………...[1]
2 Table 2.1 gives some information on the component gases of clean air in the atmosphere.
Table 2.1
Separating air into its component gases is an important process in the industries. Air is
first cooled to liquid at 200 oC before it is gradually warmed up and separated into its
component gases, as illustrated in Fig. 2.1.
stage 1 stage 2
nitrogen
air
oxygen
cooling liquid air at
200 oC
heater
argon, carbon dioxide
and water removed
Fig. 2.1
(a) Describe the arrangement and movement of the particles in the liquid air.
………………………………………………………………………………………………….
…………………………………………………………………………………………...…..[2]
(b) Explain why argon, carbon dioxide and water can be removed when air is compressed
and cooled to 200 oC in stage 1.
…………………………………………………………………………………………….....[1]
(c) Name the method used to separate the components of liquid air in stage 2.
………………………………………………………………………………………….…....[1]
(d) Which component will be collected last in stage 2? Explain your answer.
………………………………………………………………………………………….…....[2]
(e) Which component will have the largest volume collected in stage 2?
……………………………………………………………………………………...….…….[1]
3 (a) Sir James Jeans, who was a great populariser of science, once described an atom of
carbon as being like six bees buzzing around a space the size of a football stadium.
(i) Suggest what were represented by the six bees in this description.
………………………………………….………………………………..…….……...[1]
(ii) What is missing from Jeans’ description when applied to an atom of carbon?
………………………………………………..….…………………………..….…….[1]
(i) State, in terms of atomic structure, the difference between the two isotopes of
carbon.
………………………………………………………………………………….……...…
…………………………………………………………………………………...…….[1]
(ii) Draw a diagram to show the arrangement of electrons in an atom of the 126C
isotope.
[1]
In an investigation of a case which involves a bank cheque issued with a forged signature,
the sample of ink from the forged signature was tested together with inks from the pens of
five suspects V, W, X, Y and Z. Fig. 4.1 shows the chromatogram that was obtained with
the use of an organic solvent.
solvent front
Ɣ Ɣ Ɣ
Ɣ
Ɣ Ɣ Ɣ
Ɣ Ɣ Ɣ Ɣ
start line Ɣ
drawn in pencil ink from V W X Y Z
forged
signature inks from pens of suspects
Fig. 4.1
(a) Which suspect is most likely to have forged the signature in the bank cheque?
………………………………………………………………………………………............[1]
(b) Suggest why the ink from the pen of suspect W gives only one spot on the
chromatogram.
………………………………………………………………………………………............[1]
(c) Explain why the start line is drawn in pencil but not in pen.
……………………………………………………………………………………………...…..
……………………………………………………………………………………...….........[2]
(d) Suggest a reason why water would probably not be a suitable solvent to use for this
chromatography.
…………………………………………………………………………………...…….........[1]
5 Haematite, a common ore used for the extraction of iron, contains the compound, iron(III)
oxide. Iron is produced in the blast furnace by heating a mixture of iron(III) oxide, coke and
limestone with air. Fig. 5.1 gives the outline of a blast furnace in which iron is extracted
from its ore.
(a) Describe how carbon dioxide is produced under high heat in the blast furnace.
………………………………………………………………………...……………................
…………………………………………………………………………………...….............[2]
(b) The chemical equation for the production of iron in the blast furnace is shown.
…………………………………………………………………………….……….......[1]
…………………………………………………………………...……………................
……………………………………………………………………………...…............[2]
6 (a) A 500 cm3 aqueous sample contains 37.6 g of copper(II) nitrate. Calculate the
concentration of the copper(II) nitrate solution in mol/dm3.
(b) When a piece of zinc is added to the copper(II) nitrate solution, a reddish brown solid
is formed. Explain the observation.
……………………………………………………………….………………………………….
……………………………………………………………….………………………………….
………………………………………………………………………………………………..[2]
add add Y
X Z
dilute nitric acid aqueous green precipitate,
white green
and aqueous sodium insoluble in excess
precipitate solution
silver nitrate hydroxide aqueous sodium
hydroxide
Fig. 7.1
X ……………………………………………………..
Y ……………………………………………………..
Z ……………………………………………………..
[3]
(b) Write a balanced chemical equation for any one of the reactions in Fig. 7.1.
…………………………………………………………………………………………..…...[2]
© BGSS 2018 [Turn Over
www.KiasuExamPaper.com
168
8
Table 8.1
K 2 SO 4 soluble
[5]
(b) (i) Give the names of the two reagents in Table 8.1 that can be used to prepare a
white precipitate of calcium carbonate.
1…………………………………………………….
2…………………………………………………….
[1]
(ii) Describe the process of how you could prepare a pure dry sample of calcium
carbonate from the two reagents you have given in (b)(i).
…………………………………………………………………………………………….
…………………………………………………………………………………………….
…………………………………………………………………………………………….
……………….…………………………………………………………….......………[3]
Section B
Answer any two questions in this section.
9 Fluorine is an element found in Group VII. It would react vigorously with potassium, from
Group I, to form potassium fluoride. It would also react with hydrogen gas to form hydrogen
fluoride.
(a) Write a balanced chemical equation for the reaction of fluorine and potassium.
……………………………………………………………………………………….……....[2]
(b) Draw and label the electronic structures of potassium fluoride and hydrogen fluoride.
potassium fluoride
hydrogen fluoride
[4]
(c) Use these structures to explain why, at room temperature and pressure, potassium
fluoride is a solid and hydrogen fluoride is a gas.
………………………………………………………………………………………………….
………………………………………………………………...……………………………..…
………………………………………………………………………………………………….
………………………………………………………………………………………………….
…………………………………………………………………………….……………....…[4]
………………………………………………………………….….……………………
……………………………………………………………………..…………………...
………………………………………………...………………………..……………[2]
…………………………………………………………………..………………...…[1]
(i) Complete Table 10.1 by naming and drawing the full structural formula of X.
Table 10.1
[2]
(ii) Give the chemical formula of the organic substance that is formed when X
reacts with atmospheric oxygen.
…………………………………………………………………………….…………[1]
……………………………………………………………………………………..……
………………………………………………………………………………….……….
………………………………………………………………………………………..…
……………………………………………...…………………….……….…………[3]
(iv) X is commonly used in Brazil as a fuel for vehicles. Suggest a reason why X
can be used for this purpose.
………………………………………………………………………….……………[1]
11 When hydrochloric acid reacts with strips of magnesium, a gas is given off.
(a) Draw a diagram to show how you would investigate the speed of the reaction
between hydrochloric acid and magnesium in an experiment. Describe how you
would find the speed of the reaction based on the data collected in this experiment.
………………………………………………………………………………….……………..
………………………………………………………………………………………….……..
…………………………………………………………………………………………….…..
…………………………………………………………………………………………….…..
………………………………………………………………………………………..…….[4]
(b) State and explain one way of increasing the speed of this reaction. Use your
knowledge of reacting particles in your explanations.
……………………………………………………..………………………………………….
…………………………………………………………………………………………..…….
…………………………………………………………………………………………….…..
………………………………………………………………………………………..…….[3]
(c) Give the name of the gas given off in this reaction.
…………………………………………………………………………………..………….[1]
(d) G is the next member in the same group that magnesium belongs to.
………………………………………………..……………...……………………….[1]
………………………………………………..………………………….......……….[1]
Data Sheet
1 B 11 D
2 D 12 B
3 C 13 C
4 D 14 C
5 B 15 D
6 A 16 B
7 D 17 A
8 C 18 A
9 D 19 B
10 D 20 C
www.KiasuExamPaper.com
175
BEDOK GREEN SECONDARY SCHOOL
SCIENCE DEPARTMENT
MARKING SCHEME
YEAR (2018)
PAPER 3
Section A: 45 marks
[1] [1]
BGSS 2018
Page 1 of 5
www.KiasuExamPaper.com
176
5 (a) Carbon dioxide is formed when coke is burnt in hot air and when [1] [2]
limestone is decomposed at high temperature. [1]
(will not accept CO reducing Fe 2 O 3 equation given in part (b). CO 2 is
produced mainly by oxidation under this reaction.)
(b) (i) +3 [1] [1]
(c) (ii) The oxidation state of iron has decreased and thus, it is reduced. [1] [2]
Iron’s oxidation state has decreased from +3 in iron(III) oxide to 0 [1]
in iron.
8 (a) [5]
name of chemical formula solubility
so
s y iin
lubility
ty n water
substance
[1]
sodium carbonate Na 2 CO
O3 soluble
BGSS 2018
Page 2 of 5
www.KiasuExamPaper.com
177
Section B: 30 marks
K F
tran
nsfer of electr
correct transfer ron a
electron nd charge
and [1]
coorr
rrec
e t number
correct numbber of electrons [1]
10 (a) (i) Member of the same homologous series have similar chemical [1] [2]
properties and
they display a gradual change in their physical properties as the [1]
number of carbon atoms increases in their molecules.
(will not accept same functional groups or general formula because
question ask for properties and not structure.)
(ii) C n H 2n+1 OH [1] [1]
(b) (i) [2]
name of X structural formula of X
ethanol
BGSS 2018
Page 3 of 5
www.KiasuExamPaper.com
178
The conical flask is connected through a delivery tube to a test tube
with limewater to prevent oxygen in air from entering the conical [1]
flask.
*please emphasize to students that a brief mention of the 3
conditions with no proper description of experiment will be
penalised one mark in the exams because the question asked for
description of process.
(iv) X can be burnt exothermically to produce heat to power the [1] [1]
vehicles.
11 (a) [4]
agnesium
magnesium
stto
oppw
wa
attc
stopwatch ch
ch
OR
cotton
c otton wool
mag
gnes
nesium
ium
magnesium
[1]
[1]
stopwatch
diagram
label
[1]
Plot a grrap
aph
graphh of volume of gas produced against time. OR
Plot a graaph of mass of contents in flask against time.
graph
relevant graph to the diagram drawn. [1]
(Students are not required to plot/sketch the graph)
The speed of reaction can be determined by the gradient of graph.
(Stopwatch needs to be mentioned “once” either in diagram or description
to be awarded mark either in diagram or description.
(b) Increasing the temperature of reaction mixture [1] [3]
will increase the kinetic energy / speed of reacting particles. [1]
OR OR
Use powdered magnesium instead of strips of magnesium provides [1]
smaller particle size of magnesium which provide a larger surface [1]
area for collision.
OR OR
[1]
BGSS 2018
Page 4 of 5
www.KiasuExamPaper.com
179
e H+ ions
Use a higher concentration of hydrochloric acid provides more
mor
ore s [1]
[1]
per unit volume for collision with magnesium particles.
of the
either of tth
he above
ab
a bo
ovve answers
a swer
an errs
[1]
[1
1]
Higher frequency of effective collision bet twe
betweenee n H+ ion
en ions
ns and
magnesium particles increases the speed of reaction.
reaction n.
Note: Students need to mention particles,
iccles, magnesium
n the reacting partic sium and
magnessium d
+
acid particles (H ions) at least once in the answers.
(c) hydrogen [1] [1]
(d) (i) Ca [1] [1]
(ii) They have the same number of valencevallence electrons.
ele
ect
ctro
rons
ns.. OR [1] [1]
They both have two valence e electrons.
elec
el ec
ctr
trons
s.
BGSS 2018
Page 5 of 5
www.KiasuExamPaper.com
180
www.KiasuExamPaper.com
181
2018 4E Science Chemistry SA2 - Bedok South Secondary School
Which apparatus are needed to produce and collect pure water from seawater?
A 2 and 5 B 3 and 5
C 1, 2 and 4 D 1, 4 and 5
A 1, 2 and 3 B 1, 2 and 4
C 1, 3 and 4 D 2, 3 and 4
www.KiasuExamPaper.com
182
24 A sugar mixture was compared with four different simple sugars using chromatography.
The results are shown in diagram below. What types of sugars does the mixture contain?
A H2O B HCl
C CaSO 4 D NO
26 Which of the following compounds has the highest percentage of nitrogen by mass?
A NH 4 NO 3 B (NH 4 ) 2 CO 3
C CO(NH 2 ) 2 D NH 4 Cl
27 A student dissolved 14.9g of potassium chloride, KCl, in 100 cm3 of water. What is the
concentration of the resulting potassium chloride solution in mol/dm3?
A 0.002 mol/dm3
B 0.01 mol/dm3
C 0.15 mol/dm3
D 2.0 mol/dm3
www.KiasuExamPaper.com
183
28 The graph below shows the colour ranges of the acid-base indicators methyl orange,
bromothymol and phenolphthalein.
A solution, when placed in the three indicators separately, is yellow in methyl orange, yellow
in bromothymol and colourless in phenolphthalein. What is the pH range of the solution?
29 Which of the following elements burns in air to produce a substance which can react with
both hydrochloric acid and sodium hydroxide?
A lead B hydrogen
C iron D phosphorous
30 Which of the following reagents cannot be used to differentiate sodium hydroxide solution
from sodium chloride solution?
www.KiasuExamPaper.com
184
31 Separate samples of hydrogen peroxide are added to aqueous potassium iodide and to
acidified potassium manganate(VII). It is known that hydrogen peroxide is both an oxidising
agent and a reducing agent.
X forms an acidic oxide, Y forms a basic oxide and Z forms an amphoteric oxide.
If X, Y and Z are placed in increasing order of atomic number (lowest atomic number first),
which order is correct?
A X, Y, Z B Y, Z, X
C Y, X, Z D X, Z, Y
33 Rubidium is in the same group as sodium in the Periodic Table. What is a likely property of
rubidium?
www.KiasuExamPaper.com
185
34 Which of the following experiment will have the fastest speed of reaction?
35 The element chromium produces hydrogen from dilute hydrochloric acid but it does not
react with cold water. When a piece of chromium is placed in lead(II) nitrate solution, solid
of lead appear.
What is the order of decreasing reactivity of the metals lead, calcium and chromium?
A B C D
www.KiasuExamPaper.com
186
37 Which of the following shows the correct percentage composition of oxygen, nitrogen and
carbon dioxide found in dry unpolluted air?
38 Which of the following shows the correct use of the different fractions of petroleum?
Fraction Uses
A Petrol used for making chemical feedstock
B Bitumen used for lubricating machine parts
C Kerosene used as fuel for aircraft
D naphtha used to pave road
A C2H4 B C2H6
C C 2 H 5 COOH D C 2 H 5 OH
40 Which of the following is the same for both ethanol and ethanoic acid?
A empirical formula
B functional group
C number of carbon
D homologous series
www.KiasuExamPaper.com
187
Data Sheet
www.KiasuExamPaper.com
188
189
www.KiasuExamPaper.com
BEDOK SOUTH SECONDARY SCHOOL
PRELIMINARY EXAMINATION 2018 4E/5NA
CANDIDATE
NAME
CLASS REGISTER
NUMBER
Write your name, register number and class on all work you hand in.
You may use an 2B pencil for any diagrams, graphs, tables or rough working.
Write in dark blue or black pen.
Do not use staples, paper clips, glue or correction fluid.
At the end of the exam, fasten all your work securely together.
The number of marks in given in brackets [ ] at the end of each question or part question.
Section A
Section B
Section C
Section A
Answer all questions in the spaces provided.
A1 The diagram below shows the formulae of some gases found in polluted air.
O2 CO2 NO
NO2 N2 CO
SO2
Choose formulae from the diagram to answer the following questions (a) to (d). Each may be
use once, more than once or not at all.
(a) Give the formula of a gas that is produced by incomplete combustion of fuels. State the
harmful health effect of this gas.
………………………………………………………………………………………………...
………………………………………………………………………………………………... [2]
(b) Give the formulae of two gases that are produced by reactions in catalytic converters.
(c) Give the formulae of two gases that are involved in both respiration and photosynthesis.
(d) Give the formulae of two gases that produce acid rain.
5076/03/BDS4E5N/Prelim/18
www.KiasuExamPaper.com
191
3
(a) A sample of sulfur from a volcano contained two different types of sulfur isotopes: sulfur-32
and sulfur-34.
(i) Complete the table below to show the atomic structure of each isotope of sulfur.
Number of
Isotope
Proton Neutron Electron
Sulfur-32
Sulfur-34
[2]
(ii) The relative atomic mass of sulfur is 32.2. Explain why does the relative atomic mass
of sulfur is not a whole number.
………………………………………………………………………………………………………...
……………………………………………………………………………………………….…. [2]
(b) One of the gases produced during volcanic eruptions is hydrogen sulfide. H 2 S. Hydrogen
sulfide is a poisonous, colourless gas which smells of rotten eggs.
[2]
(ii) Explain, in terms of bonding and structure, why hydrogen sulfide gas does not conduct
electricity.
…………………………………………………………………………………………………………
………………………………………………………………………………………………….. [2]
[Total: 8 marks]
5076/03/BDS4E5N/Prelim/18
www.KiasuExamPaper.com
192
4
A3 The table below shows some salts and products that contain them.
Salt product
Silver chloride Photographic film
Potassium nitrate fertiliser
Zinc sulfate Health supplement
Salt: …………………………
(ii) Which salt in the table can be made by titration? Suggest two reagents needed to make
this salt.
Salt: …………………………
Use
Calcium
silicate
Calcium
hydroxide
Argon
Sulfuric acid
[2]
[Total: 6 marks]
5076/03/BDS4E5N/Prelim/18
www.KiasuExamPaper.com
193
5
(a) What is the physical property that allows the various fractions in crude oil to be separated?
……………………………………………………………………………………………….. [1]
(b) To meet the world’s demand for petrol, heavier fraction such as diesel undergoes cracking
to produce lighter fractions as shown in the equation below.
C 12 H 26 Æ C 6 H 14 + C2H4 + product
C 4 H 8P
(a) Complete the equation for the reaction by filling in missing state symbols.
2 Al (……..) + Fe 2 O 3 (……..) Æ 2Fe (……..) + Al 2 O 3 [1]
(……..)
(b) (i) The table shows some information about oxidation state changes during the reaction.
Complete the table.
……………………………………………………………………………………………………….
…………………………………………………………………………………………………. [1]
5076/03/BDS4E5N/Prelim/18
www.KiasuExamPaper.com
194
6
……………………………………………………………………………………………………….
…………………………………………………………………………………………………. [2]
(d) Predict if the melting point of aluminium oxide is high or low. Explain your answer in terms
of structure and bonding.
……………………………………………………………………………………………………….
……………………………………………………………………………………………………….
……………………………………………………………………………………………………….
…………………………………………………………………………………………………. [2]
[Total: 8 marks]
A6 Common keys are made from steel. One problem with using steel is that the iron in steel will rust.
The diagram shows the cycle of changes that happens when iron in a steel key rust and then
extracted.
iron
extraction
processed
by adding
non-metal A
rusting
iron(III) oxide
(main
component in
both rust and
iron ore)
Steel
removed
by spray
……………………………………………………………………………………………………….
……………………………………………………………………………………………………….
5076/03/BDS4E5N/Prelim/18
www.KiasuExamPaper.com
195
7
………………………………………………………………………………………………….. [2]
(b) A shop sells a spray-on rust treatment. The spray contains particles of zinc. Explain how
zinc prevents rust from forming.
………………………………………………………………………………………………………...
…………………………………………………………………………………………………………
………………………………………………………………………………………………..… [2]
(c) Write a balanced chemical equation for the extraction of iron in the blast furnace.
…………………………………………………………………………………………...……... [1]
(d) Though the extraction of iron from blast furnace is a relatively cheap process, steels are still
widely recycled.
Explain the importance of recycling of metals such as iron.
………………………………………………………………………………………………………...
………………………………………………………………………………………………..… [1]
[Total: 7 marks]
A7 (a) Propane burns completely in oxygen to form carbon dioxide and water.
The equation for the reaction is
C 3 H 8 + 5O 2 o 3CO 2 + 4H 2 O
[1]
(ii) Hence, calculate the volume of carbon dioxide that is produced from 44 g of propane
at room temperature and pressure.
5076/03/BDS4E5N/Prelim/18
www.KiasuExamPaper.com
196
8
[2]
(b) (i) State why propene can be made into polymer but propane cannot.
……………………………………………………………………………………………………..….
……………………………………………………………………………………………… [1]
……………………………………………………………………………………………………….
……………………………………………………………………………………………… [2]
………………. ……………………………………………………………………………………….
……………………………………………………………………………………………… [1]
(c) The figure below shows the structure formula of part of an addition polymer.
F H F H F H F H F H
H H H H H H H H H H
Deduce and draw the structural formula of the monomer from which this polymer is made.
[1]
5076/03/BDS4E5N/Prelim/18
www.KiasuExamPaper.com
197
9
[Total: 8 marks]
Section B
Answer any two questions in this section.
Write your answers in the spaces provided.
B8 (a) Explain why sulfuric acid can act as an acid and why potassium hydroxide can act as an
alkali. Give examples of chemical reaction that sulfuric acid and potassium hydroxide
undergo.
……………………………………………………………………………………………………….
……………………………………………………………………………………………………….
……………………………………………………………………………………………………….
……………………………………………………………………………………………………….
……………………………………………………………………………………………………….
……………………………………………………………………………………………………….
……………………………………………………………………………………………………….
……………………………………………………………………………………………………….
……………………………………………………………………………………………………….
……………………………………………………………………………………………………….
……………………………………………………………………………………………………….
……………………………………………………………………………………………………….
……………………………………………………………………………………………………….
………………………………………………………………………………………………… [4]
(b) Write the ionic equation that describes the reaction of an acid with an alkali
………………..………………………………………………………………………………. [1]
5076/03/BDS4E5N/Prelim/18
www.KiasuExamPaper.com
198
10
(c) The diagram below shows some of the properties and reactions of the substances A, B, C,
D and E.
A
green solid
add aqueous
B C E
colourless gas blue solution barium nitrate white
precipitate
white D
blue
precipitate precipitate
(d) The formation of white precipitate E shows the presence of sulfate ions.
Why does this not prove that sulfate ions are present in solid A?
……………………………………………………………………………………………………….
………………………………………………………………………………………………… [1]
[Total: 10 marks]
5076/03/BDS4E5N/Prelim/18
www.KiasuExamPaper.com
199
11
(i) State the effect that each of these has on the speed of a reaction.
(ii) Use your knowledge of reacting particles to explain your answer to (a)(i).
……………………………………………………………………………………………………….
……………………………………………………………………………………………………….
……………………………………………………………………………………………………….
……………………………………………………………………………………………………….
……………………………………………………………………………………………………….
……………………………………………………………………………………………………….
……………………………………………………………………………………………………….
……………………………………………………………………………………………………….
……………………………………………………………………………………………………….
……………………………………………………………………………………………………….
……………………………………………………………………………………………………….
……………………………………………………………………………………………………….
……………………………………………………………………………………………………….
……………………………………………………………………………………………………….
……………………………………………………………………………………………………….
………………………………………………………………………………………………… [5]
5076/03/BDS4E5N/Prelim/18
www.KiasuExamPaper.com
200
12
(b) A student carried out an experiment to investigate how the speed of reaction between
magnesium and hydrochloric acid will change with time.
(i) Draw a labelled diagram to show the experiment setup that the student use.
(ii) Describe how the student will carry out the experiment, clearly stating the physical
quantity he will measure.
(iii) Describe how the speed of this reaction would change with time.
……………………………………………………………………………………………………….
……………………………………………………………………………………………………….
……………………………………………………………………………………………………….
……………………………………………………………………………………………………….
……………………………………………………………………………………………………….
……………………………………………………………………………………………………….
……………………………………………………………………………………………………….
……………………………………………………………………………………………………….
……………………………………………………………………………………………………….
……………………………………………………………………………………………………….
……………………………………………………………………………………………………….
………………………………………………………………………………………………… [5]
[Total: 10 marks]
5076/03/BDS4E5N/Prelim/18
www.KiasuExamPaper.com
201
13
B10 (a) What is the common name given to elements in Group VII?
……………………………………………………………………………………………… [1]
(b) Give the electronic structures of fluorine and chlorine and use these to explain why they
are placed in Group VII.
……………………………………………………………………………………………………
……………………………………………………………………………………………………
……………………………………………………………………………………………… [2]
(c) Chlorine was discovered by Carl William Scheele in 1774 at Sweden. The origin of the
name came from the Greek word "chloros" meaning "pale green".
In 1886, a new element was discovered. Based on its electronic structure, colour and its
reaction with zinc chloride, this new element was placed above chlorine in Group VII of
the Periodic Table and given the name fluorine.
……………………………………………………………………………………… [1]
(ii) Suggest how the colour of fluorine could help explain its position in the Periodic
Table.
………………………………………………………………………..……………… [1]
(iii) Describe what would be observed when fluorine is bubbled into a solution of
potassium bromide. Explain your observation.
……………………………………………………………………………………………………
……………………………………………………………………………………………………
……………………………………………………………………………………………… [2]
5076/03/BDS4E5N/Prelim/18
www.KiasuExamPaper.com
202
14
(d) The element with an atomic number of 85 is so unstable that it has never been seen by
the naked human eye.
(i) Consider the properties of other elements in the same group as this element, predict
one physical and one chemical property of the element with atomic number 85.
……………………………………………………………………………………………………
……………………………………………………………………………………………………
……………………………………………………………………………………………………
……………………………………………………………………………………………… [2]
(ii) Give the chemical formula of the compound formed between magnesium and the
element with atomic number 85.
……………………………………………………………………………………………… [1]
[Total: 10 marks]
End of Paper
5076/03/BDS4E5N/Prelim/18
www.KiasuExamPaper.com
203
15
Data Sheet
5076/03/BDS4E5N/Prelim/18
www.KiasuExamPaper.com
204
16
205
16
5076/03/BDS4E5N/Prelim/18
www.KiasuExamPaper.com
206
www.KiasuExamPaper.com
2018 Bedok South Secondary School Secondary 4
Science(Chemistry) PRELIM Marking Scheme
Paper 1: 30 Marks
21 22 23 24 25 26 27 28 29 30
D B C D A C D B A C
31 32 33 34 35 36 37 38 39 40
A B A D A B C C B C
Answer
A1 (a) CO 1
Prevents blood from absorbing oxygen which cau
causes
use s headaches,
ses hea
he adache
hes,
he s
s,
giddiness or may lead to death. 1
(b) N 2 and CO 2 (both
((b
bootth must
stt b
bee co
correct) 1
(c) CO 2 and O 2 ((both
(bbo
otth mu
must
st b
bee co
corr
correct)
rrr
rrect) 1
(d) NO 2 and SO 2 ((both
both mu
m
must
st b
bee co
correct) 1
[[Total:
Total:
l: 5 marks]
A2 (ai) N mb
Nu berr of
Number of
Isotope
Proton
P
Pr
rootton
o Neutron
Neutro
on Electron
Sulfur-32
Sulfur-3
32 16 32-
32- 16 = 16 16 1
Sulfur-34
S
Su
ulffur
u -34
4 16 34 – 16 = 18 16 1
(aii) Eachh sulfur
sul
u fu
ur isotope
i otop
is pe ha
has s different
d fferent relative abundance/ percentage/
di 1
amount.
amou unt
n.
When
W
Wh en the av
aver
average
erag
agee of the
the masses of the 2 sulfur isotope is taken, there 1
is decimal.
decim
mal
a . (any
( ny p
(a phrase
hrase to the effect)
(bi) Correct valence electron for sulfur and 1
hydrogen
X Correct number of shared electrons (2 1
X single bond)
www.KiasuExamPaper.com
207
A3 (ai) Salt: Silver chloride 1
(salt Reason: It is an insoluble salt. 1
pre)
(aii) Salt: Potassium nitrate (SPA – titration (neustralisation)) 1
Reagent 1: potassium hydroxide Reagent 2: nitric acid (both correct) 1
(b) Use
Sulfuric acid ¥
(bii) Alum
Al um
min
niu
Aluminiumum is
i oxidised while iron is reduced, since oxidation and
redu
re duct
ctio
ion occur simultaneously, Thermit reaction is a redox reaction.
reduction 1
www.KiasuExamPaper.com
208
(aii) [P1] Carbon will disrupt the orderly arrangement of iron, (ALLOY) 3 pt –
[P2] making it more difficult for the iron atoms to slide past each other, 2M
[P3] thus increasing the strength of iron. (any phrasing to the effect) 2 pt –
1M
(b) [P1] zinc is more reactive than iron / zinc has higher tendency to lose 1
its electrons,
[P2] zinc will preferentially corrode in place of iron. 1
(c) Fe 2 O 3 + 3CO Æ 2Fe + 3CO 2 1
(d) The earth’s mineral ores are limited and are non-renewable. Recycling 1
helps to conserve the limited resources in our earth and make them last
longer.
With a decrease of mining for ores, land will be free for other uses eg,
agriculture.
monomer (alkene)
[Total: 8 marks]
B8 (a) [P1] An acid is a substance which produces hydrogen ions when it is 1
dissolved in water.
[P2] Example: Sulfuric acid reacts with reactive metal to produce salt
and hydrogen gas/ sulfuric acid reacts with carbonates to produce salt, Any
www.KiasuExamPaper.com
209
water and carbon dioxide gas. Sulfuric acid react with base/alkali to one 1
produce salt and water.
[P4] Example: sodium hydroxide reacts with ammonium salt to form salt,
water and ammonia gas. (full credit if formulae/ chemical equation 1
given)
[Total:
[Tot
[T otal
al:: 10 m
marks]
arks]
B9
9 (a) [Etemp] when temperature is increases, speed of ch
chemical
hem a reaction
miccal reactio
on 1
increases.
[Econc] when concentration decreases, sp
speed
pee
ed of
of cchemical
he
h em
miic
caal reaction
reac
reacti
ac tion
ti on 1
decreases.
[Rtemp] when temperature increases, particles gains kinetic
kinettic
c energy
e ergy
en gy a
and
nd
move faster. Frequency of effective collision
n will increase
es .
increases. 1
[1] [ ]
[1]
Student will record the volume of Student will record the decrease in
hydrogen gas [1] produced at mass of reaction mixture [1] at 2
regular interval. [1] regular interval [1].
www.KiasuExamPaper.com
210
[Total:
[Tot
otal
ot al:: 10 marks]
al
(ci) Yellow 1
[explanation] fluorine is
is momore
ore rreactive
eact
ea ctiive than bromine, thus it will
displace bromine from
om p
potassium
ottas
assi
sium
um bromide
bro
romi
m de and produce bromine. 1
(di) [physical]
[physica
c l]] cannot
ca conduct
con
onduduct electricity/ black colour/ solid at room 1
temperature
temp
m errat ure [any
atur ny one]
ne]] (do
one (do NOT
(d N T write “high/low” melting point)
NO
[che
hemi
mical] gain
[chemical] gain 1 electron
ele
l ctron to form anion/ least reactive in group VII/
le
wth metal
reacts wth met
etal to form ionic compound/ reacts with non-metal to form 1
al to
cova
co vale
lent
covalentnt ccompounds.
ompo
om pounds. [any one]
(dii) MgAt 2
MgAt 1
[Total: 10 marks]
www.KiasuExamPaper.com
211
www.KiasuExamPaper.com
212
9
Bendemeer Secondary School Bendemeer Secondary School Bendemeer Secondary School Bendemeer Secondary School Bendemeer Secondary School Bendemeer Secondary School
emeer Secondary Schoo
Bendemeer School Bendemeer Secondary School Bendemeer Secondary School Bendemeer Secondary School Bendemeer Secondary School Bendemeer Secondary School
emeer
Bendemeer
em
eerr Secondary
meeer
Secco
Se conon
o
eeeerr Secondary
Bendemeer Seeco
S ccon
nd
o d
daar
dar
dar
BENDEMEER SECONDARY SCHOOL
ary
ry S
arry S
choo Bendemeer Secondary School Bendemeer Secondary School Bendemeer Secondary School Bendemeer Secondary School Bendemeer Secondary School
School
choo Bendemeer Secondary School Bendemeer Secondary School Bendemeer Secondary School Bendemeer Secondary School Bendemeer Secondary School
ch
School
em
meeeer S
Bendemeer
Bendemeer
em
meee
eer
eeco
ec
cconda
Secondary
er Secondary
er Secon
Se ccoonddar
ar2018 PRELIMINARY EXAMINATION
daary
ary
y Sch
ry S
cch
School
School
cch
hoo
o Bendemeer Secondary School Bendemeer Secondary School Bendemeer Secondary School Bendemeer Secondary School Bendemeer Secondary School
hoo Bendemeer Secondary School Bendemeer Secondary School Bendemeer Secondary School Bendemeer Secondary School Bendemeer Secondary School
em
meeer
eeer
Bendemeer er Secondary
Seecco
S on
o nnd
daar
dar
d ary S cch
School ho Bendemeer Secondary School Bendemeer Secondary School Bendemeer Secondary School Bendemeer Secondary School Bendemeer Secondary School
hoo
em
meee
eer
Bendemeer err Secondary
Seccon
Se co
on
o nddar
da SECONDARY 4 EXPRESS / 5 NORMAL (ACADEMIC)
aarry S ch
ch
School hooo Bendemeer Secondary School Bendemeer Secondary School Bendemeer Secondary School Bendemeer Secondary School Bendemeer Secondary School
em
mee
eeer
Bendemeer eerr Secondary
Seecco
S con
onnd
o dar
daar
ary S ch
ccho
School hoo Bendemeer Secondary School Bendemeer Secondary School Bendemeer Secondary School Bendemeer Secondary School Bendemeer Secondary School
em
meeer
eeeerr Secondary
Bendemeer
Bendemeer
emee
Secco
Se con
onnda
o
eeerr Secondary
eer
ee Seeccon
S co
ond
on daar
dar
SCIENCE (PHYSICS, CHEMISTRY) PAPER 1
darry S
ary S
School
School
cho
cchho
h oo Bendemeer Secondary School Bendemeer Secondary School Bendemeer Secondary School Bendemeer Secondary School Bendemeer Secondary School
h o Bendemeer Secondary School Bendemeer Secondary School Bendemeer Secondary School Bendemeer Secondary School Bendemeer Secondary School
cho
cch
Bendemeer
emeer Secondary
Bendemeer
Seeccon
S co
ond
on
emeer Secondary Schoo
daar
dar
5076/01
ary SSchool
choo Bendemeer Secondary School Bendemeer Secondary School Bendemeer Secondary School Bendemeer Secondary School Bendemeer Secondary School
School Bendemeer Secondary School Bendemeer Secondary School Bendemeer Secondary School Bendemeer Secondary School Bendemeer Secondary School
Bendemeer Secondary School Bendemeer Secondary School Bendemeer Secondary School Bendemeer Secondary School Bendemeer Secondary School Bendemeer Secondary School
There are forty questions on this paper. Answer all questions. For each question, there are four
possible answers A, B, C and D.
Choose the one you consider correct and record your choice in 2B pencil on the OTAS sheet.
Each correct answer will score one mark. A mark will not be deducted for a wrong answer.
Any rough working should be done on the question paper.
A copy of the Data Sheet is printed on page 20.
A copy of the Periodic Table is printed on page 21.
The use of an approved scientific calculator is expected, where appropriate.
40
This document consists of 21 printed pages.
[Turn over
www.KiasuExamPaper.com
213
10
[Turn over
www.KiasuExamPaper.com
214
11
22 Gases can diffuse through porous pots. The diagram shows a beaker full of nitrogen
inverted over a porous pot containing carbon monoxide.
[Turn over
www.KiasuExamPaper.com
215
12
What is X?
A aluminium
B beryllium
C boron
D fluorine
26 What happens when a bond is formed between a green gaseous element and a soft
metallic element?
[Turn over
www.KiasuExamPaper.com
216
13
A aluminium carbonate
B ammonium chloride
C calcium nitrate
D iron(II) sulfate
28 The oxide of element X dissolves in water to form a solution which when tested with
Universal Indicator paper gives a pH of 14. The oxide does not react with potassium
hydroxide. Where is X mostly likely to be found in the Periodic Table?
A Group I
B Group VI
C Group VII
D Group 0
29 25 cm3 of 0.1 mol / dm3 hydrochloric acid exactly neutralise 20 cm3 of aqueous sodium
hydroxide. The equation for this reaction is:
1D2++&Oĺ1D&O+ 2 O
[Turn over
www.KiasuExamPaper.com
217
14
30 W, X and Y are metals, one of which is copper and one of which is iron.
Which metal is the most reactive and what is the possible identity of W?
A The zinc atoms form strong covalent bonds with copper atoms.
B The zinc atoms have more electrons than the copper atoms.
C The zinc atoms prevent the ‘sea of electrons’ from moving freely in the solid.
D The zinc atoms prevent the layers of copper atoms from sliding over each other.
[Turn over
www.KiasuExamPaper.com
218
15
33 The labels on two bottles fell off. One bottle was known to contain sodium chloride
solution and the other bottle contained sodium nitrate solution.
[Turn over
www.KiasuExamPaper.com
219
16
Which row shows the colour changes when X is bubbled through these two solutions?
[Turn over
www.KiasuExamPaper.com
220
17
Which two diagrams show suitable methods for investigating the rate (speed) of the
reaction?
A 38 oC B 38.5 oC C 47 oC D 47.5 oC
[Turn over
www.KiasuExamPaper.com
221
18
38 A new planet has been discovered and its atmosphere has been analysed.
Which gases present in the atmosphere of the new planet are in a higher percentage
than they are in the Earth’s atmosphere?
X Y Z
A diesel oil lubricating fraction paraffin
B lubricating fraction diesel oil paraffin
C paraffin lubricating fraction diesel oil
D paraffin diesel oil lubricating fraction
[Turn over
www.KiasuExamPaper.com
222
19
[Turn over
www.KiasuExamPaper.com
223
20
Data Sheet
[Turn over
www.KiasuExamPaper.com
224
1
225
21
[Turn over
www.KiasuExamPaper.com
1
Register No. Class
Name
Write your name, class and register number on the work you hand in.
You may use a 2B pencil for any diagrams, graphs, tables or rough working.
Write in dark blue or black pen.
Do not use paper clips, glue or correction fluid.
Section A
Answer all questions.
Write your answers in the spaces provided on the question paper.
Section B
Answer any two questions.
Write your answers in the spaces provided on the question paper.
At the end of the examination, fasten all your work securely together.
The number of marks is given in brackets [ ] at the end of each question or part question.
65
This document consists of 15 printed pages.
[Turn over]
www.KiasuExamPaper.com
226
2
Section A
Answer all the questions in the spaces provided.
1 The structures of some substances containing chlorine are shown in Fig. 1.1.
Fig. 1.1
Answer the following questions about these substances. Each of these letters A to E can
be used once, more than once or not at all.
………………………………………………………………………………………….. [1]
………………………………………………………………………………………….. [1]
…………………………………………………………………………………………..
………………………………………………………………………………………….. [2]
………………………………………………………………………………………….. [1]
[Turn over]
www.KiasuExamPaper.com
227
3
Table 2.1
A 15 15 16
B 15 18 16
C 15 15 17
(a) What is the evidence in Table 2.1 for each of the following?
…………………………………………………………………………………………..
………………………………………………………………………………………….. [1]
…………………………………………………………………………………………..
………………………………………………………………………………………….. [1]
…………………………………………………………………………………………..
…………………………………………………………………………………………..
………………………………………………………………………………………….. [2]
………………………………………………………………………………………….. [1]
…………………………………………………………………………………………..
………………………………………………………………………………………….. [1]
[Turn over]
www.KiasuExamPaper.com
228
4
3 Coal-burning power stations generate a large amount of heat from the combustion of coal
to convert steam which in turn drives turbine generators to produce electricity. Flue gas that
is produced contains sulfur dioxide and oxides of nitrogen. These two gases cause acid
rain.
Explain how nitrogen monoxide causes acid rain even though it is a neutral
oxide.
………………………………………………………………………………………….
………………………………………………………………………………………….
…………………………………………………………………………………………. [2]
(b) Acid rain impacts farming greatly as it often causes the soil to be overly acidic
and results in leaching of nutrients. In order to alleviate the effects of acid rain,
a farmer has been advised to treat the soil to reduce the acidity.
Table 3.1
Using the information in Table 3.1, suggest why calcium carbonate is less
effective at reducing acidity than calcium hydroxide and calcium oxide.
…………………………………………………………………………………………..
…………………………………………………………………………………………..
………………………………………………………………………………………….. [2]
…………………………………………………………………………………………..
…………………………………………………………………………………………..
………………………………………………………………………………………….. [2]
[Turn over]
www.KiasuExamPaper.com
229
5
(d) Besides acid rain, name two other harmful effects of oxides of nitrogen and
sulfur dioxide.
…………………………………………………………………………………………..
………………………………………………………………………………………….. [2]
(a) Complete this method of preparing dry crystals of the soluble salt cobalt(II)
chloride from the insoluble base cobalt(II) carbonate.
step 1
Add an excess of cobalt(II) carbonate to hot dilute hydrochloric acid.
step 2
………………………………………………………………………………………….
step 3
………………………………………………………………………………………….
step 4
………………………………………………………………………………………….
step 5
…………………………………………………………………………………………. [3]
(i) Write a balanced chemical equation, including state symbols, for the above
reaction.
…………………………………………………………………………………………. [2]
[3]
[Turn over]
www.KiasuExamPaper.com
230
6
5 The reactivity of different metal oxides was compared by heating them with metals in a
crucible. This is shown in Fig. 5.1.
Fig. 5.1
Table 5.2
mixture observations
(a) Use the results in Table 5.2 to suggest the order of reactivity of the metals iron,
lead, magnesium and X, starting with the most reactive metal.
…………………………………………………………………………………………... [1]
…………………………………………………………………………………………... [1]
(c) Write down two observations when lead(II) oxide reacts with iron.
…………………………………………………………………………………………...
…………………………………………………………………………………………... [2]
Which element is reduced in the reaction? Use ideas about oxidation state to
explain your answer.
…………………………………………………………………………………………...
…………………………………………………………………………………………... [2]
[Turn over]
www.KiasuExamPaper.com
231
7
6 Fig. 6.1 shows how the ions present in solution A are separated.
Fig. 6.1
(a) (i) It is known that solution A contains one anion. Suggest the identity of this
anion. Give a reason for your answer.
………………………………………………………………………………………….
………………………………………………………………………………………….
…………………………………………………………………………………………. [2]
………………………………………………………………………………………….
…………………………………………………………………………………………. [1]
B ……………………………………………………..
C …………………………………………………….. [2]
[Turn over]
www.KiasuExamPaper.com
232
8
(c) Describe the movement and arrangement of particles in precipitate E which has
been dried.
………………………………………………………………………………………….
………………………………………………………………………………………….
…………………………………………………………………………………………. [2]
Fig. 7.1
………………………………………………………………………………………..... [1]
……………………………………………………………………………………….....
………………………………………………………………………………………..... [1]
………………………………………………………………………………………..... [1]
(c) Cyclohexane reacts with chlorine in the presence of ultraviolet light. This is a
substitution reaction. Write the molecular formulae of two products of this
reaction.
………………………………………………………………………………………..... [2]
[Turn over]
www.KiasuExamPaper.com
233
9
Section B
Answer any two questions in this section.
Write your answer in the spaces provided.
Fig. 8.1
The experiment is repeated to show that the reaction between zinc metal and
hydrochloric acid is catalysed by copper. A small volume of aqueous copper(II)
chloride is added to the acid before the zinc is added. The results of this
experiment are plotted on the same grid and labelled as graph 2 in Fig. 8.1.
(a) (i) Explain why the reaction mixture in the second experiment contains
copper metal. Include an equation in your explanation.
……………………………………………………………………………………….....
……………………………………………………………………………………….....
………………………………………………………………………………………..... [2]
[Turn over]
www.KiasuExamPaper.com
234
10
(ii) If the first experiment is repeated using ethanoic acid, CH 3 COOH, instead
of hydrochloric acid, explain how and why the graph would be different
from graph 1. Indicate the speed of this reaction on Fig. 8.1 and label it as
graph 3.
……………………………………………………………………………………….....
……………………………………………………………………………………….....
………………………………………………………………………………………..... [3]
(b) When lithium reacts with water, it moves about on the surface of the water.
Bubbles are seen and the lithium disappears slowly.
Predict how the reaction of potassium with water compares with the reaction of
lithium with water.
…………………………………………………………………………………………
…………………………………………………………………………………………
…………………………………………………………………………………………
…………………………………………………………………………………………
…………………………………………………………………………………………
………………………………………………………………………………………….. [5]
[Turn over]
www.KiasuExamPaper.com
235
11
9 In the laboratory, scientists are always doing research and conducting experiments to make
useful products for mankind.
(a) One such useful product is phosphine, PH 3 , which is used as a fumigant. It has
the smell of garlic and is effective in pest control.
…………………………………………………………………………………………..
…………………………………………………………………………………………..
…………………………………………………………………………………………..
…………………………………………………………………………………………..
…………………………………………………………………………………………..
………………………………………………………………………………………….. [5]
(ii) Draw the electronic structure of phosphine. Show outer electrons only.
[2]
(b) Scientists also make margarine from vegetable oils. List the conditions and
explain how vegetable oils are used to make margarine for use in foods.
………………………………………………………………………………………
………………………………………………………………………………………
………………………………………………………………………………………
………………………………………………………………………………………
……………………………………………………………………………………… [3]
[Turn over]
www.KiasuExamPaper.com
236
12
10 Thermal decomposition of compounds breaks them down into smaller substances when
sufficient heat is applied.
(a) Air bags are used to protect passengers in a car during an accident. When the
crash sensor detects an impact, it causes a mixture of chemicals to be heated
to a high temperature. Reactions take place which produce nitrogen gas. The
nitrogen fills the air bag. This is shown in Fig. 10.1.
Fig. 10.1
(i) Balance the chemical equation and complete the state symbols in the
chemical equation above. [2]
(ii) Draw the electronic structure of nitrogen gas. Show outer electrons only.
[2]
[Turn over]
www.KiasuExamPaper.com
237
13
(iii) An air bag consists of 130 g of sodium azide. When the sodium azide
decomposed, 60 dm3 of nitrogen was obtained at room temperature and
pressure.
[3]
(b) A student used the apparatus in Fig. 10.2 to investigate what happens when
liquid paraffin is heated to a high temperature.
Fig. 10.2
Liquid paraffin contains alkanes. The most abundant alkane has a chemical
formula of C 20 H 42.
Name the reaction shown in Fig. 10.2. Describe, with the aid of a chemical
equation, what happens to the alkane molecules in the reaction.
…………………………………………………………………………………………
…………………………………………………………………………………………
…………………………………………………………………………………………
………………………………………………………………………………………… [3]
END OF PAPER
[Turn over]
www.KiasuExamPaper.com
238
14
Data Sheet
[Turn over]
www.KiasuExamPaper.com
239
1
240
15
[Turn over]
www.KiasuExamPaper.com
241
www.KiasuExamPaper.com
Prelim Exam 2018
4E/5N Sc(Chem)
Marking Scheme
1 2 3 4 5 6 7 8 9 10
B D C D B A B A B D
11 12 13 14 15 16 17 18 19
9 20
D A B D C D B B D A
Different
Differen number
entt nu
numb
mber off ne
er o neutrons / different nucleon number / different mass
number
numb berr [[1]
1]
[2]
(b) (i) 2.8.5 / 2,8,5 [1]
(ii) non-metal because it accepts electrons / needs 3 electrons to
complete valence electron shell / because it is in Group V or 5 electrons
in valence shell [1]
Note: need both non-metal and reason for one mark [2]
www.KiasuExamPaper.com
242
3 (a) NO will be oxidised by oxygen in air to form nitrogen dioxide. [1]
Nitrogen dioxide will then dissolve in rainwater to form nitric acid which
caused acid rain. [1]
[2]
(b) Calcium carbonate is very much less soluble than calcium hydroxide
and calcium oxide. [1]
Thus, CaCO 3 reacts slowly with acid / effective only in reducing acidity
of soil / surface in contact / cannot penetrate soil to neutralize acid
deeper down. [1]
[2]
(c) The high temperatures of the car engines causes[1]
nitrogen in the air to react with oxygen in the air producing
produc
uccing oxides
ing oxid
ox off
des o
nitrogen. [1] [2]
(d) ¾ irritates the eyes and lungs and cause breathi
breathing
hiin
nggd difficulties
ififffiicu
ic
cu es [1]]
ulttie
¾ high levels lead to inflammation of the lung
lungs
ng (bronchitis)
gs ((b
brro
onc
nch hiittiis)
s) [1[1]]
[2]
4 (a) Step 2 Filter to remove excess cobalt(II) carbonate;
bonate; [1]
carb
Step 3 Heat the filtrate till saturation; [1]
form;
rm; [0.5]
Step 4 Cool to allow crystals to for
Step 5 Rinse crystals with a little
litittle distilled
tle dist
dis ille water
ed wa terr to remove impurities and
wate
dry between sheets of filter p
paper;
aper; [0 [0.5]
[0.5
.5]] [3]
(b) (i) CoCO 3 V +&O DT T ĺ&R&O
ĺ
ĺ &R
& &O 2 (a
(aq) + CO 2 (g) + H 2 O (l)
State symbols
syymb
mbols [1];; balanced
ols bal
alan
an ced chemical equation [1]
nce [2]
(ii) no o
off moles
mo of HCl
HCl = ccv
v = 2 * (40/1000) = 0.08 mol [1]
Mole
Mole ratio
rat
atio
io CoCO 3 : HCl
Fm eqn
eqn
q 1 : 2
m data
Fm data 0.04 : 0.08 [1]
www.KiasuExamPaper.com
243
5 (a) PDJQHVLXPĺ;ĺLURQĺOHDG Mg > X > Fe > Pb [1]
(b) no / it will not react and zinc is more reactive / iron is less reactive; [1]
ignore: zinc is reactive / iron is unreactive [1]
(c) A greenish ppt/solid [1]
and a grey/silver solid are formed. [1] [2]
((d)) Iron is reduced.[1]
[ ]
The oxidation state of iron decreases from +3 in iron(III) oxide to 0 in
iron. [1]
[2]
6 (a) (i) Nitrate [1]
All nitrates are soluble. [1] or
Sulfate [1]
All Ag+, Cu2+, Zn2+ and Fe3+ sulfates are sol
soluble.
olub
oluble.
le. [1
le [[1]
1] [2]
(ii) Add sodium hydroxide, aluminium
a uminium foil and warm. [0.5]
al
Gas produced turns moist red litmus paperr blue. [0.5] or
www.KiasuExamPaper.com
244
Section
ction C (20 marks)
8 (a) (i) zinc displaces copper / zinc more reactive than copper; [1]
Products:: lithium
litithi
hium
um hydroxide [0.5]
potassium hydroxide; [0.5]
hydrogen/H 2 [1] [5]
www.KiasuExamPaper.com
245
9 (a) (i) Phosphine is a liquid / gas at room condition [1]
It is made up of 2 non-metals [1] which will form a covalent compound
which is a liquid / gas at room conditions. [1] /
[1] for
f P, [1] for H
[2]
(b) react with
th hydrogen
hyd
ydro
rogen or
or hydrogenation
hyd
ydro
roge
g nation [1]
in the presence kel catalyst at 60 oC (allow 50-200 oC) [1]
e of a nickel
nic
icke
because
becaus e vegetable
use vege
ve geta
table oils
ble oils are unsaturated or have carbon-carbon double
bondds (v
bonds (veg
eget
etab
able
(vegetablele oils are hardened) to make them solid at room
[3]
temperature
temper
erat
atur e or to make them useful as spreads/spreadable [1]
ure
www.KiasuExamPaper.com
246
10 (a) (i) 2, 2, 3 [1]
(s), (s), (g) [1]
[2]
(ii)
Voll of N 2 = mol
Vo mol x 24
= 3 * 24 = 72 dm3 [1]
www.KiasuExamPaper.com
247
Prelim Exam 2018
4E/5N Sc(Chem)
Marking Scheme
www.KiasuExamPaper.com
3 (a) NO will be oxidised by oxygen in air to form nitrogen
dioxide. [1]
(d) lun
ngs and
¾ irritates the eyes and lungs nd ccause
ause breeat
athi
hing
n
ng
breathing
difficulties [1]
g levels lead to inflammation
¾ high infllam
amma
mati
t on of
ti of the
the lungs
lung
lu ngs
(bronchitis) [1]
[2]
4 (a) Step 2 Filter to remove excess
exc
xces cobalt(II)
esss cobalt ( I) carbonate;
lt(I
[1]
Step 3 Hea
eatt the fi
Heat filt
ltra
rate
te till
filtrate tilll sa
ssaturation;
turation; [1]
Step 4 Cool to allowow ccrystals
allllow rystals to form; [0.5]
ry
Stepp 5 Rinse
Sttep Rin
i see crystals
cryrysstals with a little distilled water to
remove
remo ve impurities
move imp
m ur
mp uritiies and dry between sheets of filter
paper;
papeer; [[0.5]
0 5]
0. [3]
www.KiasuExamPaper.com
(b) (i) CoCO 3 V +&O DT ĺ&R&O 2 (aq) + CO 2 (g)
+ H 2 O (l)
State symbols [1]; balanced chemical equation
[1] [2]
(ii) no of moles of HCl = cv = 2 * (40/1000) = 0.08
mol [1]
4.76 g of CoCO 3 ne
needed
eededd but
but 5.95 g was
was
used. Hence, CoCO 3 wa
was s in exc
excess.
xces
ess.
s
[3]
5 (a) PDJQHVLXPĺ;ĺLURQĺOHDG
PDJQHVLXPĺ;ĺLURQĺ OHD
HDG
G Mg > X >
Fe > Pb [1]
(b) no / it will not
ott rea
react
eact and
d zinc
z nc
zin iis
s mo
more reactive / iron is
less reaactivive
reactive; e; [1]
ignore: zinc is
is reactive
reac
reaccti
tive
ve / iron
iro
r n is unreactive [1]
(c) A greenish
gree
gr eeni
nish
sh p
ppt/solid
pt/s
pt /sol
olid
id [1
[[1]]
d a gr
and grey
e /s
sililve
grey/silverver solid are formed. [1] [2]
(d) Iron is reduced.[1]
redu
reduced.[1]
The oxidation state of iron decreases from +3 in
iron(III) oxide to 0 in iron. [1] [2]
www.KiasuExamPaper.com
6 (a) (i) Nitrate [1]
All nitrates are soluble. [1] or
Sulfate [1]
All Ag+, Cu2+, Zn2+ and Fe3+ sulfates are soluble.
[1] [2]
[2
(ii) Add sodium hydroxide, aluminium foil and warm.
war
arm.
m.
[0.5]
pape
paper blue.
Gas produced turns moist red litmus paper bllue
b ue.
[0.5] or
C: copper(II) hydroxide
hydroxxid
ide / Cu u(OH) 2 / iron(II)
Cu(OH) iron
ir on(I
(II)
I)
hydroxide / Fe(OH)H) 2 [1
[1]] [2]
(c) The particles are in solid
sol
olid
id sta
state.
tate
te.
They vibrate at th
hei
eirr fi
their fixe
xe
ed po
fixed posi
siti
tion
ons.
s. [1]
positions. [1]
They are closely packed
pac
acke
ked
d in a orderly
ord
r erly manner. [1] [2]
7 (a) (i)) Cont
Co
Contains
n ains
ns o
only
nly
nly ca
ccarbon-carbon
rbon
rbo -carbon single bonds [1]
(ii)
(i i) Co
Contains
ont
ntai
a ns o
only
nlly carbon and hydrogen atoms [1]
(b)) C 6 H 12 + 9O 2 ĺ
ĺ&2
&2 2 + 6H 2 O [1]
(c)
(c HCl
HCl (1
(1)) [2]
C 6 H 11 Cl (1)
www.KiasuExamPaper.com
Section C (20 marks)
8 (a) (i) zinc displaces copper / zinc more reactive than
copper; [1]
graph 1 [1]
[5]
www.KiasuExamPaper.com
(b) 3 marks from any 3 differences in observations e.g.
• more bubbles with K;
• it /K moves faster (on water surface);
• Li does not catch fire/K catches fire/K bursts into
flame;
• it /K fizzes more than Li;
• it /K disappears rapidly;
• K explodes / lithium does not explode;
• K melts / ball with K/ lithium does not melt/ does
not go into ball [3]
potassium hydrox
hydroxide;
oxide; [[0.5]
ide; 0 5]
0.
hydrogen/H 2 [1] [5]
www.KiasuExamPaper.com
9 (a)) (i) Phosphine is a liquid / gas at room condition [1]
It is made up of 2 non-metals [1] which will form
a covalent compound which is a liquid / gas at
room conditions. [1] /
[2]
www.KiasuExamPaper.com
(b) react with hydrogen or hydrogenation [1]
in the presence of a nickel catalyst at 60 oC (allow
50-200 oC) [1]
because vegetable oils are unsaturated or have
carbon-carbon double bonds (vegetable oils are e
hardened) to make them solid at room temperatu ture
tu
temperaturerre
e
[[3]
3]]
or to make them useful as spreads/spreadabable
ab
spreads/spreadable le
e [1]
[1]
1]
M ole
ol
Molee ratio
rati
ratio NaN 3 : N 2
Fm e eqn
qn 2 : 3
F data
Fm 2 : 3 [0.5]
Vol of N 2 = mol x 24
= 3 * 24 = 72 dm3 [1]
www.KiasuExamPaper.com
(b) Cracking [1]
www.KiasuExamPaper.com
257
www.KiasuExamPaper.com
Bukit Batok Secondary School
BE OUR BEST
PRELIMINARY EXAMINATIONS 2018
BUKIT BATOK
SECONDARY 4 EXPRESS / 5 NORMAL (ACADEMIC)
SECONDARY SCHOOL
There are forty questions on this paper. Answer all questions. For each question there are four possible
answers A, B, C and D.
Choose the one you consider correct and record your choice in soft pencil on the separate Answer Sheet.
Each correct answer will score one mark. A mark will not be deducted for a wrong answer.
Any rough working should be done in this booklet.
www.KiasuExamPaper.com
258
2018 / Preliminary Examinations / 4E & 5N(A) / Science (Physics) 5076 / Paper 1
A 5.272 mm
B 5.72 mm
C 7.22 mm
D 7.72 mm
distance / m
time / s
0
10
A An object at rest.
B An object moving with a constant speed of 2.0 m/s.
C An object moving with a constant velocity of 2.0 m/s.
D An object moving with a constant acceleration of 2.0 m/s2.
4 A particle moving at constant speed of 5 m/s is being acted on by two 10 N forces as shown.
5 m/s
10 N 10 N
2
Apply past knowledge to new situations
www.KiasuExamPaper.com
259
2018 / Preliminary Examinations / 4E & 5N(A) / Science (Physics) 5076 / Paper 1
The brick is now turned so that it rests on the table on its smallest face.
How has this affected the force and the pressure exerted by the brick on the table?
force pressure
A increased increased
B increased unchanged
C unchanged increased
D unchanged unchanged
6 When solid A of mass 15 g is immersed in a displacement can filled with water, it displaced the
same volume of water as solid B of mass 10 g. Which of the following best describes the
densities of solid A and solid B?
3
Apply past knowledge to new situations
www.KiasuExamPaper.com
260
2018 / Preliminary Examinations / 4E & 5N(A) / Science (Physics) 5076 / Paper 1
7 An empty egg shell has molten wax solidified inside it as shown in the figure below.
In which position is the egg shell most stable when placed on a flat, horizontal surface?
A B C D
Ignoring air resistance, what is the speed of the pendulum bob as it passes its lowest point B?
A 1.5 m/s
B 2.3 m/s
C 3.2 m/s
D 4.1 m/s
4
Apply past knowledge to new situations
www.KiasuExamPaper.com
261
2018 / Preliminary Examinations / 4E & 5N(A) / Science (Physics) 5076 / Paper 1
10 A beaker of water is heated at the bottom to form a convection current in the water.
An explanation on how convection occurs contains four statements.
plastic lid
transparent water
white colour level marking
body
5
Apply past knowledge to new situations
www.KiasuExamPaper.com
262
2018 / Preliminary Examinations / 4E & 5N(A) / Science (Physics) 5076 / Paper 1
12 A light ray is parallel to the floor and strikes a plane mirror as shown.
A 40°
B 50°
C 90°
D 140°
13 A rope is set to oscillate up-and-down to create a transverse wave that moves to the left.
At the particular instant below, what is the direction of movement of point P?
direction of wave
6
Apply past knowledge to new situations
www.KiasuExamPaper.com
263
2018 / Preliminary Examinations / 4E & 5N(A) / Science (Physics) 5076 / Paper 1
16 Which of the following diagrams correctly shows the direction of the resultant electrostatic force
acting on a small test charge?
+ – + – + + – –
A B C D
7
Apply past knowledge to new situations
www.KiasuExamPaper.com
264
2018 / Preliminary Examinations / 4E & 5N(A) / Science (Physics) 5076 / Paper 1
A fluorine
B lithium
C petroleum
D sugar
22 Which of the following gases can be prepared and collected using the apparatus shown?
A ammonia
B carbon dioxide
C hydrogen
D oxygen
8
Apply past knowledge to new situations
www.KiasuExamPaper.com
265
2018 / Preliminary Examinations / 4E & 5N(A) / Science (Physics) 5076 / Paper 1
24 Two students carried out chromatography experiments to examine the dyes in a black ink. They
used the same ink. The chromatograms obtained by the students are shown below.
Solvent front
9
Apply past knowledge to new situations
www.KiasuExamPaper.com
266
2018 / Preliminary Examinations / 4E & 5N(A) / Science (Physics) 5076 / Paper 1
25 The diagram shows the arrangement of electrons in the outer shells of the atoms in the
compound YZ 2 .
key
Y
electrons of Y atom
Z Z electrons of Z atom
Y Z
A calcium fluorine
B carbon sulfur
C oxygen hydrogen
D sulfur chlorine
26 Which ion has the same number of protons as the hydroxide ion?
A O2-
B F-
C Na+
D Mg2+
A carbon dioxide
B aluminium oxide
C zinc oxide
D copper (II) oxide
10
Apply past knowledge to new situations
www.KiasuExamPaper.com
267
2018 / Preliminary Examinations / 4E & 5N(A) / Science (Physics) 5076 / Paper 1
29 Solid R is gradually added to aqueous solution S. The changes in pH are shown in the graph
below.
14
pH 7
0
mass of solid R added
What are R and S?
R S
A insoluble metal oxide hydrochloric acid
B insoluble non-metal oxide sodium hydroxide
C soluble metal oxide hydrochloric acid
D soluble non-metal oxide sodium hydroxide
30 Test on a sample of polluted water from a factory gives the following results.
Reagent Result
Hydrochloric acid and aqueous barium White precipitate
chloride
A 1 dm3
B 6 dm3
C 12 dm3
D 24 dm3
11
Apply past knowledge to new situations
www.KiasuExamPaper.com
268
2018 / Preliminary Examinations / 4E & 5N(A) / Science (Physics) 5076 / Paper 1
The apparatus below was set up with a 100 cm3 sample of the atmosphere of the
planet in the graduated tube. The volume of the sample was measured at intervals
until no further change in volume took place.
A 30 cm3
B 40 cm3
C 60 cm3
D 70 cm3
12
Apply past knowledge to new situations
www.KiasuExamPaper.com
269
2018 / Preliminary Examinations / 4E & 5N(A) / Science (Physics) 5076 / Paper 1
35 Which of the following statement described the conversion of a sodium atom, Na, to a sodium
ion, Na+?
36 Some crystals of magnesium carbonate were added to an excess of sulfuric acid at room
temperature. The volume of carbon dioxide gas produced was measured over a period of time.
The results are shown in graph Y.
A Acid of the same volume and half the original concentration was used.
B Half the mass of magnesium carbonate was used.
C Larger crystals of magnesium carbonate was used.
D Using a lower temperature.
37 The table below shows the boiling point ranges of fractions collected from the distillation of a
sample of crude oil.
13
Apply past knowledge to new situations
www.KiasuExamPaper.com
270
2018 / Preliminary Examinations / 4E & 5N(A) / Science (Physics) 5076 / Paper 1
A C 8 H 16 O 4 + 2O 2 8C + 8H 2 O
B C 8 H 16 O 4 + 5O 2 8CO + 8H 2 O
C C 8 H 16 O 4 + 10O 2 8CO 2 + 8H 2 O
D C 8 H 16 O 4 + 8O 2 4CO 2 + 4CO + 8H 2 O
A combustion of methane
B fermentation of sugar
C oxidation of ethanol to ethanoic acid
D reaction of ethanoic acid with calcium carbonate
- End of Paper 1 -
14
Apply past knowledge to new situations
www.KiasuExamPaper.com
271
2018 / Preliminary Examinations / 4E & 5N(A) / Science (Physics) 5076 / Paper 1
DATA SHEET
15
Apply past knowledge to new situations
www.KiasuExamPaper.com
272
DATA SHEET
The Periodic Table of Elements
Group
I II III IV V VI VII 0
1 2
H He
Hydrogen Helium
Key 1 4
3 4 proton (atomic) number 5 6 7 8 9 10
Li Be atomic symbol B C N O F Ne
Lithium Beryllium name Boron Carbon Nitrogen Oxygen Fluorine Neon
23 24 27 28 31 32 35.5 40
19 20 21 22 23 24 25 26 27 28 29 30 31 32 33 34 35 36
K Ca Sc Ti V Cr Mn Fe Co Ni Cu Zn Ga Ge As Se Br Kr
Potassium Calcium Scandium Titanium Vanadium Chromium Manganese Iron Cobalt Nickel Copper Zinc Gallium Germanium Arsenic Selenium Bromine Krypton
39 40 45 48 51 52 55 56 59 59 64 65 70 73 75 79 80 84
37 38 39 40 41 42 43 44 45 46 47 48 49 50 51 52 53 54
Rb Sr Y Zr Nb Mo Tc Ru Rh Pd Ag Cd In Sn Sb Te I Xe
Rubidium Strontium Yttrium Zirconium Niobium Molybdenum Technetium Ruthenium Rhodium Palladium Silver Cadmium Indium Tin Antimony Tellurium Iodine Xenon
273
85 88 89 91 93 96 - 101 103 106 108 112 115 119 122 128 127 131
55 56 57 – 71 72 73 74 75 76 77 78 79 80 81 82 83 84 85 86
Cs Ba lanthanoids Hf Ta W Re Os Ir Pt Au Hg Tl Pb Bi Po At Rn
Caesium Barium Hafnium Tantalum Tungsten Rhenium Osmium Iridium Platinum Gold Mercury Thallium Lead Bismuth Polonium Astatine Radon
133 137 178 181 184 186 190 192 195 197 201 204 207 209
87 88 89 – 103 104 105 106 107 108 109 110 111 112 114 116
Fr Ra actinoids Rf Db Sg Bh Hs Mt Ds Rg Cn Fl Lv
Francium Radium Rutherfordium Dubnium Seaborgium Bohrium Hassium Meitnerium Darmstadtium Roentgenium Copernicium Flerovium Livermorium
57 58 59 60 61 62 63 64 65 66 67 68 69 70 71
lanthanoids La Ce Pr Nd Pm Sm Eu Gd Tb Dy Ho Er Tm Yb Lu
Lanthanum Cerium Praseodymium Neodymium Promethium Samarium Europium Gadolinium Terbium Dysprosium Holmium Erbium Thulium Ytterbium Lutetium
139 140 141 144 147 150 152 157 159 162 165 167 169 173 175
89 90 91 92 93 94 95 96 97 98 99 100 101 102 103
actinoids Ac Th Pa U Np Pu Am Cm Bk Cf Es Fm Md No Lr
Actinium Thorium Protactinium Uranium Neptunium Plutonium Americium Curium Berkelium Californium Einsteinium Fermium Mendelevium Nobelium Lawrencium
16
www.KiasuExamPaper.com
Name:………………………………………………………………………….…. Class register no…………. Class:…………….
Section A
Answer all questions in the spaces provided.
Write your answers in the spaces provided on the Question Paper.
Section B
Answer any two questions.
Write your answers in the spaces provided on the Question Paper.
At the end of the examination, fasten all your work securely together.
The number of marks is given in brackets [ ] at the end of each question or part question.
………….
................
Total
www.KiasuExamPaper.com
274
2018 / Sec 4E5N(A) / 5076 Science (Physics) / Preliminary Exam Paper 2
1 Fig.1.1 below shows the velocity-time graph of a 900 kg car travelling on a straight horizontal
road for the first sixty seconds of its journey.
v/ms-1
t/s
Fig 1.1
(a) Calculate, for the first 10 seconds,
(i) the acceleration of the car,
www.KiasuExamPaper.com
275
2018 / Sec 4E5N(A) / 5076 Science (Physics) / Preliminary Exam Paper 2
(b) State how the braking force relates to the forward driving force,
(i) from t = 35 s to t = 45 s and
...........................................................................................................................................
...........................................................................................................................................
(ii) between t = 45 s to t = 60 s.
...........................................................................................................................................
...................................................................................................................................... [2]
(c) Determine the total distance travelled by the car during the first 30 s of its journey.
www.KiasuExamPaper.com
276
2018 / Sec 4E5N(A) / 5076 Science (Physics) / Preliminary Exam Paper 2
2 Fig. 2.1 shows a stone supported by two strings that hang from a rod. The tensions in the two
strings are 1.3 N and 2.0 N.
scale = 1 cm to …………………….. N
www.KiasuExamPaper.com
277
2018 / Sec 4E5N(A) / 5076 Science (Physics) / Preliminary Exam Paper 2
3 Fig 3.1 shows a barrier found in most carparks. The barrier is in equilibrium. The weight of the
pole is 450 N and the centre of gravity of the pole is 1.30 m away from the pivot.
movable
counterweight pivot
pole
0.30 m 1.30 m
450 N
Fig. 3.1
(a) The centre of gravity of the movable counterweight is 0.30 m away from the pivot.
(i) Calculate the weight of the counterweight.
……..........................................................................................................................................
……..........................................................................................................................................
……..........................................................................................................................................
……..................................................................................................................................... [2]
www.KiasuExamPaper.com
278
2018 / Sec 4E5N(A) / 5076 Science (Physics) / Preliminary Exam Paper 2
Fig 4.1
(a) State the Principle of Conservation of Energy.
.................................................................................................................................................
.................................................................................................................................................
.................................................................................................................................................
............................................................................................................................................ [2]
(b) State and explain at which point on the track would the speed of the toy car be at its
maximum.
.................................................................................................................................................
.................................................................................................................................................
.................................................................................................................................................
............................................................................................................................................ [2]
www.KiasuExamPaper.com
279
2018 / Sec 4E5N(A) / 5076 Science (Physics) / Preliminary Exam Paper 2
(c) (i) Calculate the gravitational potential energy of the toy car at point C.
www.KiasuExamPaper.com
280
2018 / Sec 4E5N(A) / 5076 Science (Physics) / Preliminary Exam Paper 2
5 (a) Fig. 5.1 shows how the temperature of an unknown solid substance varies when heated
over a Bunsen flame.
Fig. 5.1
...................................................................................................................................... [1]
...........................................................................................................................................
...................................................................................................................................... [1]
...........................................................................................................................................
...................................................................................................................................... [1]
(b) Explain why a pot of water boils faster with a covered lid.
……..........................................................................................................................................
……..........................................................................................................................................
……..........................................................................................................................................
……..................................................................................................................................... [2]
www.KiasuExamPaper.com
281
2018 / Sec 4E5N(A) / 5076 Science (Physics) / Preliminary Exam Paper 2
6 Fig. 6.1 shows a rectangular glass block, PQRS, with a refractive index of 1.50. A light ray is
incident on the side PS of the glass block as shown.
……..........................................................................................................................................
……..................................................................................................................................... [1]
(b) A ray of light is incident on the side PS at 75° and is refracted into the glass block at 40°.
Fig. 5.1
(i) Show that the refractive index of the glass block is 1.50. [1]
(iii) Complete the path of the light ray until it emerges into the air again.
Label all the angles clearly. [2]
www.KiasuExamPaper.com
282
2018 / Sec 4E5N(A) / 5076 Science (Physics) / Preliminary Exam Paper 2
7 (a) A beam of light is travelling parallel to the axis of a thin lens, as shown in Fig. 7.1.
Point F is the focal point of the lens.
Fig. 7.1
On Fig. 7.1, complete the paths of the two rays after passing through the lens. [2]
(b) In this part of the question, you are required to draw an accurate ray diagram using the grid
on Fig. 7.2 for the lens in part (a).
Fig. 7.2
(i) On the diagram, draw two rays from the top of the object through the lens to locate the
position of the image. Label the image “I” beside it. [2]
(ii) From your diagram, state one similarity about the image and the object.
...........................................................................................................................................
...................................................................................................................................... [1]
www.KiasuExamPaper.com
283
2018 / Sec 4E5N(A) / 5076 Science (Physics) / Preliminary Exam Paper 2
8 Fig. 8.1 shows an arrangement that is used to remove dust particles from the smoke in a
factory chimney.
Fig. 8.1
When smoke passes through the metal grid, this results in the dust particles having a net
positive charge.
(a) Explain what happened to the dust particles as they pass through the metal grid.
.................................................................................................................................................
.................................................................................................................................................
.................................................................................................................................................
............................................................................................................................................ [2]
(b) If 6.0 C of charges flow past point X in 1.0 minute, calculate the current flowing through X.
www.KiasuExamPaper.com
284
2018 / Sec 4E5N(A) / 5076 Science (Physics) / Preliminary Exam Paper 2
9 Fig. 9.1 shows a metal cylinder which contains 0.000 46 m3 of oil. The total mass of the
cylinder and the oil is 1.2 kg. The mass of the cylinder is 0.800 kg and the space above the oil
is air. The gravitational field strength is 10 N/kg.
Fig. 9.1
(a) State two differences between the mass and the weight of a substance.
Difference 1: .........................................................................................................................
.................................................................................................................................................
Difference 2: .........................................................................................................................
.................................................................................................................................................
[2]
www.KiasuExamPaper.com
285
2018 / Sec 4E5N(A) / 5076 Science (Physics) / Preliminary Exam Paper 2
(d) Calculate the pressure exerted by the oil on the base of the cylinder in SI unit.
(e) State and explain how the values of each of the following quantities would change when
the cylinder and the oil is brought to the Moon, where the gravitational field strength is
reduced.
...........................................................................................................................................
...................................................................................................................................... [1]
...........................................................................................................................................
...........................................................................................................................................
...................................................................................................................................... [2]
www.KiasuExamPaper.com
286
2018 / Sec 4E5N(A) / 5076 Science (Physics) / Preliminary Exam Paper 2
10 (a) A boy holds the loose end of a long rope which is fixed to a pole.
He moves it up and down at a rate of 20 complete oscillations in every 50 seconds.
Fig. 10.1 shows a section of the wave moving along the rope.
Fig. 10.1
www.KiasuExamPaper.com
287
2018 / Sec 4E5N(A) / 5076 Science (Physics) / Preliminary Exam Paper 2
(b) Fig. 10.2 shows the cone of a loudspeaker that is producing sound waves in air.
At any given moment, a series of compressions and rarefactions exist along the line PQ.
cone
P Q
air
Fig. 10.2
(i) On Fig 10.2, draw the wave lines to represent two wavelengths of compressions and
rarefactions between P and Q.
Use the letter C to mark two compressions and the letter R to mark two rarefactions
along PQ. [2]
(ii) With reference to the sound wave travelling along PQ in Fig. 10.2, explain what is
meant by a longitudinal wave.
...........................................................................................................................................
...................................................................................................................................... [1]
(iii) To the right of Q, there is a large vertical wall 50 m in front of the loudspeaker.
The speed of sound in air is 340 m/s.
Calculate the time taken for the echo to return to P.
www.KiasuExamPaper.com
288
2018 / Sec 4E5N(A) / 5076 Science (Physics) / Preliminary Exam Paper 2
11 (a) Fig. 11.1 below shows an electrical circuit with a lamp and two resistors
Fig. 11.1
(i) Name the component X.
...................................................................................................................................... [1]
Fig. 11.2 shows the graph of current against potential difference for the lamp alone.
The potential difference across the lamp is 4.0 V.
Fig. 11.2
Determine
(ii) the current in the lamp, I L .
I L = ………………………… [1]
I R = ………………………… [2]
www.KiasuExamPaper.com
289
2018 / Sec 4E5N(A) / 5076 Science (Physics) / Preliminary Exam Paper 2
Fig. 11.3
When switch S is closed, the iron bolt moves to the right and out of the striker plate,
allowing the door to be opened.
(i) Explain why the iron bolt moves to the right and into the solenoid when the switch is
closed.
...........................................................................................................................................
...........................................................................................................................................
...........................................................................................................................................
...........................................................................................................................................
...........................................................................................................................................
...................................................................................................................................... [3]
(ii) After using the electric lock for a year, the 6 V battery goes “flat” and its e.m.f. drops to
4 V.
Why the electromagnetic lock does not work now?
...........................................................................................................................................
...........................................................................................................................................
...........................................................................................................................................
...................................................................................................................................... [2]
www.KiasuExamPaper.com
290
2018 / Sec 4E5N(A) / 5076 Science (Physics) / Preliminary Exam Paper 2
୴ି୳ ଶି
1 (a) (i) acceleration, a = = [1 for working]
୲ ଵ
= 2.0 m/s2 [1 for ans]
(ii) Between t = 45 s to t = 60 s, the braking force is equal to the forward driving force.
[1]
www.KiasuExamPaper.com
291
2018 / Sec 4E5N(A) / 5076 Science (Physics) / Preliminary Exam Paper 2
(ii) Using W = mg
m =
ଵଽହ
=
ଵ
= 195 kg [1 for working & ans]
(c) The gate can be opened by shifting the counterweight further away from the pivot.
(OR shift to the left). [1]
so the counterclockwise moment produced by the force of the counterweight is larger
than the clockwise moment produced by the weight of the pole. [1]
www.KiasuExamPaper.com
292
2018 / Sec 4E5N(A) / 5076 Science (Physics) / Preliminary Exam Paper 2
(ii) During stage C, the molecules are sliding past one another. [1]
(b) With a covered lid, it reduces heat loss to the surroundings by:
o prevents convection current from forming.
o reducing evaporation from the water surface,
o reducing conduction through the layer of trapped air between the water and the lid.
[any TWO answers, 1 mark each, max = 2]
ଵ
(ii) n =
ୱ୧୬
ଵ
1.50 = [1 for working]
ୱ୧୬
1
c = sin–1 ቀ ቁ
1.50
= 41.8 ° [1 for ans]
(b) (iii) [1 for Total Internally Reflected ray, angle 50°, at side PQ]
[1 for emergent ray at side QR, angle = 75°]
7 (a) [1 mark for each ray converging onto F on the right after passing through the lens]
[max = 2 marks]
www.KiasuExamPaper.com
293
2018 / Sec 4E5N(A) / 5076 Science (Physics) / Preliminary Exam Paper 2
image
8 (a) When the dust particles come into contact with the grid, they lose electrons to the grid
[1]. Thus the particles will have less negative charges than positive charges and end up
with a net positive charge and this cause it to be attracted to the metal plate. [1]
(b) Using I = Q / t
= 6.0 / (1 x 60) = 0.10 A [1 for working & ans]
www.KiasuExamPaper.com
294
2018 / Sec 4E5N(A) / 5076 Science (Physics) / Preliminary Exam Paper 2
SECTION B [2 X 10 = 20 MARKS]
(b) Using W = mg
= (1.2 – 0.800)(10)
= 4.0 N [1 for working, ans & unit]
୫ .ସ
(c) Using density = = [1 for working]
.ସ
= 870 kg/m3 [1 for ans & unit]
ସ.
(d) Using P = = బ.బబబరల [1 for working]
ቀ ቁ
బ.మబ
(e) (i) Density does not change. Since Density = Mass / Volume,
both the mass and volume of the liquid does not change. [1]
(iii) Using v = f O
v = 0.40 x 60 [1 for working]
= 24 cm /s or 0.24 m/s [1 for ans & unit]
[allow ecf from (a)(ii) for value of frequency]
www.KiasuExamPaper.com
295
2018 / Sec 4E5N(A) / 5076 Science (Physics) / Preliminary Exam Paper 2
cone C C
R R
P Q
air
Fig. 10.1
ଶ × ୢ୧ୱ୲ୟ୬ୡୣ 2 ×50
(iv) Time = =൬ [1 for working]
ୱ୮ୣୣୢ 340 ൰
(ii) (From the graph) current I L = 0.060 A [1 for ans & unit]
(iii) Using V = IR
ସ.
current I R = = [1 for working]
ୖ ଵ
= 0.040 A [1 for ans & unit]
(b) (i) When the switch is closed, the solenoid becomes an electromagnet. [1]
The iron bolt is then attracted to the solenoid due to magnetic induction. [1]
This strong attractive force will overcome the force of the spring and cause the
lock to be unlocked. [1]
www.KiasuExamPaper.com
296
Name: ………………………………………. Index No. ……… Class: ……………
At the end of the examination, fasten all your work securely together.
The number of marks is given in brackets [ ] at the end of each question or part question.
Total
www.KiasuExamPaper.com
297
2018 Prelim/Sec4E5N/Science (Chemistry) 5076 P3
Fig. 1.1
(b) Name two gases that pollute the atmosphere and name the chemical source of
each.
Gas 1 …………….…………………………………………………………………………
Source ….……………………………………………………………………………………
….……………………………………………………………………………………
Gas 2 …..…………………………………………………………………………………...
Source ….……………………………………………………………………………………
…………………………………………………………………………………… [4]
(a) State a method that can separate the clay from the rest of the water.
……………………………………………………………………………………………… [1]
(b) The student then boiled the river water to obtain the soluble solids.
Fig. 2.1 shows how she heated the water.
Fig. 2.1
The student wrote in her practical sheet that “the boiled river water is pure because
the universal indicator remains green when it is added to the boiled river water.”
………………………………………………………………………………………………..
…………………………………………………………………………………………….. [1]
(c) On cooling, steam will condense. Describe what happens to the spacing and
movement of the particles of steam during condensation.
changes to spacing
………………………………………………………………………………………………..
changes to movement
…………………………………………………………………………………………….. [2]
3 Cold packs are used to reduce swelling, inflammation and pain by removing the heat.
A list of chemicals from a science laboratory is shown below:
ammonium nitrate, dilute hydrochloric acid, water,
anhydrous sodium carbonate, sodium hydroxide
(a) From the list of chemicals provided above, select a pair of chemicals that can be
used for making a cold pack in a science laboratory.
…………………………………………………………………………………………….. [1]
(b) Explain your answer in (a) and state the type of energy change.
………………………………………………………………………………………………....
…………………………………………………………………………………………….. [2]
(b) Magnesium carbonate reacts with this solution of nitric acid as follows:
MgCO 3 + 2HNO 3 Æ Mg(NO 3 ) 2 + H 2 O + CO 2
(i) What mass of magnesium carbonate react with 500 cm3 of nitric acid?
(ii) Find the volume of carbon dioxide gas produced in this reaction.
(c) Another nitric acid solution is made by diluting 1.0 mol to make 2.0 dm3 of solution.
What is the concentration of this solution in mol/dm3?
Properties X Y Iodine
melting point / oC - 7.2 - 101.0 114.0
boiling point /
oC 58.8 - 35.0 184.0
reaction with
colourless solution colourless solution
aqueous potassium
turns brown turns brown
iodide
reacts quickly and reacts rapidly and
reaction with cold reacts slowly to
less vigorously to vigorously to form
aqueous sodium form a colourless
form a colourless a colourless
hydroxide solution
solution solution
Fig. 5.1
(a) State the physical state and colour of Y at room temperature and pressure.
……………………………………………………………………………………………… [1]
(b) Using evidence from Figure 5.1, explain and deduce the identity of X.
………………………………………………………………………………………………….
………………………………………………………………………………………………….
………………………………………………………………………………………………….
………………………………………………………………………………………………….
………………………………………………………………………………………………….
……………………………………………………………………………………………… [3]
(c) Iodine reacts with cold aqueous sodium hydroxide according to the equation:
2NaOH + I 2 Æ NaI + NaOI + H 2 O
(i) Explain why both element X and Y undergoes similar reaction with cold
aqueous sodium hydroxide.
…………………………………………………………………………………………….
………………………………………………………………………………………… [1]
(ii) Hence, construct a chemical equation for the reaction between element X
and cold aqueous sodium hydroxide.
………………………………………………………………………………………… [1]
6 Equal masses of lumps of lead (II) carbonate were reacted with three different acids of
the same concentration in three separate experiments I, II and III. The acids were in
excess and all other conditions were kept the same.
Experiment Reagents
I Lead (II) carbonate Nitric acid
II Lead (II) carbonate Sulfuric acid
III Lead (II) carbonate Ethanoic acid
The mass of the lead (II) carbonate was measured and calculated at regular time intervals
and the results for experiments I and II are shown in Figure 6.1.
Mass of
PbCO3 / g
Experiment II
Experiment I
Time / min
Fig. 6.1
(a) Using Fig. 6.1, determine if lead (II) carbonate react completely with sulfuric acid.
Explain your answer.
……………………………………………………………………………………………… [1]
(b) (i) In experiments I and III, would ethanoic acid react faster than nitric acid?
Explain your answer in terms of hydrogen ions in the acids.
…………………………………………………………………………………………..
…………………………………………………………………………………………..
…………………………………………………………………………………………..
………………………………………………………………………………………… [2]
(ii) Lead (II) ethanoate is a white, crystalline substance with a sweetish taste and is
soluble in water.
Sketch on the same axes above, the result for experiment III. [1]
(c) Briefly outline how a pure and dry sample of lead (II) nitrate can be made from
lead (II) carbonate.
………………………………………………………………………………………………….
………………………………………………………………………………………………….
………………………………………………………………………………………………….
………………………………………………………………………………………………….
………………………………………………………………………………………………….
………………………………………………………………………………………………….
……………………………………………………………………………………………… [4]
………………………………………………………………………………………………….
………………………………………………………………………………………………….
………………………………………………………………………………………………….
………………………………………………………………………………………………….
………………………………………………………………………………………………….
………………………………………………………………………………………………….
………………………………………………………………………………………………….
………………………………………………………………………………………………….
………………………………………………………………………………………………….
.……………………………………………………………………………………………... [5]
……………………………………………………………………………………………... [1]
8 A mixture Z was made by dissolving two salts, X and Y, in water. A series of reactions
was carried out on mixture Z as shown below.
add excess
dilute nitric acid limewater white
Mixture Z Gas A
precipitate
dirty green
precipitate
exposed to air
brown
precipitate
(a) Identify the four ions that are present in mixture Z. Justify your answers. [4]
(b) Predict what would be observed if excess ammonia solution was added to a sample
of mixture Z.
………………………………………………………………………………………………….
……………………………………………………………………………………………... [1]
(c) No reaction was observed when ammonia solution was added to aqueous solution of
salt Y.
X : ……………………………………………………………………………………………...
Y : …………………………………………………………………………………………. [2]
- End of Section A -
9 Fig. 9.1 shows the structures of calcium chloride and chlorine gas.
………………………………………………………………………………………………….
………………………………………………………………………………………………….
……………………………………………………………………………………………... [3]
Fig. 9.2
…………………………………………………………………………………………….
…………………………………………………………………………………………….
……………………………………………………………………………………….... [2]
(ii) Explain how a calcium atom combines with chlorine atoms to form calcium
chloride. Your answer should include:
x electronic structures of atoms
x force of attraction in the compound
……………………………………………………………………………………………
……………………………………………………………………………………………
…………………………………………………………………………………………….
……………………………………………………………………………………….... [3]
(iii) Give a reason why chlorine gas has such a low boiling point.
……………………………………………………………………………………………
…………………………………………………………………………………………….
……………………………………………………………………………………….... [2]
[Total : 10 marks]
Fig. 10.1 shows how the strength of duralumin changes with the different
percentage of copper added.
Strength of alloy
Strength of alloy
Percentage
Percentage of of copper
copper
11 22 3 4 55 66 77 88
Fig. 10.1
(i) Using the information from Figure 10.1, estimate the percentage of copper that
will produce the strongest duralumin mixture.
………………………………………………………………………………………… [1]
(ii) Explain, with the aid of a well-labelled diagram of duralumin, why it is stronger
than pure aluminium.
…………………………………………………………………………………………….
…………………………………………………………………………………………….
………………………………………………………………………………………… [2]
(b) Fig. 10.2 shows the results of an experiment in which four metals are placed
in solutions of other metal nitrates.
Fig. 10.2
(i) Arrange the four metals in order of their reactivity starting with the most reactive.
………………………………………………………………………………………… [1]
(ii) Predict two observations when chromium is added to copper(II) nitrate solution.
…………………………………………………………………………………………….
…………………………………………………………………………………………….
………………………………………………………………………………………… [2]
(c) One possible chemical reaction between metal Y and the solution of salt X is as
follows:
Y + X(NO 3 ) 2 Æ X + Y(NO 3 ) 2
Y(NO 3 ) 2
[2]
(ii) Using the data above, explain whether Y is an oxidizing or a reducing agent.
…………………………………………………………………………………………….
…………………………………………………………………………………………….
…………………………………………………………………………………………….
………………………………………………………………………………………… [2]
[Total : 10 marks]
[1]
(ii) Explain why and how the monomer drawn in (a)(i) can undergo addition
polymerisation to form poly(methyl methacrylate).
…………………………………………………………………………………………….
…………………………………………………………………………………………….
…………………………………………………………………………………………….
………………………………………………………………………………………… [2]
(b) Ethanol can be obtained from glucose through the process of fermentation. One of
the conditions of this process is having an anaerobic environment. Otherwise,
substance X will be produced instead.
………………………………………………………………………………………… [1]
[1]
…………………………………………………………………………………………….
…………………………………………………………………………………………….
…………………………………………………………………………………………….
…………………………………………………………………………………………….
………………………………………………………………………………………… [3]
…………………………………………………………………………………………….
………………………………………………………………………………………… [1]
[Total : 10 marks]
- End of Paper –
DATA SHEET
7 9 11 12 14 16 19 20
11 12 13 14 15 16 17 18
Na Mg Al Si P S Cl Ar
Sodium Magnesium Aluminium Silicon Phosphorus Sulfur Chlorine Argon
23 24 27 28 31 32 35.5 40
19 20 21 22 23 24 25 26 27 28 29 30 31 32 33 34 35 36
K Ca Sc Ti V Cr Mn Fe Co Ni Cu Zn Ga Ge As Se Br Kr
Potassium Calcium Scandium Titanium Vanadium Chromium Manganese Iron Cobalt Nickel Copper Zinc Gallium Germanium Arsenic Selenium Bromine Krypton
39 40 45 48 51 52 55 56 59 59 64 65 70 73 75 79 80 84
37 38 39 40 41 42 43 44 45 46 47 48 49 50 51 52 53 54
Rb Sr Y Zr Nb Mo Tc Ru Rh Pd Ag Cd In Sn Sb Te I Xe
Rubidium Strontium Yttrium Zirconium Niobium Molybdenum Technetium Ruthenium Rhodium Palladium Silver Cadmium Indium Tin Antimony Tellurium Iodine Xenon
315
85 88 89 91 93 96 - 101 103 106 108 112 115 119 122 128 127 131
55 56 57 – 71 72 73 74 75 76 77 78 79 80 81 82 83 84 85 86
Cs Ba lanthanoids Hf Ta W Re Os Ir Pt Au Hg Tl Pb Bi Po At Rn
Caesium Barium Hafnium Tantalum Tungsten Rhenium Osmium Iridium Platinum Gold Mercury Thallium Lead Bismuth Polonium Astatine Radon
133 137 178 181 184 186 190 192 195 197 201 204 207 209
87 88 89 – 103 104 105 106 107 108 109 110 111 112 114 116
Fr Ra actinoids Rf Db Sg Bh Hs Mt Ds Rg Cn Fl Lv
Francium Radium Rutherfordium Dubnium Seaborgium Bohrium Hassium Meitnerium Darmstadtium Roentgenium Copernicium Flerovium Livermorium
57 58 59 60 61 62 63 64 65 66 67 68 69 70 71
lanthanoids La Ce Pr Nd Pm Sm Eu Gd Tb Dy Ho Er Tm Yb Lu
Lanthanum Cerium Praseodymium Neodymium Promethium Samarium Europium Gadolinium Terbium Dysprosium Holmium Erbium Thulium Ytterbium Lutetium
139 140 141 144 147 150 152 157 159 162 165 167 169 173 175
89 90 91 92 93 94 95 96 97 98 99 100 101 102 103
actinoids Ac Th Pa U Np Pu Am Cm Bk Cf Es Fm Md No Lr
Actinium Thorium Protactinium Uranium Neptunium Plutonium Americium Curium Berkelium Californium Einsteinium Fermium Mendelevium Nobelium Lawrencium
The volume of one mole of any gas is 24 dm3 at room temperature and pressure (r.t.p.).
19
www.KiasuExamPaper.com
316
www.KiasuExamPaper.com
Bukit Batok Secondary School
Sec 4 Express, 5 Normal (Academic)
Science (Chemistry)
Answer
A nswe
ns werr is NOT (B) because LIME is used to remove acidic impurities, not
32 D
limestone.
lime
me
est
stone. LIME is produced of heat limestone.
CaCO 3 Æ CaO + CO 2
CaO + SiO 2 Æ CaSiO 3
33 D 30% of O 2 will be used up, 70% of air left.
(A) formation of bond = exothermic
(B) rusting = exothermic
34 C
(C) melting (ice absorb heat) = endothermic
(D) H 2 combust in O 2 to form H 2 O = exothermic
Na atom (2.8.1) loses 1 electron to form Na+ ion (2.8).
35 D
Loss of electrons = oxidation
36 B ½ volume of CO 2 produced , thus ½ mass of limiting reactant (MgCO 3 ) used.
1
www.KiasuExamPaper.com
317
37 A Smallest molecules = lowest boiling point range
38 C Complete combustion produce carbon dioxide and water.
39 C Oxidation of ethanol to ethanoic acid produces water.
40 B It has C=C and carboxyl group (COOH).
1a. Nitrogen 1m
2a. filtration 1m
2b. No. n
noo mark
The purity of the liquid is determ
rmined by the fixed bo
determined oiling point.
boiling
OR
The universal indicator remains green can only prove tha hat the liquid
that id
d has
has
a neutral pH (pH 7).
OR
The universal indicator can only
onl
nlyy prove
prov
ove whether
w ethe
wh er the
the solution
solu
so lutition
o is acidic or
alkaline, but cannot show whether th tthe
ew ate
er is p
water ure.
ur
pure.e. either 1m
Changes to o movement
move
mo veme
m nt
The movement
mo
ove
vementnt sl
slows do wn, fr
down,
down from moving
om moving randomly at high speeds to
sliding pa
past o
one
ne ano
another
noth
ther
er in
in random
ra
and
ndom motion. 1m
3a. Am
mmo
moni
nium
um
Ammoniumm nitrate
nittra
ate and water 1m
3b. Endothermic.
Endoth
hermi
mic. 1m
When ammonium salts react with water, the reaction take in / absorb
energy from surrounding, causing surrounding temperature to drop /
decrease
decrease. 1m
2
www.KiasuExamPaper.com
318
4bi. No of moles of HNO 3 = concentration x volume
= 2.00 mol/dm3 x (500/1000) dm3
= 1.00 mol 1m
vigorously
But it is less reactive than Y since it reacts less vigoro
ro
ouus
ssly
ly w
ly with
itith
sodium hydroxide. 1m
6bii.
Experiment III
1m
3
www.KiasuExamPaper.com
319
6c. (1) Add excess lead (II) carbonate to nitric acid. 1m
(2) Filter to remove excess lead (II) carbonate.
Obtain the filtrate, lead (II) nitrate. (not necessary) 1m
(3) Heat lead (II) nitrate solution till saturation.
Leave to cool for crystals to form. 1m
(4) Filter out the crystals.
Pat dry between filter papers 1m
7a. Both propane and propene reacts with excess oxygen to produce
carbon dioxide and water. 1m
7b. 2C 3 H 6 + 9O 2 ĺ&2 2 + 6H 2 O
2C 3 H 8 + 10O 2 ĺ&2 2 + 8H 2 O
2C 3 H 6 + 6O 2 ĺ&2+ 2 O
2C 3 H 8 + 7O 2 ĺ&2+ 2 O
C3H6 + H2 Æ C3H8 either
ei
ith
ther
er eqn 1m
8c. X = ir
iron
ron (II)
(III) ch
chlo
chloride
l ride 1m
Y = ammo
moni
ammonium n um carbonate 1m
Explanation:
Possible answers are
(a) iron (II) chloride and ammonium carbonate
(b) iron (II) carbonate and ammonium chloride
However, iron (II) carbonate is an insoluble salt (both X and Y dissolve).
Also, no reaction when aq NH 3 added to Y so Y is an ammonium salt.
4
www.KiasuExamPaper.com
320
Paper 3 Section B : 3 Questions choose 2 (20 marks)
35
Cl has 18 neutrons/ mass number of 35.
37
Cl has 20 neutrons/mass number of 37. 1m
9bi. Strontium chloride conducts in molten state but not in solid state. no marks
The ions (Sr2+ and Cll-) are mobile in molten state. 1m
But they cannot move (and are fixed in positions) when in solid state. 1m
chlorine
Each calcium atom transfers two valence electrons to 2 chlo
ori
rine atoms.
ne a toms
to ms.
ms 1m
Ca2+ and Cl- ions are formed which are attracted by elect
electrostatic
cttro
rost
static
attiicc fforces
a orce
or ces
of attraction. 1m
9biii. molecules
Weak intermolecular force between chlorine molec
eccu es required
ulle
les rre
eq
quuiirre
red
ed 1m
little amount of energy to overcome. 1m
10ai. 4% 1m
10aii.
copper atoms
alum
minium
m atoms
aluminium atom
at omss
1m
10bi. Calcium
m > chromium
chromiium > ccobalt
ch obal
oba t > copper
al 1m
10bii. Blue
ue ssolution
olut
olu io
ion
n tu
turnss gr
gree
green.
een. 1m
B ow
Br own
Brownn so
soli
lids
ds fformed.
solids orme
or med.
d 1m
10ci. Y: 0 1m
Y(NO 3 ) 2 : +2 1m
10 ii
10cii. i a reducing
It is d i agent.
t no marks
k
5
www.KiasuExamPaper.com
321
11ai.
1m
Each monomer forms single bonds and joins with two other monomer
ers.
s.
monomers.
form the polymer, poly(methyl methacrylate). 1m
11bi. C 6 H 12 O 6 ĺ
ĺ&
& 2 H 5 OH + 2CO 2 1m
11bii.
1m
Also accept –
Use off blue litmus paper
X turns blue litmus paper red
Blue litmus paper remains blue
ue ifif its
its ethanol.
etha
ethano
n l.l
11biv. When
en the
the mixture
mix
ixtu
ture is heated, the yeast denatures and stops the reaction. 1m
6
www.KiasuExamPaper.com
322
Bukit Batok Secondary School
Sec 4 Express, 5 Normal (Academic)
Science (Chemistry)
1a. Nitrogen 1m
2a. filtration 1m
Changes to movement
The movement slows down, from mmmoving
oving ra
ov randomly
rand
n omly at high speeds to
sliding past one
e ano
noth
ther
e in random
another rand
ra ndom
om
m motion.
mot
o ion. 1m
3a. Ammoni
n um
m nitrate
Ammonium nittra
rate
te and
nd water
wat
ater 1m
3b. Endoth
her
e mi
mic.
c.
Endothermic. 1m
When amm
mmono iu
on
ammoniumium
m salts react with water, the reaction take in / absorb
energy from
m su
ssurrounding,
rrounding, causing surrounding temperature to drop /
decrease. 1m
www.KiasuExamPaper.com
323
2 mol of HNO 3 reacts with 1 mol of MgCO 3
1.00 mol of HNO 3 reacts with 0.500 mol of MgCO 3
It has / produce
produceses lesser
le
ess
sser
er H+ io ons p
ions er u
per nit volume compared to nitric acid.
unit 1m
(Frequency y of co
ollllis
isiions iis
collisions s le
less
sser
er tthus
lesser huss reaction is slower)
hu
6bii.
Experiment III
1m
www.KiasuExamPaper.com
324
7a. Both propane and propene reacts with excess oxygen to produce
carbon dioxide and water. 1m
7b. 2C 3 H 6 + 9O 2 ĺ&2 2 + 6H 2 O
2C 3 H 8 + 10O 2 ĺ&2 2 + 8H 2 O
2C 3 H 6 + 6O 2 ĺ&2+ 2 O
2C 3 H 8 + 7O 2 ĺ&2+ 2 O
C3H6 + H2 Æ C3H8 eithe
er eqn 1m
er
either
Explanation:
Possible answers
ansnswe
w rs sa re
are
(a) iron
on (II)
(II
II)) chlori
ride
de and
chloride and ammonium
amm
m onium carbonate
mm
( iron (II) carbonate
(b) carb
ca rbon
onat
ate
e and
an
nd ammonium chloride
Howevever,, iron
However, iro
on (II)
(II) carbonate
carrbo
bona
nate
te is an insoluble salt (both X and Y dissolve).
Also, no rreaction
e cttio
ea ionn wh
whenen aq NH 3 added to Y so Y is an ammonium salt.
www.KiasuExamPaper.com
325
Paper 3 Section B : 3 Questions choose 2 (20 marks)
35
Cl has 18 neutrons/ mass number of 35.
37
Cl has 20 neutrons/mass number of 37. 1m
9bi. Strontium chloride conducts in molten state but not in solid state. no marks
The ions (Sr2+ and Cl-) are mobile in molten state. 1m
But they cannot move (and are fixed in positions) when in solid state. 1m
Ca2+ and Cl- ions are formed which are attracted by electrostat
tic fforces
electrostatic orce
or cess
ce
of attraction. 1m
10ai. 4% 1m
10aii.
copper atoms
alumin
niu
ium
aluminiumm atoms
a om
at oms
1m
10bi
b.
10bi. chro
omi
m um > cob
Calcium > chromium bal
altt > co
cobalt opp
pper
e
copper 1m
10bii. solutio
ion
n tu
Blue solution turns gr
gree
een.
n.
green. 1m
Brown solids forme
med.
d.
formed. 1m
10ci. Y: 0 1m
Y(NO 3 ) 2 : +2
+ 1m
www.KiasuExamPaper.com
326
11ai.
1m
11bii.
1m
Also accept
Also acc c ept –
Use of bl blue
ue llitmus
ititmu
muss pa
paper
X tu
turn
ns bl
turns blue lit itmu
muss paper red
litmus
Blue
Bl e llitmus
ititmu
muss pa
paper remains blue if its ethanol.
Add su
Add suit
itable metal / metal carbonate
suitable
Iff ssusbstance
usbstance X, effervescence seen.
us
H 2 produced with caused lighted splint to extinguish with pop sound (if
use metal)
CO 2 produced which formed white precipitate in limewater (if use metal
carbonate)
11biv. When the mixture is heated, the yeast denatures and stops the reaction. 1m
www.KiasuExamPaper.com
327
www.KiasuExamPaper.com
328
Geylang Methodist School (Secondary)
Preliminary Examination 2018
5076/01
SCIENCE (PHYSICS/CHEMISTRY)
Paper 1 Multiple Choice Sec 4 Express
Sec 5 Normal (A)
Write your name, index number and class on the Optical Answer Sheet in the spaces
provided.
There are forty questions on this paper. Answer all questions. For each question there
are four possible answers A, B, C and D.
Each correct answer will score one mark. A mark will not be deducted for a wrong
answer.
[Turn over
www.KiasuExamPaper.com
329
2 GMS(S)/Sci(P/C)/P1/Prelim2018/4E/5NA
Silver chloride is made by adding 20.0 cm3 of aqueous silver nitrate to 20.0 cm3 of
dilute hydrochloric acid.
Which pieces of apparatus are needed to obtain solid silver chloride from aqueous
silver nitrate and dilute hydrochloric acid?
1 2 3 4 5
A 1, 2 and 4 B 1, 4 and 5
C 2, 3 and 4 D 2, 4 and 5
www.KiasuExamPaper.com
330
3 GMS(S)/Sci(P/C)/P1/Prelim2018/4E/5NA
first last
A filter dissolve evaporate crystallise
B dissolve evaporate crystallise filter
C dissolve evaporate filter crystallise
D dissolve filter evaporate crystallise
4 A mixture containing equal volumes of two liquids that mix completely but do not
react together is placed in the apparatus shown and heated until the thermometer
first shows a steady reading.
At which position will there be the highest proportion of the liquid with the lower
boiling point?
A
C
cold water in
heat
5 Which of the following changes will result in the particles moving at a higher speed?
A I2 (g) o I2 (s)
B CO2 (s) o CO2 (g)
C H2O (l) o H2O (s)
D N2 (g) o N2 (l)
www.KiasuExamPaper.com
331
4 GMS(S)/Sci(P/C)/P1/Prelim2018/4E/5NA
7 The nucleon number and proton number of an atom of P and atom of Q are shown.
P Q
nucleon number 85 80
proton number 37 35
electrical conductivity
melting point/oC
at room temperature
A 31 does not conduct
B 39 conducts
C 650 conducts
D 755 does not conduct
www.KiasuExamPaper.com
332
5 GMS(S)/Sci(P/C)/P1/Prelim2018/4E/5NA
9 A student thinks that element Q is a metal because it has a high melting point and a
high boiling point.
10 In the Periodic Table, caesium, lithium and sodium are in the same group.
12 When two aqueous solutions are mixed in a test-tube, a reaction occurs and the
test-tube feels hot.
2Mg + O2 Æ 2MgO
Which volume of oxygen is used in this reaction at room temperature and pressure?
www.KiasuExamPaper.com
333
6 GMS(S)/Sci(P/C)/P1/Prelim2018/4E/5NA
waste gases
raw materials
hot air
slag
molten iron
blast furnace
www.KiasuExamPaper.com
334
7 GMS(S)/Sci(P/C)/P1/Prelim2018/4E/5NA
16 Zinc powder was added to excess dilute sulfuric acid at room temperature. The
volume of hydrogen gas produced was measured over a period of time.
experiment 1
volume of
hydrogen experiment 2
gas
time
fraction use
A bitumen feedstock for the petrochemical industry
B diesel oil fuel for aircraft engines
C petrol fuel for engines in buses, lorries and trains
D petroleum gas fuel for cooking and heating
18 Many countries have taken measures to ensure that the amount of sulfur in
unleaded petrol and diesel fuels are kept low.
www.KiasuExamPaper.com
335
8 GMS(S)/Sci(P/C)/P1/Prelim2018/4E/5NA
A addition B cracking
C oxidation D substitution
20 The table shows the observations made when an organic compound X reacts with
aqueous bromine and acidified potassium manganate (VII).
reagent observation
aqueous bromine no change
acidified potassium manganate(VII) purple solution turns colourless
What is compound X?
End of paper
www.KiasuExamPaper.com
336
DATA SHEET
The Periodic Table Of Elements
Group
I II III IV V VI VII 0
1 2
H He
hydrogen helium
Key 1 4
3 4 proton (atomic) number 5 6 7 8 9 10
Li Be atomic symbol B C N O F Ne
lithium beryllium name boron carbon nitrogen oxygen fluorine neon
7 9 relative atomic mass 11 12 14 16 19 20
11 12 13 14 15 16 17 18
Na Mg Al Si P S Cl Ar
sodium magnesium aluminium silicon phosphorus sulfur chlorine argon
23 24 27 28 31 32 35.5 40
19 20 21 22 23 24 25 26 27 28 29 30 31 32 33 34 35 36
K Ca Sc Ti V Cr Mn Fe Co Ni Cu Zn Ga Ge As Se Br Kr
potassium calcium scandium titanium vanadium chromium manganese iron cobalt nickel copper zinc gallium germanium arsenic selenium bromine krypton
39 40 45 48 51 52 55 56 59 59 64 65 70 73 75 79 80 84
37 38 39 40 41 42 43 44 45 46 47 48 49 50 51 52 53 54
Rb Sr Y Zr Nb Mo Tc Ru Rh Pd Ag Cd In Sn Sb Te I Xe
337
9
rubidium strontium yttrium zirconium niobium molybdenum technetium ruthenium rhodium palladium silver cadmium indium tin antimony tellurium iodine xenon
85 88 89 91 93 96 – 101 103 106 108 112 115 119 122 128 127 131
55 56 57 – 71 72 73 74 75 76 77 78 79 80 81 82 83 84 85 86
lanthanoids
Cs Ba Hf Ta W Re Os Ir Pt Au Hg Tl Pb Bi Po At Rn
caesium barium hafnium tantalum tungsten rhenium osmium iridium platinum gold mercury thallium lead bismuth polonium astatine radon
133 137 178 181 184 186 190 192 195 197 201 204 207 209 – – –
87 88 89 – 103 104 105 106 107 108 109 110 111 112 114 116
Fr Ra actinoids Rf Db Sg Bh Hs Mt Ds Rg Cn Fl Lv
francium radium Rutherfordium dubnium seaborgium bohrium hassium meitnerium darmstadtium roentgenium copernicium flerovium livermorium
– – – – – – – – – – – – –
57 58 59 60 61 62 63 64 65 66 67 68 69 70 71
lanthanoids La Ce Pr Nd Pm Sm Eu Gd Tb Dy Ho Er Tm Yb Lu
lanthanum cerium praseodymium neodymium promethium samarium europium gadolinium terbium dysprosium holmium erbium thulium ytterbium lutetium
139 140 141 144 – 150 152 157 159 162 165 167 169 173 175
89 90 91 92 93 94 95 96 97 98 99 100 102 103
101
actinoids Ac Th Pa U Np Pu Am Cm Bk Cf Es Fm Md No Lr
actinium thorium protactinium uranium neptunium plutonium americium curium berkelium californium einsteinium fermium mendelevium nobelium lawrencium
– – – 238 – – – – – – – – – – –
The volume of one mole of any gas is 24 dm3 at room temperature and pressure (r.t.p.).
www.KiasuExamPaper.com
Geylang Methodist School (Secondary)
End of Year Examination 2018
Candidate
Name
Write your name, index number and class on all the work you hand in.
Write in dark blue or black pen.
You may use a pencil for any diagrams, graphs, tables or rough working.
Do not use staples, paper clips, highlighters, glue or correction fluid.
Section A
Answer all questions.
Write your answers in the spaces provided on the question paper.
Section B
Answer all questions. For Examiner’s Use
Write your answers in the spaces provided on the question
paper. Section A
/45
A copy of the Periodic Table is printed on page 14. Section B
www.KiasuExamPaper.com
338
2 GMS(S)/Sci(Chem)/P3/EOY2018/3E
Section A
Answer all the questions in this section.
Write your answers in the spaces provided on the question paper.
1 The diagram shows part of the Periodic Table. Only some of the elements are
shown.
H
C N F
Si P S Cl
Ti Fe Cu Zn As Br
(a) Answer each of the following questions using only those elements shown
in the diagram. Each element may be used once, more than once or not
at all.
…………………………………………………………………………...
(ii) forms an ion of Y which has only three completely filled shells of
electrons,
…………………………………………………………………………...
…………………………………………………………………………...
…………………………………………………………………………...
…………………………………………………………………………...
[5]
(b) Element X is a noble gas with two electron shells. Label this element as
X in the Periodic Table above. [1]
www.KiasuExamPaper.com
339
3 GMS(S)/Sci(Chem)/P3/EOY2018/3E
colourless gas B
black solid brown gas that relights a
A NO2 glowing splint
heat strongly
blue crystals
Cu(NO3)2 (s)
dissolve in water
pungent gas C
light blue
that turns damp
precipitate D
red litmus blue
Fig. 2.1
A ……………………………………………………
B ……………………………………………………
C ……………………………………………………
D ……………………………………………………
[4]
www.KiasuExamPaper.com
340
4 GMS(S)/Sci(Chem)/P3/EOY2018/3E
…………………………………………………………………………...
[1]
(ii) Construct the balanced chemical equation, with state symbols, for
the reaction between copper(II) nitrate solution with reagent X to
form copper(II) carbonate.
…………………………………………………………………………...
[2]
…………………………………………………………………………...
…………………………………………………………………………...
…………………………………………………………………………...
…………………………………………………………………………...
…………………………………………………………………………...
…………………………………………………………………………...
…………………………………………………………………………...
[3]
(c) Describe how hydrochloric acid and limewater can be used to show that
carbonate ions are present in copper(II) carbonate.
…………………………………………………………………………………...
…………………………………………………………………………………..
…………………………………………………………………………………..
…………………………………………………………………………………..
[2]
www.KiasuExamPaper.com
341
5 GMS(S)/Sci(Chem)/P3/EOY2018/3E
www.KiasuExamPaper.com
342
6 GMS(S)/Sci(Chem)/P3/EOY2018/3E
poly(propene) cap
and stopper
Fig. 4.1
(a) With reference to the arrangement of atoms, explain why alloys such as
steel are used as materials in the pen, instead of pure metals.
…………………………………………………………………………………..
…………………………………………………………………………………..
…………………………………………………………………………………..
…………………………………………………………………………………..
[2]
(b) Give one advantage and one disadvantage of recycling the materials
from this ballpoint pen.
…………………………………………………………………………………..
…………………………………………………………………………………..
…………………………………………………………………………………..
…………………………………………………………………………………..
[2]
www.KiasuExamPaper.com
343
7 GMS(S)/Sci(Chem)/P3/EOY2018/3E
…………………………………………………………………………...
…………………………………………………………………………...
[2]
…………………………………………………………………………...
…………………………………………………………………………...
…………………………………………………………………………...
…………………………………………………………………………...
[2]
Write a chemical equation for the reaction. State symbols are not
required.
…………………………………………………………………………...
[1]
www.KiasuExamPaper.com
344
8 GMS(S)/Sci(Chem)/P3/EOY2018/3E
(a) Answer each of the following questions using the letters that represent
each compound.
…………………………………………………………………………...
…………………………………………………………………………...
…………………………………………………………………………...
[3]
[1]
…………………………………………………………………………...
…………………………………………………………………………...
[2]
www.KiasuExamPaper.com
345
9 GMS(S)/Sci(Chem)/P3/EOY2018/3E
[2]
…………………………………………………………………………...
[1]
chemical test
…………………………………………………………………………...
…………………………………………………………………………...
…………………………………………………………………………...
…………………………………………………………………………...
[2]
www.KiasuExamPaper.com
346
10 GMS(S)/Sci(Chem)/P3/EOY2018/3E
Fig. 7.1
…………………………………………………………………………...
[1]
[2]
www.KiasuExamPaper.com
347
11 GMS(S)/Sci(Chem)/P3/EOY2018/3E
Section B
8 Fluorine is the lightest halogen and exists as a highly reactive pale yellow
diatomic gas at room temperature and pressure. Fluorine is found in nature in
the form of calcium fluoride crystals, called fluorite.
(a) The following equation describes the reaction between fluorine gas and
water.
Given that 48 cm3 of fluorine gas reacts with excess water, calculate the
volume and mass of oxygen gas produced at room temperature and
pressure. [2]
(b) Draw and label the electronic structures of fluorine gas and calcium
fluoride.
[Proton numbers: F, 9; Ca, 20] [4]
(c) Use these structures to explain why, at room temperature and pressure,
calcium fluoride is a solid and fluorine is a gas. [4]
9 (a) The reaction of metal X with water places it between calcium and iron in
this order of reactivity. Explain why X would displace copper if added to
a solution of copper(II) sulfate. [2]
(b) Aluminium does not react with cold water. Does this give a true indication
of the reactivity of this element? Explain your answer. [2]
www.KiasuExamPaper.com
348
12 GMS(S)/Sci(Chem)/P3/EOY2018/3E
(a) (i) With the aid of a diagram, design and describe an experiment in
a laboratory to show how the rate of reaction between these two
substances depends on the particle size of calcium carbonate.
[4]
(ii) Describe the measures you would take to ensure that your
experiment is fair. [2]
(iii) State and explain how the rate of reaction between two
substances is affected by the particle size of one substance.
Use your knowledge of reacting particles in your explanation. [2]
(b) Increasing the concentration of acid can change the speed of a reaction.
State and explain how it affects the speed of the reaction. Use your
knowledge of reacting particles in your explanation. [2]
End of Paper
www.KiasuExamPaper.com
349
13 GMS(S)/Sci(Chem)/P3/EOY2018/3E
BLANK PAGE
www.KiasuExamPaper.com
350
1 GMS(S)/Sci(Chem)/P3/Prelim/2018/4E/5NA
Geylang Methodist School (Secondary)
Preliminary Examination 2018
Secondary 4E5N Science (Chemistry) Answer Scheme
Paper 1
1 2 3 4 5 6 7 8 9 10
C D D A B D B A C C
11 12 13 14 15 16 17 18 19 20
D B B A D A D B B C
Paper 3 Section A
Question Marking Point Marks
Marks
1 (a) (i) Fe 1
(ii) Cl 1
(iii) Zn 1
(iv) H or N 1
(v) S 1
(b) Position of X: element (N
0 2nd el
X Group 0, (Ne)
e) 1
Total: 6
2 (a) A – copper(II) oxide 4
B – oxygen gas (1M
C – ammonia gas each)
D – copper(II)
co
opp
pper
er(I
( I) hydroxide
hyd
ydro
roxi
xide
de
(b) (i) An
Any
ny soluble
solubl
so ble
e carbonate
carb
ca rbon
onat
a e e.g. 1
sodium
m ccarbonate
arbo
ar bona
nate
te / potassium carbonate
(ii) Cu(NO
Cu(N
Cu (NO3)2 (a
(aq) + Na2CO3 (aq) Æ CuCO3 (s) + 2 NaNO3 (aq) 2
Balanced
Bala
Ba anc ed chemical equation – 1
nced
State
Stat
Statee symbols – 1
Total: 12
www.KiasuExamPaper.com
351
2 GMS(S)/Sci(Chem)/P3/Prelim/2018/4E/5NA
Question Marking Point Marks
3 (a) No. of moles of titanium = 12 000 / 48 = 250 moles 1
No. of moles of MgCl2 = 250 x 2 = 500 moles
Mass of MgCl2 = 500 x (24 + 35.5 x 2) = 47500 g = 47.5 kg 1
(b) % mass of Ti in FeTiO3 1
= 48 / (56+48+16x3) x 100% = 31.57894 %
Mass of FeTiO3 to produce 12kg of Ti 1
= 12/31.57894 x 100 = 38 kg
Total: 4
4 (a) Alloys are harder than pure metals. 1
The different sized atoms disrupt the orderly arrang
arrangement
gem
emen
entt of
of 1
atoms,
Hence making it difficult for the metal atoms to slide
slid
sl ide
eoover
ver on
onee
another.
(b) Advantage: Any 1
x Conserves finite resources of crude
crud
ud
de oil/metal
oiill//m
o meetta
al ores.
orres
o es.
x Reduces use of landfill.
x Less problems caused from disposal.
x Materials used are non-biodegrada able.
non-biodegradable.
x Less expensive than producing from raw materials.
mate eria
ialss.
Disadvantage: Any 1
x Difficulty / high cost to se
separate tthe
he m
materials.
ater
at e ials.
x Not all materials can n be recy
recycled.
cycl
cled
ed.
Total: 4
5 (a) (i) They all have 2 valence
vale
lenc
nce
e electrons.
e ec
el ectr
tron
o s. 1
(ii) They all
all have
hav
ave complete/full
comp
co mple
lete
te/f
/full valence shell. 1
(b) (i) x Chlo
Ch
Chlorine
l rine
ne has
has lower
low
ower
e boiling/melting point than astatine/Z. Any 2
OR Chlorine
Chlo
Ch lori
rine
ne is
is a gas at room temperature but astatine/Z
iss a solid
sol
olid
id at
at room temperature.
x Ch
Chlorine
C lo
ori e is greenish yellow in colour but astatine/Z is
rine
black
blac
bl ackk in colour.
x Ch
C
Chlorine
lorine is more reactive than astatine/Z.
(b) (ii) Cl2 + 2 NaZ Æ Z2 + 2 NaCl 2
Total: 6
6 (a) (i) A and D 1
(ii) E 1
(iii) B 1
www.KiasuExamPaper.com
352
3 GMS(S)/Sci(Chem)/P3/Prelim/2018/4E/5NA
Question Marking Point Marks
(b) (i) 1
Propor
Proportional
Prop orti
t on
onalal drawing
drawing – 1
Appropriate
Appr
Ap prop
opriiatte labels – 1
Total: 6
www.KiasuExamPaper.com
353
4 GMS(S)/Sci(Chem)/P3/Prelim/2018/4E/5NA
Section B
Question Marking Point Marks
8 (a) Volume of oxygen = 48/2 = 24 cm3 1
No. of moles of oxygen = 0.024 / 24 = 0.001 1
Mass of oxygen = 0.001 x 32 = 0.032 g.
(b) 4
Fluorine gas
Correct sharing – 1
Correct number of valence electrons – 1
Calcium fluoride
Correct calcium ion – 1
Correct
C
Co rrect fluoride ion – 1
wh
which
hic
ichh requireses small
smal
sm alll am
amount of energy to overcome/ 1
has lo
llow
w bo
boil
boiling
ilin
ingg po
point hence it is a gas at r.t.p.
Calciu
Calcium
Calc ium
m fl
fluo
fluoride
uoriide is an ionic compound. 1
It h a strong
as
has stro
str ng electrostatic forces of attraction between the
ions
io
ons
Total: 10
www.KiasuExamPaper.com
354
5 GMS(S)/Sci(Chem)/P3/Prelim/2018/4E/5NA
Question Marking Point Marks
9 (a) X is more reactive than iron, which is more reactive tha
than
han 1
copper.
Hence X is more reactive than copper. 1
(b) No. Aluminium reacts with oxygen to form a layer
lay
a err of
of 1
aluminium oxide
which is unreactive and hence prevents
preven
en
nts
ts the
th
hee aluminium
alu
lummiini
n um metal
met
etal
al 1
from reacting with water.
(c) Coke burns in air to form carbon dioxide 1
Carbon dioxide reacts with more coke
e to form carbon
carbo
bonn 1
monoxide.
Carbon monoxide reacts with
with
h iron(III)
iron(IIII) ox
oxid
oxide
idee to
t form iron and 1
carbon dioxide.
Limestone decomposes
decomposses to
o form
for
ormm calcium
calc
ca lciu
i m oxide and carbon 1
dioxide.
Calcium oxide reacts
reac
acts
ts w
with
itith sand (silicon dioxide) to form 1
calcium
m si
sili
icate
e/m
/mol
olte
ten
silicate/molten n sl
sslag.
ag.
Eq
Equation
qua
uati
tion of
of Redox
Redo
Redoxx reaction
re Any 1
x C + O2 Æ CO2
x CO2 + C Æ 2 CO
x Fe2O3 + 3 CO Æ 2 Fe + 3 CO2
Total: 10
www.KiasuExamPaper.com
355
6 GMS(S)/Sci(Chem)/P3/Prelim/2018/4E/5NA
Question Marking Point Marks
10 (a) (i) 4
Markrking
Marking g po
p ints
ts (1
points (1MM ea
each
each):
c ):
x Appropriate
Appr
Ap p opriatate
e di
diag
diagram
agra
am with suitable labels
x Us
U
Usee of ap
appr
prop
opriate mass of calcium carbonate and volume
appropriate
of a
acid
cid
ci
x Us
Usee of different
different particle size of calcium carbonate (lumps vs
powdered
powd
po wdered or large lumps vs small lumps) in two
experiments.
ex
x Observation or change that is measured to compare the rate
of reaction (time taken for effervescence to stop / time
taken to collect fixed volume of gas)
(ii) x Concentration of dilute hydrochloric acid used is the same. Any 2
x Mass of calcium carbonate used is the same.
x Temperature of both experiments is the same.
Reject:
Equal volume of dilute hydrochloric acid
Shake the acid to ensure reaction is complete
Repeat experiment and obtain average of results.
(iii) The smaller the particle size, the larger the total surface area 1
for particles to collide,
resulting in increased number of effective collisions and hence 1
higher speed of reaction. (opposite is true)
(b) The higher the concentration of acid, the higher the speed of 1
reaction
The more concentrated the acid, the more acid particles per 1
unit volume, resulting in increased number of effective
collisions
Total: 10
www.KiasuExamPaper.com
356
www.KiasuExamPaper.com
357